Fry On the Problem Of Evil, Part One

My favorite philosophical conundrum has been back in the news lately, thanks to a recent interview with British actor Stephen Fry:

Asked by the interviewer what he would say to God were he to discover, after his death, that He existed, Fry replied:

I'd say, bone cancer in children? What's that about? How dare you? How dare you create a world where there is such misery that is not our fault. It's not right. It's utterly, utterly evil. Why should I respect a capricious, mean-minded, stupid God, who creates a world that is so full of injustice and pain? That's what I'd say.

Skipping ahead a bit he adds:

Atheism is not just about not believing there is a God, but, on the assumption that there is one, what kind of God is He? It's perfectly apparent that He's monstrous, utterly monstrous, and deserves no respect whatsoever. The moment you banish Him your life becomes simpler, purer, cleaner and more worth living in my opinion.

That's all very well said, both in tone and substance. There is, of course, a voluminous philosophical literature trying to explain away the problem of evil, and it's all very scholarly and erudite. But Fry's simple anger is a useful reminder that it is also all very tawdry and unseemly. The various proposals theists make for absolving God are so unequal to the magnitude of the problem that it reflects poorly on them for putting them forward at all.

Moreover, when treated as an abstract philosophical problem we generally assume that we are discussing a tri-Omni God. That is, God is assumed to be omnipotent, omniscient, and omnibenevolent. We should keep in mind, though, that something a good deal less than omnibenevolence is needed for the problem to go through. We need only assume that God has some sort of conscience to have a serious problem of evil. Humans are not omnibenevolent, after all, but most of us regard it as obvious that you don't allow children to suffer from dread diseases if you have the power to stop it.

Predictably, Fry's answer provoked a great deal of response. Much of it was flatly stupid, such as this essay by Tim Stanley in the British newspaper The Telegraph. He writes:

Not only has theology dedicated itself for thousands of years to unpicking that problem but the answer to it is there in the very Bible itself. Since Adam and Eve ate the apple, we've been living in a fallen world full of pain. God granted us free will not only to do bad things but also good things – like finding a cure for cancer or caring for those dying from it.

Terrible things happen because of a) random acts of nature, b) the intervention of the Devil or c) the corruption of man. I'm not saying anyone has to believe what I write, but please don't act like it's never been said before or that the answer to Fry's facile question doesn't exist. Dear Stephen imagines that he's the first person in history to wonder why folks suffer. He's not. He is, however, strangely upset about something that he doesn't even believe in. Who gets angry about an imaginary conversation?

At what point, exactly, did Fry pretend he's the first person to wonder why folks suffer?

I suspect all those theologians referred to in the first paragraph are wishing Stanley would stop helping them. In explaining evil by reference to Adam and Eve, Stanley is throwing his hat in with the young-Earth creationists. Once you accept evolution and the antiquity of life, you realize that this explanation simply doesn't work. Evil and suffering long predate the appearance of human beings, after all. Stanley's other off-the-cuff suggestions for resolving the problem fare no better, making it clear that it is not Fry who is being facile.

Perhaps fearing that his previous remarks were insufficiently stupid, Stanley dials it up a notch with his closing sentences. He is pretending, for some reason, not to understand the concept of a hypothetical question. Having decided that the evidence is against God's existence, Stanley now thinks atheists are barred even from imagining how we might react were we shown to be wrong. His logic is murky, to put it kindly.

A more serious response came from Vincent Torley over at Uncommon Descent:

You're accusing God of wrongful creation – making a world in which suffering can occur, and then populating it with people. But by the same token, you would have to say that two parents who chose to bring a child into the world, knowing that it was liable to inherit a life-threatening form of cancer, would be wronging that child, simply by procreating it. And to that I say: how dare you tell someone that they have no right to create a human being? Whether it be short or long, life, in itself, is a good thing.

“I've got another question for you. Suppose instead that the parents in the hypothetical scenario above were told by their doctor that while any child they chose to bring into the world would probably get cancer, the cancer would not be terminal. Suppose that it could be treated over the course of several months, by a very painful course of chemotherapy, but that after that, their child would enjoy a full and happy life. Surely even you would concede that it would be morally justifiable for the parents to bring a child into the world, in this case. Now suppose, hypothetically, that the child's full and happy life turned out to be an indefinitely long one, because scientists had recently discovered a way to make people live forever. In that case, no-one would say that the prospect of getting bone cancer would constitute a valid reason not to create a child: it would be a treatable illness. All right, then. Heaven is forever. How, then, can you accuse God of being unjust?”

Unlike Stanley, Torley at least attempts a thoughtful response to Fry's question. His reply does not work, however.

The most serious flaw in Torley's argument is that is non-responsive to Fry's question. Speculating about possible moral dilemmas faced by human parents does nothing to explain why God allows suffering and evil. God is omnipotent, after all, but human beings are not. If the question is, “Why would God create a world with horrible suffering, cruelty, and evil?” the answer is not, “It might be ethical for humans to bring children into the world even knowing that those children will suffer.” The questions confronting the parents in Torley's analogies are just entirely different from the one that is being put to God in the problem of evil.

So, even if Torley's analysis of his analogies was correct, his response to Fry would still be entirely inadequate. As it happens, though, his analysis is not correct.

With regard to the first analogy, his assertion that, “Whether it be short or long, life, in itself, is a good thing,” is so absurd, and so monstrous in its consequences, that it is hard to believe he is serious. There are afflictions out there far worse even than bone cancer. Let us imagine a child born with maladies so severe that his life, in its entirety, will consist of a few months of suffering, hooked up to machines, before inevitable death. How can you possibly say there is something inherently good about that child's life? If would-be parents somehow knew ahead of time that their decision to have a child would unavoidably lead to that, how could you not judge them harshly for going through with it anyway?

Torley acts as though the decision to bring a child into the world is always morally praiseworthy. As he tells it, if you think that certain people should not become parents you are, necessarily, being morally obtuse. But this is all just absurd. A person who signed a contract knowing they were unable to fulfill its terms would be commonly thought to have done something unethical. But there is a tacit contract in every decision to bring a child into the world. That contract says, among other things, that you will tend to that child's emotional and physical needs. In fact, you will place those needs before your own. If you know you are unable to provide for those needs, (and, let's be honest, many parents are not) then it is absolutely unethical for you to bring a child into the world. An awful lot of social dysfunction is the direct result of irresponsible people bringing children into the world they are unable to raise. I don't understand why Torley finds those people morally admirable.

His second analogy is scarcely better. If you punch someone in the nose for no reason, and then later give him a lollipop, you have not absolved yourself of the wrong that you did. If someone asks you, “Why did you wrong that person?” you can never plausibly answer, “I was nice to him later.” Using heaven as an answer to the Problem of Evil commits precisely this fallacy. If we ask of God, “Why do you allow children to suffer from bone cancer,” we would not accept as an answer, “Later I allow those children to have eternal life in heaven.”

Parents know that their children will experience hard times in their life. Unlike God, they have no control over that. For them the decision is whether or not the good in their life will outweigh the bad. That is not at all the decision faced by God in creating the world. That is why most philosophers who address the problem of evil take an entirely different approach from the one Torley has chosen. They try to explain why God in fact must allow great suffering as the price for achieving some greater good. I don't think they are successful in that project, but they, unlike Torley, understand what needs to be done.

Enough of that. There was one further response to Fry that I found especially interesting. Since this post has gotten rather long, we shall save it for another time.

More like this

The gist of this article seems to be
If there is a God of the universe, you’re better than him.

By See Noevo (not verified) on 22 Feb 2015 #permalink

"But Fry’s simple anger"
Yeah - I like to stress that the Problem of Evil is about something christians claim, but tend to forget immediately when discussing the issue: compassion and empathy. SN above immediately confirms this.

@SN: "If there is a God of the universe, you’re better than him."
And why would this be impossible? If I read some stories in the OT I think it very easy to claim that I'm better than the god there described. I never ordered a genocide for instance.

If there is a God of the universe, you’re better than him.

Not at all. The gist of the article is that if there is a God with the attributes Christians say He has, then it is hard to understand why He would allow tremendous evil and suffering. Do you have anything to offer that would make it easy to understand?

“Once you accept evolution and the antiquity of life, you realize that this explanation simply doesn’t work. Evil and suffering long predate the appearance of human beings, after all. Stanley’s other off-the-cuff suggestions for resolving the problem fare no better, making it clear that it is not Fry who is being facile.”

I think it is the evolutionists and atheists who are being facile, for several reasons:
1)In evolution, there is no such thing as evil or good. So, why are you concerned about “evil”?
2)In atheism, there is no objective morality of evil or good. So, why are atheists concerned about “evil”?
3)In evolution, we’re told that this random, non-rational process continually advances a biological robustness which includes longer life and successful reproduction (along with, of course, a virtually limitless number of new complex functions, organs, and organisms). If death and disease (and deadly, dire human designs) work against species’ success, why hasn’t the problem of “evil” been solved by evolution in the last several billion years?

By See Noevo (not verified) on 22 Feb 2015 #permalink

To Jason Rosenhouse #3:

Me: “If there is a God of the universe, you’re better than him.”

JR: “Not at all. The gist of the article is that if there is a God with the attributes Christians say He has, then it is hard to understand why He would allow tremendous evil and suffering. Do you have anything to offer that would make it easy to understand?”

OK. The gist of the article is not that you’re better than the God of the universe, but rather that you’re better than Jesus Christ.

By See Noevo (not verified) on 22 Feb 2015 #permalink

For me, the weakness of such discussions is that they concede the reality of gods. All my outrage at the hideous cruelty and monstrosity of gods tends to dissipate into indifference when I think again that they are not real, after all. They are nothing more than fictional characters, like blood-sucking vampires or flesh-eating zombies, and the obscenity of their behavior in the god stories is fictional. It is the product of human imagination, not reality.

This is not to deny the presence of suffering in the world. It is only to deny that imaginary gods have any hand in it.

@See Noevo

What makes you think that good and evil cannot exist without gods?

Good and evil are the product of human morality. Imaginary supermen have no part in it.

To phhht #7:

“What makes you think that good and evil cannot exist without gods? Good and evil are the product of human morality. Imaginary supermen have no part in it.”

You are an atheistic evolutionist. Or at least you would claim to be. Unfortunately, your opinions have no credibility or integrity since they are not at all consistent with your materialistic evolutionary world-view. Because as I said, in evolution, of which you would say humans are a product, there is no such thing as good or evil, right or wrong. There is only life and death. Life is no better than death, because there is no “better” when there is no objective good or bad to begin with. In evolution, there is only what “is”. And it ain’t “much”.

By See Noevo (not verified) on 22 Feb 2015 #permalink

[i]n evolution, of which you would say humans are a product, there is no such thing as good or evil, right or wrong. There is only life and death. Life is no better than death, because there is no “better” when there is no objective good or bad to begin with. In evolution, there is only what “is”.

Yes, that is my understanding of how evolution operates, in that there is no invisible thumb of a god on the scales. Gods aren't necessary.

That doesn't mean that we human beings cannot recognize the sweetness of life or the suffering of death from the outside, so to speak. Whatever gave you such a silly idea?

I'll just pop in to remind folks that See No Evolution is a Young Earth Creationist, more or less.

No evidence will suffice.

By MobiusKlein (not verified) on 22 Feb 2015 #permalink

Pro tip for SN:
Stop making lists about atheists. Your assertions only demonstrate your ignorance.

Your list in this thread implies that evolution and atheism are philosophies that are followed like you follow your religion. Evolution is a process that produced the variety of life on Earth. Atheism is the lack of belief in a god. Neither one includes nor compels a position on evil. Most discussion of the topic by atheists (or evolutionists, which are not synonymous - and I use that term only in polite deference to you as my interlocutor) is in reaction to the claims of religious believers. Mr. Fry's comments are no exception.

By Walt Jones (not verified) on 22 Feb 2015 #permalink

@See Noevo

When you say
[I]n evolution, of which you would say humans are a product, there is no such thing as good or evil, right or wrong. There is only life and death. Life is no better than death, because there is no “better” when there is no objective good or bad to begin with. In evolution, there is only what “is”.

you seem to suggest that evolution cannot operate without some notion of good and evil, right and wrong.

Why do you think that?

I assure you it is not true. Evolution works without any such notions - just like an internal combustion engine does, say. Gods and goddy morality are wholly unnecessary to either of those processes, and as far as anybody can tell, there are no gods involved.

Jason,

It is obvious that you do not read the comments to your articles beyond the first day or so. I have addressed this issue in detail, and was continuing to address it in the "Proving God Exists" thread, which has just closed.

Fry goes off the rails in his third indignant question to God:

"How dare you create a world where there is such misery that is not our fault?"

What he and you are up against, is a story that predates your indignation by multiple millenia. Your dismissal of the explanation offered by the Garden of Eden story is particularly blind.

As an evolutionist, you cannot imagine a world where there was no Evil. And yet this is precisely what the Genesis account claims that God created. But rather than engage this in any meaningful way, you dig deep into your vast reserve of factual evidence and declare with certainty:

"Evil and suffering long predate the appearance of human beings, after all."

You have two problems. First, this is pure speculation. It is based on the assumption that Evolutionary Theory is correct. But there is not a shred of observational evidence, nor even a good ancient myth that suggests this might be the case. To the contrary, Pandora's box would be just one example of how different mythology tells the exact same story as Genesis, and thus contradicts your evolutionary presumption.

As an aside, it is this precise point - that evolution requires an endless series of suffering and death, exactly contrary to the Creation story - that causes Young Earth Creationists to reject any notion of Theistic Evolution.

You are not a YEC at all, and yet you reach the exact same conclusion. The problem for Creationists is not whether the details in Genesis are literally and precisely true, but that allowing Evolutionary Theory into the mix immediately introduces suffering and death, and hence Evil, prior to the appearance of mankind.

Which is why it is so important to highlight the point that the Genesis account has Man created first, with Evil introduced later. Throw out this little detail, and indeed your argument is correct and the Bible has no answer whatsoever to offer for the problem of Evil. God is just one Evil dude.

Your second problem is that, while your logic is correct so far as Evolutionary Theory goes, you have ignored or missed this fact that the Genesis account runs directly contrary to it. The Garden of Eden story does in fact explain the problem of Evil in a logically consistent manner, and it lays the blame at our feet, much to the chagrin of Mr. Fry.

First, it says that God made everything Good.
Next, it says he created Man, as part of his perfect/good creation.
Finally, it says that Man made a decision that introduced Evil, suffering, and Death, into the world.

So if Mr. Fry meets God, he has not considered what God has already said on the subject. He is approaching his conversation purely from an atheistic and evolutionary presumption. He has been unable to set aside that set of presumptions as part of his questions to the God he has just discovered is real. In other words, his presumption have just been proven wrong, and yet he still stands on them when formulating his questions for God.

If it turns out that God exists, then you can't turn around and blame him because you ignored his instruction manual for your whole life. So the only meaningful answer would have to import all the presumptions inherent in the discovery that God does exist. We can discuss that hypothetically now, rather than waiting for you to meet your Maker, but you have to see that you cannot pick and choose your presumptions. Once we start discussing God, even hypothetically, then you must import all that goes with him, including the Garden of Eden story. Until you have engaged that story, a casual dismissal of it based upon an evolutionary presumption is either ignorant, lazy, irresponsible, illogical, or all of the above.

Otherwise, you and Mr. Fry are embracing Evolutionary Theory, and then blaming God for all the Evil, suffering and Death that it entails.

Had you followed my posts, you would know that I stated that God allowed Man to choose Evil, because it was part and parcel of giving us Free Will. Sean Samis and I are presently in the middle of trying to sort out what Free Will actually is, and where we each stand on the subject. Stay tuned.

Meanwhile, I suggest you respond directly and tell me where I am wrong before you go calling someone like Stanley stupid. Otherwise it reflects rather poorly on you.

There is another point that you missed in what Torley said. Indeed, he may have missed it himself. But you could not miss it had you read my posts starting in the thread, "The Meaning of Life". In there I show that the teachings of Jesus tell us specifically how to reverse the mistake of Adam and Eve, and thus how to bring us back to that state of perfection described in the Garden of Eden. I am not talking the pie-in-the-sky version of Heaven that comes to mind when Torley uses the term. I am talking about paradise on Earth, which is what the Creation story clearly describes.

And this is why this is such an important issue. Evolutionary Theory can only tell us that we are locked into a cycle of birth, suffering and Death, forever. The Creation account in Genesis, combined with the teachings of Jesus, provides the only alternative model I am aware of: A method for undoing the Evil in this world. A means for correcting the original mistake.

And so whether Torley was clear on this or not, his point is really the same as that made by the Apostle Paul in Romans:

"I reckon that the sufferings of this present time are not worthy to be compared with the glory which shall be revealed in us."

And again in 2 Corinthians:

"For momentary, light affliction is producing for us an eternal weight of glory far beyond all comparison."

So Stanley is right: the question has been debated since forever, and the answers are already out there. Address the answers before you go applauding the next lazy atheist for his self-righteous indignation.

@Gordon

I wonder, before we get into all your claims, if you would be good enough to explain just what it is that makes you think gods are real.

I hope you will not answer "faith," since that simply reasserts that you DO think gods are real.

I hope you will not answer "the bible," since that is circular.

So what does convince you that gods are real?

Maybe you could explain that for me.

Gordon--

I read your comments, I just mostly don't find them interesting enough to reply to. Your present comment is a case in point. Yes, obviously, you can reject modern science (not just evolution but the geological evidence for the antiquity of life) and thereby cling to the YEC explanation for evil. I said as much in the post, and I devote a chapter to it in my book. So what?

Even if we were to go the YEC route, however, it is very debatable whether the problem has been resolved. Is it really an appropriate response to human sin to allow children to be tortured with dread diseases? The punishment seems out of proportion to the crime.

I must sympathise with Mr Fry and your support of his angry veiwpoint.
Man is a moral being ( when it suits him) be he believer or atheist, he cannot escape his nature. This is not true of us all Robert Hare points out he believes 1% of the American population are pyschopaths; he spent years studying them.

By magnocrat (not verified) on 23 Feb 2015 #permalink

I'm amused by Stanley's attempt to make something out of the fact that Stephen Fry was saying nothing new. Fry was asked a question, and he gave a good answer. So what if it isn't a new one? If I'm asked "what's 2+2?", should I be pilloried for giving the tired old answer "4"?

That said, the problem of suffering is worth bringing up occasionally, even without being invited. Though there are dogmatists like Stanley who will find excuses to avoid facing up to the problem, I think there are plenty of more thoughtful theists who find it discomfiting. And drawing their attention back to the problem may help move them a little further towards a more skeptical position.

By Richard Wein (not verified) on 23 Feb 2015 #permalink

First, as an aside, for eric and sean samis since the "Proving God Exists" comment thread is now closed I continued the discussion on my own blog. I'm not sure about the etiquette of posting that sort of link here, but my blog shouldn't be hard to find.

On this topic, both of these responses aim at challenging the idea that the morality of goodness of an action is determined by whether one allows others to suffer or not. Stanley's can be boiled down to "God never promised us that this world wouldn't have suffering, and in fact rather promised us that it would.". Torley's, on the other hand, says that creating something into an environment where it can suffer isn't morally bad, or else all parents are immoral for having children. And if that suffering will eventually go away, then bringing about existence that starts with suffering but ends with eternal bliss isn't a bad thing either,

Since in Stoic morality suffering itself is neither virtuous nor vicious, I'm in some sense sympathetic to those sorts of claims. It does seem like the proponent of the Problem of Evil has to do a lot more than simply say "People suffer" to have this be a problem for a being that knows more than we do ... including about what makes an action moral or immoral, or about whether suffering in this world serves a purpose. I don't say that the Problem of Evil has been shown to not be one, but more that it hasn't really been shown to be one either.

By Verbose Stoic (not verified) on 23 Feb 2015 #permalink

JR:

Evil and suffering long predate the appearance of human beings, after all.

Just an amusing aside, Demski gets around this by claiming the Fall was a retcon/retroactive. IOW there was this young Earch created in accordance with a literal Genesis, but when A&E ate the apple, they caused it to be replaced by an old earth where evolution etc... took place. Its a variation of omphalos logic, but more amusing than the standard form just because of its sheer ridiculousness.

SN @4:

I think it is the evolutionists and atheists who are being facile, for several reasons:
1) In evolution, there is no such thing as evil or good. So, why are you concerned about “evil”?

You seem to be repeating Tim Stanley's error, i.e., not recongizing that atheists can answer hypothetical questions about your God. Of course we can.

Gordon @13 :

First, it [Genesis] says that God made everything Good.

A claim flatly contradicted by the fact that God created a tree for which eating the fruit would bring death and destruction to all the cosmos.
If I build a house and hook it up to a nuclear bomb placed under the basement, which can be triggered by a big red button in the house labeled "don't push," that is not capital-G Good, it's capital-I Insane. Yet that is what Genesis says God did.

So if Mr. Fry meets God, he has not considered what God has already said on the subject. He is approaching his conversation purely from an atheistic and evolutionary presumption.

We do pay attention to what Genesis says on the subject. As my example above shows, one need not assume or accept evolution at all to be able to point out that the being Genesis describes does not act in an entirely benevolent manner.

A method for undoing the Evil in this world. A means for correcting the original mistake.

You now seem to be contradictingn some of your earlier claims on the other post. There you claimed that it was flatly impossible to have free will without evil. Now you claim that if we follow Jesus' instructions, our free-willed selves can achieve an evil-free paradise. Which is it?

phhht,

For me, the most compelling argument is the one from Design. Funny where the discussions on that issue have ended so far.

Jason,

Don't worry, this will be the last thread I comment on. Once it closes, you will hear no more from me. Sorry, I have not read your book.

Bone cancer in children is a cheap shot, and avoids the real issue. They are the collateral damage from the real problem. But an example that tugs on the heart strings, and so a great way to win an argument without actually having one!

We all know that children suffer greatly if their parents are drug addicts, or alcoholics, or desperately poor. The children did not ask to be born to bad parents. Why should those children suffer while others are born into good, caring homes? But the consequence of Evil is simply unpredictable in this way. A drunk driver might kill someone in another car, but he is indiscriminate in who he kills. They just happened to be in the wrong place at the wrong time.

So blaming God for children suffering is just an extension of the original question, but now with an emotive spin to it. You, Mr. Fry, Eric and the two Seans appear to want a world without cause and effect. You want one person to be able to do stupid or Evil things, yet have it have absolutely no effect on anyone else, especially their children. Eric wants to be able to jump off of high mountains and not die when he hits the ground. Then he wants to do it again, and again, and again. God owes it to him.

You call this issue your "favorite philosophical conundrum", yet you appear only interested in griping about it, not exploring it fully.

Let me frame it for you this way. If God is to blame for the problem of Evil, then there is nothing we can do about it. We're simply screwed, and that's it. If we are to blame, then this fact immediately empowers us to undo the mistake, once we understand it.

Atheism is disempowering, because it blames God. By denying the existence of God, you leave yourself in a position where there is undeniably Evil in the world, and yet there is nothing you can do about it.

The biblical narrative is empowering, because it explains the problem, and tells us what to do about it.

By Gordon (not verified) on 23 Feb 2015 #permalink

In reply to by eric (not verified)

Gordon:
I can't speak for phhht, but I ask only for the same level of evidence for your god as I accept for evolution. As phhht notes, the bible saying a god exists is not evidence. I don't accept evolution based on the Origin alone, but on the research conducted in the 156 years since its publication - independent research in geology, biology, astrophysics, and other fields, not just analysis of Darwin's text. If you consider that speculative, it should be easy for you to provide a similarly varied set of sources and evidence for your claims (and I mean positive claims, not just assertions about problems with evolutionary theory). .

By Walt Jones (not verified) on 23 Feb 2015 #permalink

It does seem like the proponent of the Problem of Evil has to do a lot more than simply say “People suffer” to have this be a problem for a being that knows more than we do …

Well, a theist can invoke the "God's ways are mysterious" deffense against theodicy, but that is intellectualy and philosophically dissatisfying, because it can be used to claim any event at all is consistent with omnibenevolence (by saying: we just don't understand the consistency). So we should probably discount it, and say that any defense has to explain the presence of evil and suffering without resorting to the claim that it's just really good in a way we can't possibly understand.

Having said that,
1)the claim of an evil-free heaven seems to be inconsistent with the claim that suffering on earth is necessary for free-will and growth. Unless one is willing to say that heaven is a place without intellectual/moral growth and free will. That seems somewhat unheavenly.
2) Even assuming its impossible for God to create a completely evil-free and suffering-free world (and thus an unfair criticism to say he didn't), the amount of it and the nature of humans is a problem because the evil and suffering appears to be human reason to be highly excessive and unnecessary. There are many many examples. Smallpox is just one: the world gets along without it just fine now, so it obviously wasn't philosophically necessary. But it caused millions or possibly billions to suffer and die in misery when it existed. Why not simply prevent it from ever being? Atheistic evolution is consistent with this data. So is hands-off deism. Hands-on benevolent theism is not.

It is precisely the idea of the tri-Omni God that causes the problem of evil. Give up any of the "omnis" and the problem goes away. This is trivially true if we give up omnibenevolent. Giving up omniscience could allow one to conclude that God simply doesn't know about all the evil that occurs. Giving up omnipotence would lead to the conclusion that God simply cannot prevent evil from occurring. It seems to be this last one that most Christians choose to resolve the problem of evil, even if they won't readily admit to it. The free will defense certainly falls within that realm - if free will exists, it is not possible for God to eliminate evil.

Phil, Gordon, SN (or any other believer on here): I am very interested to hear a defense of the POE that does not deny one of the "omnis", if you are capable of producing one. If you explicit don't believe in the "tri-Omni God", well then my apologies for making unwarranted assumptions about your beliefs, and of course, you have no need to worry about the POE.

Sean T:

The free will defense certainly falls within that realm [removing the omnipotent omni] – if free will exists, it is not possible for God to eliminate evil.

Maybe. However, even if some evil is uneliminatable the same way God can't lift an unliftable rock, or can't create a married bachelor, the world we see is still not consistent with what one would expect from a tri-omni being. There's a lot of contingent evil which is inconsistent with a claim that God allows free will yet otherwise intervenes in benevolent ways.

#21

"1)the claim of an evil-free heaven seems to be inconsistent with the claim that suffering on earth is necessary for free-will and growth. Unless one is willing to say that heaven is a place without intellectual/moral growth and free will. That seems somewhat unheavenly."

Eric, around here, and especially lately, we hear advertisements for retirement communities that emphasize the fact that you will not have to shovel snow or do other worldly tasks. So I don't know if that argument really holds up.

After all, the business about free will has deeper flaws, since we are not given the option of deciding what counts as sin, only whether to participate or not. A truly muscular and demanding religion should have a rule like "ignorance of the law is no excuse".

Good point eric. For the purpose of philosophical argument, you are right. That still doesn't seem to me to get theists out of the woods, though. How does free will explain the fact that some children suffer from bone marrow cancer? Whose free will would be violated if God cured such children?

Taking the free will defense back to original sin in such cases would seem to run smack into a denial of omnipotence. It's hard to see why the concept of original sin should REQUIRE that some children suffer from horrible diseases. In particular, why those particular children and not others? if it were required that children suffer because of original sin, why not ALL children? I still think there's omnipotence denial going on there.

Taking the free will defense back to original sin in such cases would seem to run smack into a denial of omnipotence.

Also a denial of benevolence, as it's a very very bad case of collective punishment. They sinned so my cat has to grow old and die. Or in Gordon's version, I still sin today, so my cat has to grow old and die. Either way, its the sort of collective punsihment we generally consider to be evil.

It’s true, of course, that anyone can announce anything — regardless of what it is — as his fundamental, non-negotiable axiom, and then go on to construct a philosophy based on that foundation. But the universe doesn’t care what we think; it is what it is and it does what it does, even if we think it isn’t and doesn’t. If your axioms result in a belief system that is out of sync with reality, you have two options: revise your axioms or revise reality. The former is usually easier.

So sayeth SC over on his blog.

Reality is completely out of sync with a literal reading of the Bible. The creation story would have us believe that living things were created from dust shaped into animal forms or, as in the case of Eve, a rib bone carved into human form. Both of these are very human means of creating - long known from archeological sites. Read about golems to see how these forms could be animated. Living things aren't clay or bone figurines animated - they are built in a totally different way. The story is told from the view of a human almost completely ignorant of biology, chemistry and physics - creating things in the only way humans knew how at the time. It is so obviously a human story - a product of a human mind with no revelations from any Gods.

By Michael Fugate (not verified) on 23 Feb 2015 #permalink

"as in the case of Eve, a rib bone carved into human form."

There are two descriptions of Eve's creation: in one she was created at the same time as Adam, in the other, from his rib.

When the book of fairy tales starts off with blatantly contradictory versions of one key story, you have to tie yourself into fits of dishonesty to continue to support it - as phil, sn, gordon, and vs so continually demonstrate.

Pro Tips to Walt #11:

“Neither one [neither atheism nor evolution] includes nor compels a position on evil.”

Please read (and comprehend) before you write.
I never even implied atheists and evolutionists couldn’t have positions on evil. Almost all of them certainly do. What I said is that their position is not consistent with an atheistic evolutionist world-view.

“… by atheists (or evolutionists, which are not synonymous …)

Rule of thumb: All atheists are evolutionists, but not all evolutionists are atheists.

By See Noevo (not verified) on 23 Feb 2015 #permalink

#29

Your reasoning is very circular.

First: " All atheists are evolutionists, but not all evolutionists are atheists."

That's not necessary at all. I might believe that species came into being ab initio from a quantum foam. No God, no evolution.

and

"their position is not consistent with an atheistic evolutionist world-view."

Evil is an arbitrary construct-- categorizing some action as "evil" is not different from categorizing it as "blue". If God can decide that wearing mixed fabrics is a sin, why cannot atheistic evolutionists decide that e.g. claiming belief in God is "evil", or that God's ordering of the universe is "evil"?

SN: Read and comprehend. Atheism is not a worldview. It is the rejection of a worldview. Evolution is a process.

By Walt Jones (not verified) on 23 Feb 2015 #permalink

To Gordon #13:

Well done.

By See Noevo (not verified) on 23 Feb 2015 #permalink

@32: since gordon contributed nothing of substance, your comment is perplexing.

To Verbose Stoic #18:

Not bad!

By See Noevo (not verified) on 23 Feb 2015 #permalink

eric,

So we should probably discount it, and say that any defense has to explain the presence of evil and suffering without resorting to the claim that it’s just really good in a way we can’t possibly understand.

But we don't have to appeal to the "mysterious ways" argument, and I'm not doing that here. I'm pointing out cases where the impression of whether an action is immoral or not could depend on the amount of knowledge you have, so that if you knew more an action that you thought was immoral would no longer seem so. And that wasn't even my starting point, which was that the idea that not preventing suffering is inherently immoral is no where near as intellectual or philosophically safe as you think it is. To expand on that:

1) Kantians and Stoics don't consider suffering to be inherently vicious or immoral, as suffering is an indifferent. They can therefore easily see cases where a person would themselves prefer to suffer incredibly instead of taking a vicious action. From there, it isn't hard to see how not preventing someone else from suffering might be a better choice than acting viciously, thus breaking the link from "You could have prevented their suffering" to "You are acting immorally", which is what is required to break omnibenevolence. So from their perspectives, even if they roughly agree that one should prevent the suffering of others when one can, to make that claim as strongly as necessary to pull off even an argument from probability requires more work.

2) Utilitarians DO judge morality by the suffering the action causes ... but have to note that that's the total amount of suffering, and that a being that knows more might know more about the consequences and so about the total utility.

So, because God knows more both about what is moral and about what the consequences the actions are, the idea that we only think it immoral because of an at least currently limited perspective is not an invalid counter. But all that means is that the person claiming that there is a Problem of Evil needs to do more than simply say "Bone cancer?" or "Childhood leukemia" as if that's an automatic win, and something that the theist needs to explicitly demonstrate is necessary. They need to tie suffering more strongly to morality and essentially claim to know that their moral view is right AND that allowing that would be immoral for someone in God's position. That's a bit more than most are willing to do.

Also note that it is the Problem of Evil theorist who has the burden of proof here, not the apologetic theist, since it is the former who is making a claim -- that there is an incompatibility between the world and their conception of God -- and not necessarily the latter. As I flat-out stated, I'm not saying that the Problem of Evil is necessarily WRONG, just that it's not necessarily CORRECT either.

1)the claim of an evil-free heaven seems to be inconsistent with the claim that suffering on earth is necessary for free-will and growth. Unless one is willing to say that heaven is a place without intellectual/moral growth and free will. That seems somewhat unheavenly.

You shouldn't bring up arguments directly against me that we've argued over before as if we've never argued over them, without at least making it clear that it's your personal view. We've talked about this before. You don't need moral growth in heaven because the people who get into heaven have succeeded, so you wouldn't need any kind of moral dilemmas to work at. And presumably people who were doing science or philosophy in heaven would be people who developed an ingrained love for doing it, and would not need any push to do it by trying to stop suffering in the world. So no problem, even if we put aside that this might be far too far a humanized idea of heaven.

In summary, they still have free will in heaven but are the sort of people who use it morally, and who developed that ability, and any intellectual growth follows from their developed desire for that as well, so it isn't an issue.

There are many many examples. Smallpox is just one: the world gets along without it just fine now, so it obviously wasn’t philosophically necessary. But it caused millions or possibly billions to suffer and die in misery when it existed. Why not simply prevent it from ever being?

Because there was value in our eliminating it as human beings? Heck, smallpox is a bad example because when we look at how it was eliminated and how vaccination relates to other diseases there are indeed choices for us to make in relation to it, ones that matter morally. But again you can't toss out one example and insist that it could have been eliminated or, rather, that not eliminating it from the start means that God CAN'T be good, because THAT'S the claim you need to make: that it being there means God CAN'T be good.

Tell ya what: I'll agree that God can eliminate any specific suffering as long as you can show that your reasoning for the elimination of that specific suffering doesn't also mean that God has to eliminate ALL suffering. If you can't, then I'll demand that you show that we have to have a suffering-free world or else God can't be benevolent. And no introductions of degree unless you can show how we can objectively draw the line of what level of suffering is allowed, because this is not the worst of all possible worlds even if it might not be the best of all possible worlds.

By Verbose Stoic (not verified) on 23 Feb 2015 #permalink

To Sean T #22:

“Phil, Gordon, SN (or any other believer on here): I am very interested to hear a defense of the POE that does not deny one of the “omnis”, if you are capable of producing one. If you explicit don’t believe in the “tri-Omni God”, well then my apologies for making unwarranted assumptions about your beliefs, and of course, you have no need to worry about the POE.”

How about this:
Free will is God’s greatest gift to man. For without free will there can be no true love, and God is all about love (cf. 1 John 4:8). However, free will is also the scariest gift, for its misuse (i.e. sin) will lead to the opposite of love. And there is no end to how bad and disordered this can be (cf. Mat 6:23).

There is no question that the heart and mind of man seeks meaning, life, and love – ideally, eternally. God desires and wants to reward this seeking. But sin has largely (not completely) darkened and disordered our hearts and minds.

As I wrote to you on another thread:
God is true. God is not only all-good, God IS the Truth. He is pleased by things which accord with His nature. We are not God. We are creations of the Creator. Unlike all other creatures, we are made in His image and likeness. However, we often don’t live up to His standards and His nature because we have a fallen nature due to our sin. In other words, we often don’t behave “morally.” He has given us His moral standard. We CAN live up to it, with the continual forgiveness and help He has promised us. But many give up (cf. Mat 10:22; Mat 13:20-21).

Bonus feature:
Religions result from this seeking, but are almost always (but not always) similarly disordered. Common sense shows that one can be wrong in limitless ways, but right in usually only one way. Given that each religion conflicts with the others, logic tells us that either 1) all religions are wrong, or 2) at most, only one is right.

By See Noevo (not verified) on 23 Feb 2015 #permalink

To dean #28:

“There are two descriptions of Eve’s creation: in one she was created at the same time as Adam, in the other, from his rib. When the book of fairy tales starts off with blatantly contradictory versions of one key story, you have to tie yourself into fits of dishonesty to continue to support it…”

Genesis 1 & 2 do not contradict. Chapter 2 simply reiterates the account with different points of emphasis.

By See Noevo (not verified) on 23 Feb 2015 #permalink

@37: Thanks for demonstrating the comment about how blatant a lie must be told in order to reconcile things.

To zebra #30:

[Your reasoning is very circular. First: ” All atheists are evolutionists, but not all evolutionists are atheists.” That’s not necessary at all. I might believe that species came into being ab initio from a quantum foam. No God, no evolution.]

And how did the first life on earth lead to all the life we see today?

[“their position is not consistent with an atheistic evolutionist world-view.” Evil is an arbitrary construct– categorizing some action as “evil” is not different from categorizing it as “blue”.]

I think I agree with you 100% here! From an atheistic evolutionist world-view, “evil” is qualitatively and objectively no different than “blue.” It’s essentially like a matter of taste. You’re not right or wrong if blue is your favorite color.

By See Noevo (not verified) on 23 Feb 2015 #permalink

Genesis 1 & 2 do not contradict. Chapter 2 simply reiterates the account with different points of emphasis.

Why do you think those stories are true, and not fairy tales, as they appear to be?

I bet you don't know why you believe what you do. I don't think you can say. You just do it, blindly, without reason, without rationale, without self-insight.

It's certainly not because there is even a scintilla of empirical evidence to back you up.

To dean #28 & #38:

I’d rather not talk about the alleged contradiction between Genesis 1 & 2 right now.

I’d like to take a break, and tell you about one of my pet projects.

I’m considering writing a biography of Eli Manning. I have a rough outline for the first two chapters.

Chapter 1:
Eli was born 1/3/81.
Seven years later he began to take a keen interest in playing football.
Seven years after that, he began to excel in the game at Isidore Newman High School.
Seven years after this, he was well on his way to building an impressive college career at Ole Miss.
Seven years later, he had already won the first of his two Super Bowl rings!

Chapter 2:
Eli Manning quarterbacked the New York Giants to Super Bowl Championships in 2007 and 2011.
He’s definitely a candidate for eventual Hall of Fame status. What a professional football career!
Now, Eli learned many valuable lessons about football and about life in high school.
And then, he had a brother, Peyton, who also had a great influence on him.

That’s all I have so far. I’ve never written a book before, so I could use some advice. I was wondering, does anyone out there think I should re-order the Chapter 2 stuff? I mean, you don’t think anyone would misread it and think Eli went to high school and received a sibling after he won the two Super Bowls do you? No way. Maybe I’m just being paranoid. Hey, gimme a break. I’m just a novice at this.

By See Noevo (not verified) on 23 Feb 2015 #permalink

Genesis 1 & 2 do not contradict. Chapter 2 simply reiterates the account with different points of emphasis

And neither has any relation to reality.

By Michael Fugate (not verified) on 23 Feb 2015 #permalink

#39

"And how did the first life on earth lead to all the life we see today?"

Perhaps you are not familiar with the terminology ab initio?

All the species that exist now simply came into being as they are-- it's like 'what God did' except without God. So there is no need for evolution, or God. Just that quantum foam and chance.

"From an atheistic evolutionist world-view, “evil” is qualitatively and objectively no different than “blue.”"

It has nothing to do with being " atheist evolutionist". It is also true for theists. The word "evil" has no intrinsic meaning. You have your list of entities with that label; others have a different list.

SN: "I think I agree with you 100% here! From an atheistic evolutionist world-view, “evil” is qualitatively and objectively no different than “blue.” It’s essentially like a matter of taste. You’re not right or wrong if blue is your favorite color."

You're actually close here. We evolved as a social species for whom survival was enhanced by cooperation. Our morality developed out of this, and was eventually codified by religion and law (not necessarily in that order). We wouldn't be here if most of our ancestors hadn't acted with mutual concern - a preference, like a favorite color, only more important. "Evil," from a materialistic perspective (which I see is what you seem to mean by "atheistic evolutionist") is an adjective, like blue. Its use as a noun is a metaphor for the actions of evil people - or gods.

By Walt Jones (not verified) on 23 Feb 2015 #permalink

#44

I haven't heard this reasoning before so maybe I am missing something:

"Atheism is disempowering, because it blames God."

Huh? How can you blame something that you don't believe exists?

Eric,

Last I heard from you, you were busy proving that people knew how to butcher the English language. If your bottom line is to simply deny the obvious, then there is no purpose in my replying to you at all.

You have become a broken record on the "booby trapped Garden" scenario. I have answered you, and just as with Design, you simply go silent when you have run out of silly rebuttals. Then you repeat your point about the booby trap for the umpteenth time.

You want a world without cause and effect. You want stoves that cook food but don't burn hands. We have been over all this. You want justice and mercy from men, but only Mercy from God, with no justice. You want water that is not wet.

Sorry, but while most atheists here have butchered the meaning of Omnipotence, you have run off with the prize on this one. And you won't even come straight out and admit that you find Free Will to be a very, very bad idea.

I have responded to all your points, and not one of those discussions has gone anywhere.

"Atheism is disempowering, because it blames God"

A god that atheists don't believe in?

By Walt Jones (not verified) on 23 Feb 2015 #permalink

#45

"We evolved as a social species for whom survival was enhanced by cooperation. Our morality developed out of this,"

No, what developed were mechanisms that modulated behavior as needed in particular circumstances. "Morality" is an arbitrary set of rules that may or may not have utility, and need not involve cooperation.

"You want a world without cause and effect. "

Whether you are willfully ignoring the point or just not capable of getting the point is irrelevant: what you just stated is not the point.
It is impossible to view any creature as benevolent when it sets its creation, which it claims to love, up for failure by threatening them with things they cannot understand. If a parent were to tell a child who had just learned how to crawl "Don't touch this fire pit" and leave it at that and the child unattended, that parent would rightfully be viewed as horrible. If, even after the worst happened, the parent then vowed to punish all offspring of that child because of what the child did, that simply compounds the horror.
That despicable behavior is what you and the others so foolishly ignore (or claim does not exist in the first place).

The claptrap about "design" - why that is meaningless cow crap has been explained many times. It has become clear that you all prefer your self-imposed scientific ignorance to reality.

For me, the most compelling argument is the one from Design.

How can I detect design?

How can I test your assertion that your perception of "design" is a reason to think gods are real? How does "design" lead you to think that there are creator gods, as opposed to creator aliens, say? How does a purported quality of "design" provide even the slightest support for the existence of the supernatural?

Of course you cannot say. Like most believers, you are unable to offer any rationale for your convictions. You can neither explain nor back up your silly beliefs. All you can do is emit puffs of smoky pseudo-science.

Sean T,

I agree with you that the Free Will argument does indeed place a limit on God's omnipotence. Similarly, it places a limit on his Benevolence. This is precisely why it is the answer to the problem of Evil.

Because the only answer that sticks strictly to the 3 Omnis requires a controlled world where nothing bad can ever happen. Sure, God could get of smallpox or bone cancer without eliminating Free Will. But he cannot get rid of the consequences of stupid, wrong or Evil acts by people with Free Will, without doing away with Free Will itself.

The question about children suffering shows that omni-benevolence is also limited, because God cannot stop the suffering of innocent third parties, even though that is an obviously benevolent thing to do. You can see this in the words of Jesus:

"O Jerusalem, Jerusalem, the city that kills the prophets and stones those sent to her! How often I wanted to gather your children together, just as a hen gathers her brood under her wings, and you would not have it!"

Here we see the compassion and benevolence of Jesus clearly limited by the will of the people to do wrong.

So there is no answer that leaves the 3 omnis in tact. Which is precisely why it is so important to see that this "favorite philosophical conundrum" was addressed right from the very beginning. The same book that says in chapter 1 that everything was created Good, by chapter 3 is busy explaining to us the origin of Evil.

Indeed, were the Creator silent on the matter, then we would certainly have to question his Benevolence. But if we take his answer, and see that he chose to limit his powers in order to give us a genuinely Free Will, then that tells us where we must dig for further answers.

I consider Free Will to be that greater Good that made it worth the risk of allowing Man to choose Evil. And to clarify for Eric, while God must allow us the opportunity to choose Evil in order to grant us a genuinely Free Will, the highest Good is then achieved when we, of our own Free Will, choose consistently to do Good.

And perhaps you can see that I still believe that the 3 omnis are true of God. But he chooses to limit two of them in his interactions with our world, as that is the only way he could give us Free Will. But he was up front about all this. He made no secret of it. It is at the beginning of our earliest sacred writings, which is exactly where you would expect a Benevolent God to put it.

VS:

the idea that not preventing suffering is inherently immoral is no where near as intellectual or philosophically safe as you think it is.

"Preventing suffering" is only half of it. You yourself say that stoics and kantians may allow it when the choice is some vicious action that they choose not to do. But presumably they don't do this vicious action because they think it immoral. Which brings in the other half of theodicy: when God does such vicious actions that neither stoics nor kantians nor pretty much anyone else would choose to do, it sure looks like nonbenevolence.

They need to tie suffering more strongly to morality and essentially claim to know that their moral view is right AND that allowing that would be immoral for someone in God’s position

I don't have to know that my moral view is right to judge Christianity's God by the moral claims of Christians and others. If I point out that murder of children is wrong, that Christians generally agree that murder of children is wrong, and yet that God killed countless children in the flood, and your best defense is "well, we can't know if that was wrong, because our data is limited," then I'll be happy to rest my case and we can both declare victory, I guess.

Also note that it is the Problem of Evil theorist who has the burden of proof here, not the apologetic theist, since it is the former who is making a claim — that there is an incompatibility between the world and their conception of God

I don't think so. The theist is making the claim that this entity exists, has done certain actions, and is benevolent. The theodicy theorist is pointing out that the actions the theist claims this entity has done don't look benevolent. Its really up to the theist to show why a prima facie assessment must be wrong. Saying "well its wrong in a way we understand" is, IMO, just hiding a theistic circular argument: what is really meant by such a claim is "I assume the prima facie assessement must be wrong because God is good," even though the goodness of God is what they originally set out to show to the theoretician.

You don’t need moral growth in heaven because the people who get into heaven have succeeded

Even those aborted babies? No moral growth for them? Well, that sucks! Good god, imagine having to go through eternity with the moral and intellectual capacity of a zero-year old.

you can’t toss out one example and insist that it could have been eliminated or, rather, that not eliminating it from the start means that God CAN’T be good, because THAT’S the claim you need to make: that it being there means God CAN’T be good.

No, I thnk you've drastically moved the goalposts. Nobody is arguing that God can't in theory be good. They are arguing that his actions as recorded by his followers sure don't look like they are good; they do not appear to be the acts of a benevolent deity. If you concede that they do not appear to be benevolent, then I'm fine conceding that the philosophical possibility of them being good has not been definitively ruled out.

Hmmm...I missed a negative in there. My third paragraph, first quote, should read "...in a way we don't understand..."

You have become a broken record on the “booby trapped Garden” scenario. I have answered you

Where? Link or just tell me the number of the post in the previous thread, and I'll look at it. As far as I can tell, you had no answer at all for why God would booby trap the tree, and your answer for collective punishment is that granting gifts to the children of people who do good somehow makes it moral to punish the children of people who do evil. We would never call that moral when a human does it, why would we call it moral when God does it?

Zebra,

The Problem of Evil is a favorite of atheists because they think it disproves the existence of a Benevolent God, because if there is a God, then he must be responsible for all of the Evil in the world. In my discussion in this and previous threads, every rebuttal has come back placing the blame for Evil squarely at the feet of the hypothetical benevolent God.

So the atheist position, while denying God's existence, is to say that if he does exist, he is to blame for the presence of Evil in our world. This blames God and not ourselves for the problem.

Where the discussion does not then go is, "But there is no God, so that means that WE are to blame!" The train of thought simply stops here, with the blame laid at God's feet, in a world in which there is undeniably Evil, and yet there is no God to rightfully take the blame. This is disempowering. It does not lead to any potential answers to the problem.

For the atheist, the Problem of Evil is not a problem to be solved. It is merely a "favorite philosophical conundrum".

For the Christian, the Problem of Evil is a problem to be solved, and one that we do have the power to solve. This is empowering.

There’s a lot of contingent evil which is inconsistent with a claim that God allows free will yet otherwise intervenes in benevolent ways.

Hell, the Bible is chock full of stories of contingent evil that were a direct consequence of God having intervened in malevolent ways.

By Another Matt (not verified) on 23 Feb 2015 #permalink

And to clarify for Eric, while God must allow us the opportunity to choose Evil in order to grant us a genuinely Free Will, the highest Good is then achieved when we, of our own Free Will, choose consistently to do Good.</blockquote

Which is a complete nonanswer to theodicy. Older women have free will, do you agree? So it cannot violate the free will of a young man to give him the propensity to avoid violent retaliation that an older women has. Do you agree?

Gordon,

The Problem of Evil is a favorite of atheists because they think it disproves the existence of a Benevolent God...

And of course it does.

The gods you christians worship are depraved monsters. They are merciless murderers, eternal torturers, purveyors of genocide and sicko bullies.

It makes you look like lickspittle cowards. How do you manage to retain any self-respect?

Gordon:

the atheist position, while denying God’s existence, is to say that if he does exist, he is to blame for the presence of Evil in our world. This blames God and not ourselves for the problem.

It is hard to see how humans are to blame for Pharoah's refusals when God hardened his heart. Isn't that pretty much a textbook definition of taking away someone's free will? Or how humans are to blame for the punishment visited on Babel. Building towers is now an unmitigated moral evil?

Its also hard to understand why a great-grandchild must be punished for their great-grandparent taking the Lord's name in vain - not only is it not fair, it just doesn't even make sense. Yet God does that. Oh, my mistake, it's not hard to understand - God says he does it because he's jealous. That, IMO, is clearly not benevolent.

Where the discussion does not then go is, “But there is no God, so that means that WE are to blame!”

Is it really hard to understand why discussions of theodicy center around thestic notions of God's morality, rather than other problems in ethics? The subject is kinda in the title there, Gordon.

Phhht,

In a world of Cause and Effect, there can only be one original Cause. Even if there are "gods" or "aliens", there is a Creator who created them. I do believe he is the original, higher intelligence behind the designs that we call "Life", because I have no evidence that would lead me to believe otherwise. So I don't spend my time scanning the skies for aliens or praying to lesser gods.

Design is something we all recognize intuitively, but that atheists deny consistently. Otherwise they would have to concede the argument. Eric has already demonstrated that admirably, by proving beyond reasonable doubt that people are capable of misusing language. Nice way to avoid the issue.

I can see from your final paragraph that this discussion will not go anywhere. I expect less from you than I got from Eric.

Like most atheists, you are unable to offer any rationale for your convictions. You can neither explain nor back up your silly beliefs. All you can do is emit puffs of smoky pseudo-science.

Eric,

My last, succinct summary was a reply to Sean S in #246 of "Proving God Exists". Just the second half of that post, starting with, "Read the Adam and Eve story again."

I had not answered the collective punishment issue before, but I have now in #52, above.

#56

Gordon, I don't follow your reasoning at all. Who says there must be someone to 'blame' for the existence of "evil" if there is no God?

I don't think the concept "evil" is meaningful myself, but as it is being used here, there are unfortunate events like sick children, and there are conscious acts that harm people. Do you mean that those who commit "evil" acts are responsible for their actions? Who would disagree with that?

But who is this "we" you are talking about? I am not 'to blame' for children being sick, except perhaps if the cause is environmental and my existence contributes to that.

If you think otherwise, please explain.

Where the discussion does not then go is, “But there is no God, so that means that WE are to blame!"

Does is it matter if there is or isn't a God; aren't WE always to blame either way?

By Michael Fugate (not verified) on 23 Feb 2015 #permalink

Eric,

My last, succinct summary was a reply to Sean S in #246 of “Proving God Exists”. Just the second half of that post, starting with, “Read the Adam and Eve story again.”

I had not answered the collective punishment issue before, but I have now in #52, above.

Yes, God appears to have taken away Pharaoh's Free Will. It says so right in the story. The Tower of Babel was a problem. I can't see how dividing mankind by confusing their languages qualifies as Evil. But God did solve the problem.

Yeah, God can be hard to understand at times. Get things wrong, and your great-grandchildren will suffer. But get things right, and the effect will ripple down in the form of blessings for at least the next 20,000 years. Very puzzling.

Which is why, rather than questioning God, I'm just busy figuring out how to be on the blessing side of the equation, because it sounds like an incredibly good deal to me!

Free your mind, Eric. Look at where contemplating and debating theodicy leads you. It does not lead anywhere positive. You are stuck on insulting a God you claim does not exist. That's it.

And yet my study of this same subject, from the biblical perspective, has led me to a place of understanding, hope, freedom and empowerment that I never dreamed possible. I went looking for answers, and the answers have proven to be far better than I was taught in church, and far better than I even hoped for when I started down this path.

What I think is being missed in these discussions is a practical matter about human morality: even if "all morals come from God" as theists assert, it's still the case that theists disagree among themselves about what this means, how moral reasoning should work, what kinds of ethical systems ought to be in place, etc. "God says so" -- or worse, "God told me" (as I was used to hearing from relatives growing up) -- can only ever be a way to stop conversation and reasoning. It's the ultimate trump card.

Evidently if God exists, he has chosen to show his true will to only a tiny minority of people in the world, so the only way to forge ahead is to assume you aren't part of that minority and never allow "God says so" to count as an argument in any moral conversation. So while we may disagree on gods' existence, if you can say why God says so without further appealing to his will, that gives us something that can actually be debated.

By Another Matt (not verified) on 23 Feb 2015 #permalink

Gordon wrote to eric; "My last, succinct summary was a reply to Sean S in #246 of 'Proving God Exists'."

... and it was pretty bad. The story of the Fall utterly fails to answer this problem. It posits the idea that an uninformed, even misinformed choice is the epitome of a "free" choice, which is nonsense. From this story Gordon infers that information destroys freedom.

sean s.

By sean samis (not verified) on 23 Feb 2015 #permalink

Zebra,

The biblical notion of Evil is linked to suffering and especially, to Death. Some Evil is caused directly, while other is caused indirectly. Initially we only have control over the Evil we cause directly ourselves.

If you read some of what Eric has written, he claims that if there was a supernatural Creator, he should have made this world such that it was much harder, or maybe impossible for bad things to happen. This is blaming God for the presence of Evil in the world.

Embedded in this thinking is the notion that the deck is simply stacked against us. Evil happens because of things fundamental to how the world operates. Therefore, we can do nothing about it. Even once you stop hypothesizing about the malevolent Creator, the presumption remains in your mind that there is nothing you can do about the presence of Evil in our world.

When I say that "WE" are to blame, I mean every last member of the human race. "For ALL have sinned," as Paul puts it.

The moment I take responsibility for the presence of Evil in my world, the moment I accept the blame, in that same instant I empower myself to do something about it.

I cannot change God. I cannot change naturalistic forces. I cannot change other people. I can only change myself. So if I am to blame, then I can do something. This puts me in control of the problem of Evil, rather than looking in on it from the outside feeling completely powerless to do anything about it.

G K Chesterton was once asked, "What is wrong with the world?" His reply: "I am!"

Another Matt,

I answered "Why" God says so, as I understand it, in post #84 under "The Meaning of Life".

Sean S,

Welcome to the new thread, which flows nicely on from our last one, which is now closed.

I answered you in my very last post in that thread. I have been waiting for your answer, but it appears that you have not read it yet.

Gordon,

I wasn't part of that thread. But you can't have possibly answered "why God says so" for every single situation a theist might be tempted to use the "God says so" trump.

If I have a break later I'll maybe go check out that thread.

By Another Matt (not verified) on 23 Feb 2015 #permalink

#67

Gordon,

The capital-E evil you are talking about is a part of theism, so it is ridiculous to expect atheists, having rejected the concept of God, to embrace the concept of Evil as you present it.

It sounds like you've been doing this a while; how have you missed this elementary error? Believing in your version of Evil is the same as believing in your God.

Sean S,

I see that our discussion from the last thread is now over. You are now responding to Zebra, but not to me.

By Gordon (not verified) on 23 Feb 2015 #permalink

In reply to by zebra (not verified)

Zebra;

Gordon wrote that “The Problem of Evil is a favorite of atheists because they think it disproves the existence of a Benevolent God, ...

The most important effect of the Problem of Evil (POE) is that it induces believers to all manner of semantic and logical contortions trying to “solve” the problem, which discredits every other thing they “testify” to.

The only coherent response a believer can give the POE is to say “I can’t explain it”. Many do give that answer (much to their credit) but many others think they can make one plus one equal three.

Gordon wrote that “For the atheist, the Problem of Evil is not a problem to be solved.” Well, I’m not an atheist (I’m a doubter.) but even I know the POE is really only a problem for believers in the Abrahamic God.

Pretty much everyone knows that evil in the world needs a response, but the POE is for the Abrahamic believers to solve.

Gordon wrote that “If you read some of what Eric has written,” ... blah blah blah.

... and if you read what Gordon’s written, you’ll see he thinks God is Omnipotent, but that there’s also tons of stuff God cannot do. In fact, Gordon seems to think you and I are more powerful than God.

Gordon wrote that eric and others are “blaming God for the presence of Evil in the world.

If God is at fault for the presence of evil in the world, why not blame him? If God did create the world, He’s the one responsible for how it came out.

Gordon wrote that “Embedded in this thinking is the notion ... Evil happens because of things fundamental to how the world operates. Therefore, we can do nothing about it.

No. That’s nonsense. What’s embedded in this thought is that for many of us, the only reason we might believe in any deity is because persons like Gordon tell us we should; yet Gordon’s stories are incoherent and self-contradictory. Therefore we have good reason to reject his claims. His testimony does not add up.

Gordon wrote that “I cannot change God. I cannot change naturalistic forces. I cannot change other people. I can only change myself.

This much is true. But then he writes “So if I am to blame, then I can do something.

This is unnecessary. We don’t have to be “to blame” to do something about the way the world is. Even if we are innocent, we can do something about the world. I have never taken food from a hungry child, but I can still do things to feed hungry children.

However, since it appears reasonable to believe whatever God there is, He IS TO BLAME, it is not reasonable to expect anything good from Him.

Gordon wrote that “...rather than questioning God, I’m just busy figuring out how to be on the blessing side of the equation, because it sounds like an incredibly good deal to me!

Yet Gordon’s also gone to great lengths to claim that no moral code applies to his God. The moral rule that demands honesty and the keeping of promises is part of this code that Gordon exempts his God from, so Gordon is trumpeting the value of promises made by a God whom he insists is not trustworthy. And he wonders why I hesitate to join him in this “deal”.

sean s.

By sean samis (not verified) on 23 Feb 2015 #permalink

Gordon,

In a world of Cause and Effect, there can only be one original Cause.

First you must demonstrate that we live in "a world of Cause and Effect," whatever that means. When you have done that, you will have to demonstrate that "there can only be one original Cause" . Even assuming you can manage all that, you will still have to demonstrate that your postulated "one original Cause" is the act of a supernatural agent.

Until you do those things, you're just blustering, without the least foundation.

Even if there are “gods” or “aliens”, there is a Creator who created them. I do believe he is the original, higher intelligence behind the designs that we call “Life”, because I have no evidence that would lead me to believe otherwise.

In other words, you cannot say why you believe what you do; you just believe it. You believe, without reason, without rationale, and, I note you never address this one, utterly without any empirical evidence to back you up.

How can you tell you're not mistaken? How can you tell you're not deluded?

And you cannot say even how to define "design," much less how to detect it. Like gods, you simply insist that "design" must exist because you believe in it. You insist, without any justification whatsoever, that everybody just knows that design is real - and you insist the same thing about your purported gods.

As to my rationale for atheism, it goes like this.

There is not the slightest iota of empirical evidence for the existence of gods. In all the vast intellectual structure of science, technology, engineering, and mathematics, erected
through centuries of the painstaking study of reality, there is no hint of a god. Gods cannot be detected. Gods have no unambiguous effects on reality. They are indistinguishable from other fictional characters like vampires, werewolves, and superheroes.

Given all these facts, I conclude, tentatively, that gods do not exist.

Why should I think otherwise, Gordon?

It seems pretty clear you cannot say why you think otherwise.

We’re told that the random, non-rational process of evolution continually advances a biological robustness which includes longer life and successful reproduction (along with, of course, a virtually limitless number of new complex functions, organs, and organisms). If death and disease (and deadly, dire human designs) work against species’ success, why haven’t the “evils” of death and disease (and dirty dealings) been “solved” by evolution in the last several billion years?

By See Noevo (not verified) on 23 Feb 2015 #permalink

We’re told that the random, non-rational process of evolution continually advances a biological robustness which includes longer life and successful reproduction

Who is telling you that?

By Walt Jones (not verified) on 23 Feb 2015 #permalink

In reply to by See Noevo (not verified)

#73

Fundamental misunderstanding of Evolution. There is no goal.

We get Bible verses like Isaiah 55:8
"For My thoughts are not your thoughts, neither are your ways My ways, saith the LORD."

Always?, 90% of the time?, 50%?

Why does most of the OT seem like God's ways and thoughts are our ways - or at least the ways of humans living at the time?

Here's what I want - an explanation of how God creates. How is it different from our ways of creating? How did God make light before making the sun and stars? How did God make the first living thing? How did God make dust into DNA, proteins, lipids, organelles, cells, tissues? I want enough detail so that if I had the ability I could repeat the experiment.

If Genesis were a scientific paper it would be rejected for insufficient methods. If this God were capable of dictating thousands of rules about what is pleasing and displeasing, it can certainly take the time to tell us how it created. There are too many arguments from authority without the least bit of backing evidence.

So before we hear one more whine about randomness and directionlessness, let's hear the nonrandom, direct procedure that you claim this God employed.

By Michael Fugate (not verified) on 23 Feb 2015 #permalink

"Get things wrong, and your great-grandchildren will suffer."

And there it is - there is the simple reason, gordon, the god you pledge your life to is monumentally evil. The fact that you don't realize that doesn't speak well of you either.

zebra,

“Fundamental misunderstanding of Evolution. There is no goal.”

Nice axiom. Would you care to explain how, starting with nothing and no goal, how a pancreas might have developed?

===

Michael Fugate,

“Here’s what I want – an explanation of how God creates. How is it different from our ways of creating?”

We don’t create. We fashion from existing materials. God creates by speaking.
-
“How did God make dust into DNA, proteins, lipids, organelles, cells, tissues? I want enough detail so that if I had the ability I could repeat the experiment.”

You have enough detail to understand why nobody is having much luck making such things in a lab, from dust or anything else. If you put enough thought into it, you might begin to comprehend why accidental dust events have never resulted in anything either.

Verbose Stoic--

You are welcome to post a link to your blog post.

Gordon--

Bone cancer in children is a cheap shot, and avoids the real issue. They are the collateral damage from the real problem. But an example that tugs on the heart strings, and so a great way to win an argument without actually having one!

Bone cancer in children is simply a reality. The reason you are so touchy about anyone bringing it up is that your abstract hand waving amounts to nothing in the face of concrete examples of evil and suffering.

So blaming God for children suffering is just an extension of the original question, but now with an emotive spin to it. You, Mr. Fry, Eric and the two Seans appear to want a world without cause and effect. You want one person to be able to do stupid or Evil things, yet have it have absolutely no effect on anyone else, especially their children.

No. I want a world where children don't die of dread diseases. Your assertion that “Evil is unpredictable,” or your blather about cause and effect, are examples of what I mean when I accuse you of hand waving. How do they help me understand why God thinks it's appropriate to respond to human sin by torturing children?

You call this issue your “favorite philosophical conundrum”, yet you appear only interested in griping about it, not exploring it fully.

Your reply to the problem requires that I deny modern science and that I accept an explanation that makes no moral sense at all. Those seem like good reasons for rejecting it.

Let me frame it for you this way. If God is to blame for the problem of Evil, then there is nothing we can do about it. We’re simply screwed, and that’s it. If we are to blame, then this fact immediately empowers us to undo the mistake, once we understand it.

Atheism is disempowering, because it blames God. By denying the existence of God, you leave yourself in a position where there is undeniably Evil in the world, and yet there is nothing you can do about it.

Pure gibberish. Evil and suffering are just facts about the world. They are hard to understand under the assumption that there is a loving God in charge of it all, but easy to understand if there is no God.

I don't agree that the Biblical narrative is empowering. I find it horrifying, for the reasons I have explained. But whether it is empowering or not is irrelevant, since it is demonstrably false.

Whether we believe in God or not, we can all try to live our lives in a decent way. That belief in God is in some way a prerequisite for reducing evil and suffering is belied by the fact that the most socially conscious and morally decent societies on earth are precisely the ones with the lowest rates of religious belief.

To Walt Jones #75:

Me: “We’re told that the random, non-rational process of evolution continually advances a biological robustness which includes longer life and successful reproduction…”

You: “Who is telling you that?”
Just about every pro-evolution piece I’ve read. Hey, here’s a recent one, and I dedicate it to the commenter here named “zebra”:
“… the researchers make the case that the association between striping and temperature likely points to multiple benefits -- including controlling zebras' body temperature and protecting them from diseases carried by biting flies.”
http://www.sciencedaily.com/releases/2015/01/150130092923.htm

By See Noevo (not verified) on 23 Feb 2015 #permalink

Jason - your summary comment @79 shows why you're the host. Thanks.

SN:
Pro tip #2: When citing an article, make sure that it says what you claim it does.

By Walt Jones (not verified) on 23 Feb 2015 #permalink

In reply to by See Noevo (not verified)

See No,

We’re told that the random, non-rational process of evolution continually advances a biological robustness which includes longer life and successful reproduction (along with, of course, a virtually limitless number of new complex functions, organs, and organisms). If death and disease (and deadly, dire human designs) work against species’ success, why haven’t the “evils” of death and disease (and dirty dealings) been “solved” by evolution in the last several billion years?

We're told that the omnipotent, benevolent actions of gods continually express the "love" and "justice" of those gods toward us. If death and disease (not to mention eternal torture in hell) work against the goals of the gods, why haven’t the “evils” of death and disease (and hell) been “solved” by the gods?

It cannot be for any humanly comprehensible reason such as the preservation of "free will," because, well, the gods are omnipotent. They can do whatever they damn well want, and human reason and consistency do not constrain them. It cannot be because human beings "brought it on themselves," for the same reason: benevolent gods can forgive anything - if they want to.

The persistence of death and evil and hell can only be explained if the gods will them. They are vengeful, depraved monsters, unworthy of any worship except by psychopaths and sniveling cowards. Their only saving grace is that they do not exist.

“We’re told that the random, non-rational process of evolution continually advances a biological robustness which includes longer life and successful reproduction…”

This could only be the case if the fitness landscape were static. It isn't. Biological robustness and successful reproduction are not merely properties of organisms – they are types of relationships between organisms and their environments. There isn't some universally maximum fitness that evolution is "advancing" toward.

By the way, Dan Dennett has been saying for years that living things are clearly designed – it's just that the designer is a process rather than a sentient agent.

By Another Matt (not verified) on 23 Feb 2015 #permalink

Phil, How did your God do it? - please explain. You can't because your God explains nothing. It is the ultimate in vacuousness - tired, useless, intellectually impoverished.

May next time you God stops by to chat, you can ask him how it was done?

Evolution can explain it on the other hand.

By Michael Fugate (not verified) on 23 Feb 2015 #permalink

Michael Fugate,

"Evolution can explain it on the other hand."

Well, cool. I've been waiting for someone to explain how replication enzymes evolved, without a goal of course. How did evolution do it?

Another Matt,

“There isn’t some universally maximum fitness that evolution is “advancing” toward.”

I love axioms. Actually, inasmuch as Gould and Eldredge made it clear that stasis is the rule, it would be more accurate to say that not much documentable advancing actually occurs. There is an abundance of forms who seem to stall out at maximum fitness, even it there isn’t such a thing.

Could you help Michael out on the replication enzymes deal?

Well, cool. I’ve been waiting for someone to explain how replication enzymes evolved, without a goal of course. How did evolution do it?

"How could evolution have done it" rather than "how did evolution do it" is a better question when you are thinking about plausibility. Here's a very crude analogy that might help. Evolution of replication enzymes seems circular for the same reason that the fact that a C compiler written in C seems circular (the teleology in your question is a separate issue). Neither of them is circular, though, if a forerunner with similar function was available.

We have a history describing how the latter evolved, but one workable hypothesis for the former is the "RNA world" hypothesis -- RNA can do similar things as proteins, which means that they could have gradually been replaced as catalysis factors and coding factors by structures that work better for those two functions,* separately, but keeping RNA as an intermediary in many of the relevant points of the process.

*The usefulness of the term "function" is relative to whether or not it makes sense to speak of something's interests in relation to an environment. A mouse crosses a small fallen tree from one side of the river to the other to escape a predator -- there didn't need to be any goals involved for the tree to function as a bridge, but if something were to serve as a bridge, there are only a few things that will serve, and it's not surprising that something like a fallen tree would suffice. The tree's function in that situation is only a function from the perspective of the mouse and the predator -- it's not a universal function. Evolution works like this -- it stumbles on something that suffices for some function whose advantageous nature is built into the environment. No goal, but not surprising that something would come to function in some way given that environment.

By Another Matt (not verified) on 23 Feb 2015 #permalink

Just in case there's anyone out there who's still confused about why atheists raise the "problem of evil", it's a thought experiment to show that the notion of a just, benevolent god is not consistent with the reality we see around us - similar to the way Einstein and others have used thought experiments to explore scientific hypotheses and confirm or refute them. Evil is not a problem for the Theory of Evolution. Under evolution some organisms have the survival strategy of preying on others, because it is one possible way to survive and evolution finds ways for organisms to survive. Also if death did not exist, evolution would have had to invent it because without death there would be no survival mechanism to drive evolution. So everything we see is consistent with and understandable by the paradigm of evolution, whereas people have to tie their reason in knots to make sense of the Bible and other religions, in ways they would never do in science, medicine, engineering, plumbing, or auto mechanics (among other things). For example, in courts of law we convict murders without struggling with the issue that they might have some hidden, benevolent reason for the murders, and don't entertain the possibility that space aliens, or some other entities that we have never seen might have used superior technology to frame the defendants. That is, assuming there are no theological philosophers in the jury.

Diversionary tactics Phil - when are you going to produce any positive evidence for creation? Please tell us everything you know about this supposed creator God and how you know it.
Evolutionary biologists are no shy about providing evidence, why are you? What are you trying to hide?

By Michael Fugate (not verified) on 23 Feb 2015 #permalink

#89, there is more evidence for the evolution of the red blood cell than evidence of gOd. That never stops the creationists from spouting about what they know not.

(evidence such as comparative analysis of DNA encoding hemoglobin proteins in different lineages.)

By MobiusKlein (not verified) on 23 Feb 2015 #permalink

Phil and his fellow believers don't have even a femto-speck of empirical evidence for the existence of gods.

In a world chock full of empirical evidence for everything from apples to zebras, from cosmic background radiation to the HIggs boson, there is none whatsoever for gods.

Why is that, Phil? Gordon? See No? Why is that?

The simplest explanation is that your gods are not real.

Walt Jones --

Glad you liked the comment! I suspect Gordon will have a different reaction though.

#71

Sean, thanks. I think the logical flaws have been covered pretty well by others, so I'd like to ask a question since you are familiar with this group and the discussion.

What always puzzles me is not the question of the existence of God, or the nature of God, but the existence of Believers and their nature.

Has anyone asked of the people who claim to be Believers how we can determine which among them are sincere and which are not?

I would think that a Believer must have some test to determine first that he himself Believes, and by comparison this could be applied to others. Has Gordon ever shared this with his interlocutors?

The point is that while I can understand the idea that God is 'unknowable', Believers are humans just like me. So there should be a metric I can apply to tell that I am speaking to 'the real thing'.

Thanks for any input.

Phhht @ #72,

You deny Cause and Effect. You deny Design.

Of course nothing can be proven to you.

I'm surprised that you even mention Science as an authority, as Science is the study of Cause and Effect.

It is not my job to tell you which end is up.

Michael,

Phil has given you THE answer to your question in #76. "and God SAID ... and there was ..."

Put another way, this world is not made up of atoms. It is made up of Words. Thus, the Gospel of John starts with this take on Genesis:

"In the beginning was the WORD ..."

A mere glimmer into the profound nature of this truth would stop you from putting your ignorance on open display with comments such as, "It is the ultimate in vacuousness – tired, useless, intellectually impoverished."

This is not Phil's opinion you are spitting on. It is the wisdom of the Ages, casually dismissed because you are so much smarter than your ancestors.

Jason,

I too want a world where children don't die of dread diseases. I want it for my own children.

Before I had children of my own, I didn't take this very seriously. But once I was faced with the question of what sort of future I was giving my children, I became more serious than most.

I made it my objective to give them the best possible future I was able to, and not merely in the financial sense that most think of. Imagine my surprise when I found out I had the power to give them a far better future than I had dared to dream of.

So what separates you and I is, not that we don't both want good things, but I know how to achieve them while you are not even interested in considering that it is possible.

The same goes for all your fellow atheists on this forum. They are simply regurgitating the very first arguments they threw at me the first time I posted, three threads back.

You have nicely summarized where the mental process stops for you all:

"I don’t agree that the Biblical narrative is empowering. I find it horrifying, for the reasons I have explained. But whether it is empowering or not is irrelevant, since it is demonstrably false."

So, as I already said in my last post, just like Mr. Fry, you want to discuss a hypothetical God so far as necessary to prove that he must be Evil or not exist. But you cannot and will not import the full set of presumptions necessary to actually hold a meaningful discussion about the real God.

Your "god" is a straw man who indeed does not exist and who certainly has no connection with reality. So of course your argument works. I have just admitted that he does not exist.

So you all rest your laurels in disproving your straw man, and will not engage the subject further because of your absolute certainty that the Bible is "demonstrably false". But unless you can discuss the Word of God with an open mind, it is meaningless to say you have honestly engaged your "favorite philosophical conundrum". No, you have just stacked the deck, and stopped the conversation the moment it looked like the house of cards might be crumbling.

So let me express my appreciation for the generally polite spirit of the discussions on your blogs. I hold a genuine respect for those posters who have engaged me in lengthy and vigorous exchanges. But it is now obvious that your minds are fixed and immovable, even when presented with new information. And so I shall not trouble you further.

God bless

Zebra @ #94,

We are ALL "Believers". You are one too.

Everyone on this thread is posting ideas that express his beliefs. In that sense, we are all "true believers" in that it is safe to assume that each one honestly believes his views to be correct.

I, for one, would not waste my time detailing my ideas about God if I actually believed there was no God. I'm sure the same can be said about Jason and Eric and the two Seans and you. You all would not waste your time writing what you do if it did not reflect a genuinely held belief that there is no supernatural Creator.

And that really is the answer. Because, while Phil and I both believe in God, it is obvious from previous threads that our actual beliefs differ quite markedly. Yet, even if I said that Phil was wrong, that would not make him any less of a Believer in God. His belief in God is genuine. What further test do you need to prove this than the consistency with which he has stuck to that conviction?

Sean Samis has said @ #71 that he is a doubter. So he is not a Believer in God, nor a believer that there is no God. But he still holds some fairly strong opinions, despite his doubts. And those opinions are in fact his beliefs. If he had no beliefs, he would not disagree with anyone here on any point.

Now, if you want some way to find out if someone is sincere about their belief in a Creator, it is the same test you would use to determine if they hold any belief. You believe in Evolution. Why should I question your sincerity? I have never met Phil, but the only thing that would make me question his sincerity would be if he kept making statements clearly contradictory to a belief in the supernatural. He has made no such statement over 3 threads now.

So if you can come up with a test for the sincerity of atheists, then I can give you a test for the sincerity of Believers in God. Because it will be exactly the same.

eric,

I was going to talk about you changing the subject and/or moving the goalposts, but I think I might have confused things a bit, so I'll give you the benefit of the doubt.

“Preventing suffering” is only half of it. You yourself say that stoics and kantians may allow it when the choice is some vicious action that they choose not to do. But presumably they don’t do this vicious action because they think it immoral.

Sorry, I was using the Stoic/Virtue Theory technical term there, relating specifically to vice, which means that anything that is vicious is immoral by definition. So the whole question -- and not only half of it -- is whether not preventing someone else's suffering when you can do so is necessarily immoral. For Kant and the Stoics, it is at least possible that it is at least not immoral to do so, and our moral intuitions seem to agree with that (as we don't consider ourselves immoral for not eliminating all suffering that we might encounter). So, again, more work needs to be done there.

This should also show how your point about the flood isn't relevant here. The discussion is about suffering caused by natural forces, which is what both Jason and Stephen Fry talked about. Direct actions are another argument.

The theist is making the claim that this entity exists, has done certain actions, and is benevolent. The theodicy theorist is pointing out that the actions the theist claims this entity has done don’t look benevolent.

Let's address the "doesn't look benevolent" point here. At first glance, it does look like there is an incompatibility there. That's why there's a philosophical debate in the first place. But the question here is not whether here seems to be an issue at first glance, but whether there really IS an issue. If you think it is, then you have to support that. As long as I don't insist that there can't be a problem, it's perfectly reasonable for me to point out that you're quite short on demonstrating that the problem really is one ... especially since I do that by citing actual known arguments around morality to point out that your argument that there is a problem is not exactly safe.

The odd thing here is that in the other thread you demanded that I demonstrate that there actually was a problem of fine tuning and that there wasn't some hidden mechanism that changed the probabilities. Thus, you insisted that those saying that there was a problem had the burden of proof. Here, you are saying that the person who is saying that there is a problem does NOT have the burden of proof, but instead those who think that there isn't a problem must prove that there isn't one. This is despite the fact that in both cases all we have is an apparent, prima facie, "looks like a" problem here.

I don't mean to be uncharitable, but it's starting to look like the burden of proof is always with the person who is not you.

IMO, just hiding a theistic circular argument: what is really meant by such a claim is “I assume the prima facie assessement must be wrong because God is good,” even though the goodness of God is what they originally set out to show to the theoretician.

No, they didn't, actually, The context of the Problem of Evil is the theoretician's attempt to show that a good God is incompatible with the world that we have. So the theist is defending against an assertion, not demonstrating one. So they don't need to prove the theoretician false to show that their argument isn't as good as they claim it is.

Even those aborted babies? No moral growth for them? Well, that sucks! Good god, imagine having to go through eternity with the moral and intellectual capacity of a zero-year old.

This isn't any kind of argument against my claim. As heaven by definition is supposed to filter for people of the proper moral character, what happens to those who have not developed morally enough to be properly judged will always be an issue. But your point was about free will and intellectual development in heaven, and given the basic principles those that get in there because of their moral development don't need to have their moral character tested any more, at which point they still have free will and the ability to potential pursue intellectual pursuits without having to have any suffering or moral tests. So your contradiction just isn't one at that point, and this point is nothing but a distraction from your purported contradiction.

One final note: again, you seem to be implying that I require philosophical certainty. I don't. As usual, you don't have anything like knowledge, but answer demands on that with assertions that I'm demanding certainty. I would greatly appreciate it if you would stop doing that.

By Verbose Stoic (not verified) on 24 Feb 2015 #permalink

"Your “god” is a straw man who indeed does not exist and who certainly has no connection with reality."

The reality is this: that comment applies equally to any god. The fact that you, phil, the verbose whatever, engage in convoluted discussions empty of fact to dismiss the contradictory items from your bible is telling (about the lot of you).
But, when push comes to shove, the lack of physical evidence - any evidence, really, outside of one book written by people with keen interest in keeping the story alive- for your deity is the primary point.

No Gordon, you are quite wrong. There is agreement; we atheists certainly believe that humans are to blame for evil. How could it be otherwise? We don't believe that there are any intelligent moral agencies in existence other than humans.

The POE is simply a "let's assume for the sake of argument" kind of thing. We assume for the sake of argument that God exists. We assume further the tri-omnis that most Christians profess apply to God. One of the consequences of that is that we must either deny one or more of the omnis or deny that God exists. Atheists are quite happy denying that God exists; that's kind of the definition of atheism. Thus, there's no need for atheists to deal with the POE from an atheist perspective. From a Christian perspective, however, it's a big problem. If you want to get around it by denying one or more Omni, (as you stated in your last reply to me), fine. That addresses the problem. God is not capable of eliminating evil from the world. Your reply is still wanting, however, because as you admit yourself God could eliminate bone cancer and smallpox without disturbing free will. If that's true, why doesn't He? Further, why should we worship a moral agency that has the power to reduce suffering but refuses to do so? I'm not being ungrateful or blaming God here; we could still be grateful to God for creating us. In similar fashion, if my father turned out to be a psychotic serial killer, I would still be grateful to him for giving me life, but I would not exactly go seeking his advice on moral issues (or pretty much anything else.) God is essentially, by your own admission, a serial killer. He allows people to die from all kinds of diseases that He has the power to prevent WITHOUT affecting anyone's free will. Why should we look to such a God for spiritual advice? Why should we believe anything that such a God has to say about anything?

(Again, in case you aren't getting it: this is another "assume for the sake of argument" kind of deal. I am NOT really blaming God for anything. Diseases happen for natural causes. Human evil stems fully from humans, etc.)

Gordon,

This is a science blog. (Look at the URL of the website). Most of us commenting on here have some basic understanding of science. When you post things like "most of us intuitively recognize design," what you fail to realize is that any argument relying on intuition or common sense will be immediately suspect to those of us with a modicum of scientific knowledge.

I won't go into a lot of gory detail here, but one of the main themes of science since the start of the 20th century is precisely that reality is counter-intuitive. Relativity tells us that, contrary to intuition, the length of an object can change, two observers can disagree on whether or not two events are simultaneous, and if you stand on a fast spaceship travelling at 0.9xthe speed of light, and you throw a baseball at a speed of 0.9xthe speed of light, the speed of that ball observed by someone outside the ship will be LESS THAN the speed of light. Quantum mechanics tells us that objects can be in multiple locations at once, that objects can have multiple speeds, and that if we throw a tennis ball at a cement wall it's possible that the ball can pass right through the wall and be found on the opposite side (it's just VERY improbable that this will occur). Most of this is hideously counterintuitive, but all of it agrees with experimental results.

I might even suggest that the entirety of scientific history is precisely the overthrow of intuition. Intuitively, the earth is stationary and all heavenly objects rotate around this stationary earth. Intuition tells us that the earth is at the center of the universe. Naïve intuition would likewise tell us that we cannot combine an explosive gas with another gas and drink the result. Science tells us otherwise.

Your arguments from intuition or common sense may not be wrong. However, intuition or common sense is NOT evidence that most of us are likely to accept in and of itself. Such arguments have been shown false too many times in the past.

#99 Gordon,

Yes, of course, we all know the expression "no atheists in foxholes", which, if not really universal, certainly applies in some cases.

But it is not sufficient to say that your *profession* of belief is a demonstration that there is something about you that sets you apart from other humans. This appears to be what you are claiming-- you say that your revelatory experience has changed you. But any charlatan can make such a claim to satisfy a need, whether economic or simply psycho-social.

And putting aside any conscious fraud, we know that people delude themselves all the time, even to the extent of creating false memories about actual events. Since there is a ready-made narrative about 'finding God', what would be more natural than saying "yes, I've had that experience", whatever the actual case? Atheists, on the other hand, are operating in the default state-- raised by wolves, a human would hardly think "ah, today I found Jesus".

So yes, there is a difference, and the burden is on the theist to make the case if he wants to be 'believed'. Remember, we are not talking about the existence of God, but the internal state of a human being.

Gordon,

Just another point. Your reliance upon cause and effect is also suspect. Modern science has pretty much shown that uncaused events are quite possible. Quantum mechanics is random in its essential nature, despite the desperate attempts of Einstein and others to eliminate this essential randomness. For instance, consider an atom of uranium 238. In any given time period, there is some probability that this atom will transform into another element by emission of an alpha particle. If we observe this atom for many years, we likely will see nothing happen. One day, the atom decays. What was the cause of that decay? Why did it not decay at some earlier time (or some later one)? Modern science answers that question by stating that the event is random and has no cause.

I just looked in after an absence of several weeks. Honestly, I don't understand why you people even bother arguing with these cretins. They're best ignored, and if I had a blog, I'd ban them outright.

There is a growing body of experimental data that is strongly suggestive of a neurological foundation for ideological orientation in general, fundamentalism in particular. It isn't yet conclusive, but it's headed in that generation and that 's where my money lies. I've suspected it for decades.

These people are genetically programmed for authoritarianism. They see the world in terms of rigidly defined structures of hierarchical authority, and any attempt to get them to move out of that box terrifies them. They believe what they are told by their authority figures, and because their minds work that way, they assume everyone else's mind does as well, hence their constant accusations that we merely believe whatever Dawkins et. al. tell us - because they simply believe whatever their pastors and talking heads tell them. Any "research" they perform subsequently is in the service of keeping that delusion going.

This also explains the initial premise of their belief system, that we are all in a state of rebellion against a lawful sovereign, to the extent that eternal damnation is the only just consequence.

Which brings me to this; they are also psychopaths and narcissists. They've absolutely no problem with the notion that the vast majority of their fellow human beings will be tortured unimaginably for all of eternity. The only thing that matters to them is clinging desperately to that security blanket for a few precious moments longer and not having to step outside of a worldview that assuages their existential terror. If billions have to be sacrificed in the process, in this world and the next, it matters not at all to them. They see the rest of humanity as a collective acceptable loss. Zero sum game and all.

I'm told occasionally, by a "former fundamentalist", that s/he was persuaded of the error of his/her ways by arguing with liberals and humanists. However, when questioned, it invariably turns out to be the case that s/he was never really comfortable with the belief system in the first place, especially with the doctrine of eternal damnation. I'm convinced they would have left eventually anyway.

In short, there is absolutely no point in arguing with these people. For the most part, they cannot change, nor have they any desire to, and as far as the few who can change are concerned (those who win the genetic lottery) - the information is out there. If they wish to avail themselves of it, they can.

Sean T,

Did you read my last reply to you? It looks like you read what I wrote to everyone else, but missed that one.

You have failed to address my direct answer to your question on the 3 omnis. I admitted that God was neither acting as fully omnipotent or fully benevolent in how he interacts with our world. You should be having a field day with that one. But read my full post first at #52.

What you have described about Science is how it needs to depart from the normal in order to fulfill the promise of omniscience by offering explanations for absolutely everything. What follows is a need to stand common sense on its head, and still call that "Science". People understand gravity. No one understands quantum physics.

I am reminded of the words of G.K. Chesterton in Orthodoxy:

"Mysticism keeps men sane. As long as you have mystery you have health; when you destroy mystery you create morbidity. ...

"The morbid logician seeks to make everything lucid, and succeeds in making everything mysterious. The mystic allows one thing to be mysterious, and everything else becomes lucid."

By Gordon (not verified) on 24 Feb 2015 #permalink

In reply to by cipher (not verified)

Cipher,

A correction about Authoritarian Personality. It is probably not genetic in the sense of being heritable; rather it is strongly determined by childhood experience.

Whether there are immutable neurological differences associated with it I don't know.

Correction: "It isn’t yet conclusive, but it’s headed in that generation"

should be: "It isn’t yet conclusive, but it’s headed in that direction"

Zebra: fundamentalism/authoritarianism - two closely related facets of the same affliction. The evidence is heading in the direction of neurology and heritability.

#108 cipher,

Yes, fundamentalists are almost certainly people with AP unless they are 'cynical exploiters' who are just using the followers.

I would be careful though about assuming that neurological differences make the (or any) case for heritability-- remember, the brain is 'designed' for learning.

AP has been studied for some time and as far as I remember there is only a slight indication of genetics being a factor; we are all probably born capable of falling into this pattern through upbringing and early experience.

Gordon, not an answer. Magic words? - you really are talking fairy tales now. What are these "magic" words? What language does God speak? How do words create? Sounds a bit like a tornado in a junkyard forming a 747 - where have we heard that before?

Phil's problem is that he can't generalize - he thinks that every protein, every gene in every living thing needs to be explained in detail - as if there is no common thread connecting them all. We have explained and explained this and it never sticks.

By Michael Fugate (not verified) on 24 Feb 2015 #permalink

Gordon on collective punishment:

The question about children suffering shows that omni-benevolence is also limited, because God cannot stop the suffering of innocent third parties, even though that is an obviously benevolent thing to do

That is no answer at all to the question of why God intentionally engages in collective punishment. Saying you will punish kids unto the fourth generation when their great-grandfather worships another god [my earlier post mistakenly linked it to the 'take my name in vain' commandment] is an intentional choice to use collective punishment. He could have done otherwise, could have chosen to say he is going to punish nonbelievers only not nonbeliever kids and grandkids and greatgrandkids. Killing the firstborn sons of Egypt for Pharoah's decision is also an intentional choice to use collective punishment. He could've just killed Pharoah. Or hey, just killed anyone trying to stop the Jews from leaving the country. Either would've targeted far more deserving targets than the children of random Egyptians.

Phil @85 Right back at you.
Explain using the creation model how replication enzymes formed. If as Gordon says, it was a God's words, then what words were used? How did those words interact with the material world to create an enzyme? Did God create one enzyme and use it over and over again or was it de novo for each organism? Did God need a different word for each organism?, each cell?, each molecule?

When God supposed said let there be light was God talking about photons or was this a metaphor for something else like "good"? If one were a literalist, wouldn't one need to believe that the sun is not the source of daylight?

So many questions Phil, any answers?

By Michael Fugate (not verified) on 24 Feb 2015 #permalink

Gordon:

Which is why, rather than questioning God, I’m just busy figuring out how to be on the blessing side of the equation, because it sounds like an incredibly good deal to me!

That's Pascal's Wager, which I don't find particularly compelling. But be that as it may, are you saying that a pragmatic calculation of net benefit as a reason to believe is going to get you (and could get me) into heaven?

Yes Zebra,

That is why it is called "Revelation". Knowledge of the supernatural cannot be acquired in the natural state, through mere natural means.

And yes, people do often identify with a ready-made narrative, so as to prove that they fit in, even though it doesn't really describe their experience. So if you walked into a church, I don't know what sort of test you could apply to distinguish people, because they would land along all points on a scale, from the True Believers all the way down to the marginally curious.

Now, you could read some of Phil's views and you would find that he has swallowed a theological view of the Bible called "Dispensationalism". Did he come up with it himself? No. Did he experience it in some meaningful way? No. It was taught to him, and thus far he has heard no other explanation that makes better sense of the Bible for him. I do not doubt that he sincerely believes it. I think his source of ultimate authority is a bit misplaced, because no amount of citations from the Bible itself will get him to consider reading it in a different light. But he is a Believer in God just as much as I am. That I also do not doubt. Consequently, much of what the Bible says, we find ourselves in agreement on.

But to you, neither Dispensational theology, nor the Bible itself are a source of any authority whatsoever. So how could we possibly change this for you? What evidence could we possibly provide? Phil's beliefs are what they are, and they are sincerely held and he can adequately explain them. The same is true of my beliefs. What is it about our "internal state" as humans that is not coming through in what we write?

Do you doubt that we find our belief system to be a very satisfactory guide and set of presumption upon which to make decisions and live our life? Do you doubt that we believe the world would be a better place if everyone believed as we do? Should we give up rational argument, and simply tell you our life story? "God met me in this way." "God provided for me during that time of need." "I prayed, and this is how God answered."

If you go back and read my early posts on "The Meaning of Life", you should quickly see that I am not following anyone's narrative. I have my own, discovered over decades of searching. You will not hear my views taught by any church, even though I pull them all straight out of the Bible. Surely that should tell you something about me, and about my sincerity in searching for answers and my dissatisfaction with spoon-fed answers from the various churches.

Tell me what I'm missing, in your mind.

Gordon,

I certainly did address your reply to me. I stated that I am fine with denial of omnipotence and/or benevolence. That solves the logical problem; evil exists because God either cannot prevent it or God does not wish to prevent it. That's your belief. I won't argue with your belief as such. However, I still do question why such a God is worthy of our worship. You have yet to reply to this question.

As for science, nobody is claiming omniscience for science. We know we don't know everything. If we did, science would stop. Why continue to investigate when you already know everything? We don't invoke counterintuitive explanations to maintain omniscience; we do so because the observational evidence indicates that these are the best explanations. Do you really think the physics community LIKES the fact that quantum mechanics is so convoluted? Einstein, in particular, spent most of his later career trying desperately to find alternative explanations, but unsuccessfully.

The fact that reality is counterintuitive should not truly be surprising, however, at least if you grant me my own "for the sake of argument" assumption. Assume that humans did evolve from other primates. In that case, what we call "intuition" would simply be those features of our brains which increased our tendency to survive and reproduce. In the savannahs of Africa, we very rarely encounter situations where we observe submicroscopic particles or bodies travelling at appreciable fractions of the speed of light. It's not surprising, then, that our brains lack the ability to intuitively grasp what happens in those domains. Thus, evolutionary development is perfectly consistent with our lack of an intuitive understanding of fundamental reality.

Obviously, this would not be an argument against creationism, per se. If God had desired He might well have set up figuring out the fundamental nature of everything as a challenge to us. Making it intuitive would have made it too easy.

cipher:
I hold no delusions that any of these commenters will change their minds (the religious arguments all come down to "the Lord works in mysterious ways" - an unfalsifiable position). However, the discussions are valuable, since they expose the arguments on both sides for evaluation and criticism - hopeful by lurkers whose positions are waivering. This is how knowledge increases and spreads. Banning countering opinions only leads to intellectual Lysenkoism.

By Walt Jones (not verified) on 24 Feb 2015 #permalink

VS @100:

The discussion is about suffering caused by natural forces, which is what both Jason and Stephen Fry talked about.

JR's last Fry quote arguably goes into the evil of God's chosen actions, because the question "what sort of God is he?" is very often answered by both Christians and atheists by going to the bible and discussing the actions and commands of God therein.

the question here is not whether here seems to be an issue at first glance, but whether there really IS an issue. If you think it is, then you have to support that. As long as I don’t insist that there can’t be a problem, it’s perfectly reasonable for me to point out that you’re quite short on demonstrating that the problem really is one

As I said, I'm perfectly willing to concede that it's philosophically possible that all these apparently evil things might serve some greater good. If you concede that they are apparently evil. Have we reached that point?

As heaven by definition is supposed to filter for people of the proper moral character, what happens to those who have not developed morally enough to be properly judged will always be an issue.

I don't think you can handwave away the vast majority of Christian sectarian beliefs about heaven by saying "by definition" it only lets in those of proper moral character. This is simply not true. Protestant sects generally accept sola fides, an entry criteria having nothing to do with proper moral character. The RCC accepts works and faith, which is a bit more consistent with what you want to claim because works serve as an ok proxy for character much of the time, but the RCC still theologically teaches that nobody has the proper moral character to be saved, which is why God's forgiveness is necessary.
So you're just wrong about who occupies heaven, or at least Christian heaven. Which means you're just wrong in claiming this solves the theodicy problem because the people in heaven are not the individuals of proper moral character your solution requires.

Cipher,

In general, I would agree with you that "fundamentalists" lean heavily towards an authoritarian personality. Perhaps some of the Christians on this forum fit that mold. I don't know.

But I am about as anti-authoritarian as you can get. Thus the reason why I do not buy into anyone's official explanation of what the Bible means. I have come to my own conclusions exactly by NOT caring what anyone else thought. And I can tell you story after story of how Christians have rejected me personally as a result. They simply cannot handle someone who questions their particular party line.

So it is a sad irony, that the very system of belief that should free men's minds to explore the limitless possibilities of God's universe, has instead shut most of them down to a great degree.

But I will tell you this. The answers I get back from atheists sound just as parroted as any cookie-cutter answer you have ever heard from a Christian. Thus the presumption that you all have someone whose thinking you all follow, be it Dawkins or whoever. You all as an unorganized group are actually more predictable that any Christian with the pat answers you dish up.

In this very thread I have shown that not one atheist will take a serious look into how the Bible explains the Problem of Evil. They simply won't go there, even hypothetically. The train of thought is stopped dead in its tracks by a presumption of already having all the answers. So I have to conclude that atheism results in shutting down the mind just as certainly as fundamentalism does.

Read G. K. Chesterton's "Orthodoxy" and you will understand why atheism carries with it all the logic and certainty of a madman. Just as you cannot prove to a madman that he is not Jesus Christ, so you cannot prove to an atheist that there is a God. To the contrary, the more proof you offer, the more certain he becomes in his position!

Walt Jones @ #117

Two thumbs up, Walt.

Gordon @111:

I admitted that God was neither acting as fully omnipotent or fully benevolent in how he interacts with our world.

That can be an acceptable solution to the theodicy problem (depending on the details). As Sean has said, it's really only a problem/paradox for a certain type of theology. Outside of that theology, not much of a problem.

What you have described about Science is how it needs to depart from the normal in order to fulfill the promise of omniscience by offering explanations for absolutely everything.

I don't think it promises answers to everything. But I don't think we should be preferring human intuition over it when the two disagree. Science is repeatable, reproducible independent of people's cultural beliefs. Intuition often leads to contradictory results for different people that correlate with cultural background. Where science is a microscope, intuition is a mirror.

Eric @ #114,

No Eric, I am not arguing Pascal's Wager. I don't find it compelling either. This is here and now stuff, not some insurance policy, just in case God exists.

Perhaps you have forgotten how I contemplated the problems of the world, reached a conclusion, and only then did I realize that my conclusion had been sitting in the Bible for over 3000 years before I reached it.

I have seen in practice the punishment "to the third and fourth generation." My great-grandfather was a workaholic. My grandfather was unloved, and did not know how to give love. My father tried to be loving, but ended up divorcing my mother when I was very young. I determined not to allow the same thing to happen to my children, and yet in the end I had no option but to separate from their mother. Four generations, starting at least with the sins of my great-grandfather.

Am I mad at God about this? No. I am grateful that he offers such a clear explanation. In the end, these are the natural results of mistakes chosen by people in each generation. In once sense, I could not avoid this curse. But in a larger sense, I have now broken it. And what I believe will follow is a thousand generations of blessing for my children and their descendants.

These are Here and Now blessings, that will affect their life on this earth. My goal is to achieve Heaven on Earth, which is something I did not start out believing to be possible, and yet which I now know is completely achievable. Just follow the instruction manual.

Sean T:

what we call “intuition” would simply be those features of our brains which increased our tendency to survive and reproduce. In the savannahs of Africa, we very rarely encounter situations where we observe submicroscopic particles or bodies travelling at appreciable fractions of the speed of light. It’s not surprising, then, that our brains lack the ability to intuitively grasp what happens in those domains.

Yep. My pet example is our untuitive underestmation of geometric and exponential functions. We are very very bad at intuiting things like compound interest over 50 years, or how many paper folds it takes to reach the moon, or (to quote a classic) how many gold coins you'll end up with if you put one on the first square of a chess board, two on the second, four on the third, etc...
I think this particular failure is actually directly related to evolution. Evolution is a system in which mutation occurs in previously mutated genomes; its growth on growth...IOW, exponential change. People reject evolution because they don't think its possible for such an amount of change to happen in a certain period of time, a very similar underestimation as with compound interest. Yes of course religion plays into it too. But in this case the problem is pernicious because religion aligns with our (very poor and wrong) intuitions about how fast exponential change/growth occurs, while evolutionary science is counter-intuitional the same way actual compound interest results are counter-intuitional.

#115 Gordon,

This

"Knowledge of the supernatural cannot be acquired in the natural state, through mere natural means."

is troubling, contained in the same writing as this:

"Do you doubt that we find our belief system to be a very satisfactory guide and set of presumption upon which to make decisions and live our life? Do you doubt that we believe the world would be a better place if everyone believed as we do? Should we give up rational argument, and simply tell you our life story? “God met me in this way.” “God provided for me during that time of need.” “I prayed, and this is how God answered.”"

Maybe the answer to...

"Should we give up rational argument, and simply tell you our life story?"

...is...yes.

One of the most obvious reasons for me to doubt your 'belief' is that your behavior looks very much like that of someone who needs to convince others in order to convince himself. That is all the explanation I need for your actions; there is no need for any other event or condition to explain them.

Am I mad at God about this? No. I am grateful that he offers such a clear explanation. In the end, these are the natural results of mistakes chosen by people in each generation. In once sense, I could not avoid this curse. But in a larger sense, I have now broken it. And what I believe will follow is a thousand generations of blessing for my children and their descendants.

You're taking a kind of egocentric view of this by considering yourself the fourth generation. Maybe your great-grandfather got caught up in the curse from his great-grandfather. What makes you think you are the end of the line? Shouldn't your kids and grandkids be punished for your dad's shortcomings?

If the answer is "no" -- and I hope it is -- then the whole thing makes no sense.

By Another Matt (not verified) on 24 Feb 2015 #permalink

Gordon:

I have seen in practice the punishment “to the third and fourth generation.” My great-grandfather was a workaholic. My grandfather was unloved, and did not know how to give love. My father tried to be loving, but ended up divorcing my mother when I was very young. I determined not to allow the same thing to happen to my children, and yet in the end I had no option but to separate from their mother. Four generations, starting at least with the sins of my great-grandfather.

Am I mad at God about this?

If you think God cursed your family and caused bad things to happen to them for the sin of your great-grandfather, then holy cow you absolutely should be, because that means He punished you and your father and grandfather for crimes you didn't commit!

Maybe we just need to boil it down to that singular question: do you think it is benevolent to punish people (and cats, dogs, etc..) for crimes they didn't commit?

eric,

JR’s last Fry quote arguably goes into the evil of God’s chosen actions, because the question “what sort of God is he?” is very often answered by both Christians and atheists by going to the bible and discussing the actions and commands of God therein.

And since Fry STARTED with the problem of suffering, the Problem of Evil relates to general suffering and not direct actions, Jason's examples were all aimed at general suffering, and talking about direct actions is a different sort of argument entirely -- much more theological in nature -- let's try to stay on topic, shall we, without diving into another complicated and different argument?

As I said, I’m perfectly willing to concede that it’s philosophically possible that all these apparently evil things might serve some greater good. If you concede that they are apparently evil. Have we reached that point?

You tell me. What do you think the below means?

Let’s address the “doesn’t look benevolent” point here. At first glance, it does look like there is an incompatibility there. That’s why there’s a philosophical debate in the first place.

Which were the two sentences that preceded where you started quoting me. You DID read them, right?

I don’t think you can handwave away the vast majority of Christian sectarian beliefs about heaven by saying “by definition” it only lets in those of proper moral character. This is simply not true.

Okay, brass tacks time, then: do you think that there are people in heaven who do bad things? Do you think that it is consistent with ANY definition of heaven to have people there who do immoral things? And, for bonus points, do you think that there's any way to, in fact, avoid my consequence for any view of heaven that would say that not having free will in heaven would be a BAD thing?

By Verbose Stoic (not verified) on 24 Feb 2015 #permalink

Do you think that it is consistent with ANY definition of heaven to have people there who do immoral things?

Of course. This is basic Christian doctrine -- after Adam nobody can avoid doing immoral things, so (according to the doctrine) heaven should be empty if justice is all there is to it. But it isn't, because mercy is just as big a part of it.

As an aside: I grew up in fundie culture (young-earth creationism, Left Behind, the whole bit). Their view of heaven leads to some magical thinking: it's not what you do that matters but what you believe, and what you believe about what you do. Everyone is immoral, but only the people who believe correctly go to heaven. It's why Catholics are just as damned as atheists in the fundie mind: Catholics believe the wrong things about the nature of sin and atonement, and are worshipping a false god. But if you are a real true Christian, as long as you pray before you make a decision, God will make sure you make the right one. If it means you lose your house, then God's plan for you was for you to lose your house. If I had a dollar for every time someone in my family told me, "God is telling me to..." or "God wouldn't let me be wrong about something this important."

By Another Matt (not verified) on 24 Feb 2015 #permalink

Unless you meant that people continue to do immoral things while they're in heaven, which is a different question....

By Another Matt (not verified) on 24 Feb 2015 #permalink

Another Matt,

Of course. This is basic Christian doctrine — after Adam nobody can avoid doing immoral things, so (according to the doctrine) heaven should be empty if justice is all there is to it. But it isn’t, because mercy is just as big a part of it.

My point wasn't about the entrance criteria, but about what people do when they're in heaven. I'm not a theologian, but my understanding is that it is indeed generally assumed that people in heaven will sin no more by pretty much all Christian denominations.

By Verbose Stoic (not verified) on 24 Feb 2015 #permalink

Sean T and Eric,

I think we have made some progress, as you both acknowledge that I have answered the Problem of Evil, and Sean at least then raises a question that naturally follows on from my answer: Why is such a God worthy of our worship?

I have given hints at my answer to this question, but it would probably take a book to flesh it all out. I have said that I consider that providing us with Free Will was an incredible gift, and that it achieved a higher good than simply placing us like animals with limited choices in a world where nothing could ever go wrong. But I have not detailed what I mean by Free Will. That would take at least one chapter. And I have not detailed why I consider this to be a higher good. That would take at least half the book.

Two things come to mind from Sean's question. First, I began to genuinely worship God with all my heart back in the early days of my faith, as a young adult who understood almost nothing of what I know today. I accepted most of what I was told about God simply because I had no experience to go on.

There is something powerful in acting on faith out of an almost child-like innocence. I worshiped God simply because I chose to, not because I understood all that I do about him today. That worship experience is more real to me than most of what I do in everyday life today. It is not something I can convey to others. I simply know today that it was right. I was not led astray by the advice I followed almost blindly.

Which means that I cannot really explain to you in any meaningful way why I worship the God of the Bible. I can only demonstrate to you that his Word is true, and I can testify that it has proven true in my life time and again.

But there is a second problem. Logical analysis is not always the best way of arriving at a full understanding. Some things can only be experienced, and in fact, analyzing them will tend to destroy the experience.

As an Engineer, I was overly analytical about everything, but this point was brought home to me by a quote from an old professor from Wheaton Bible College. He would address his class and say:

"Fellas, ya can't kiss yer girl and think about the kiss at the same time."

And from the day I heard that, I knew that all truth was not to be discovered by analyzing facts. I could see that there were parts of life that went exactly against this, in which the truth would elude me if I insisted upon constantly analyzing everything. So I changed my approach, and my understanding of my world then started to grow by leaps and bounds.

For me, the proof is in the pudding. I have taken the Bible as the guidebook for my life, and the results have been more than satisfactory. And the more closely I followed it, especially the teachings of Jesus, the more I found other parts of it unexpectedly proving true in my life.

It should be obvious enough that in order to understand why you should worship a Creator who would set aside part of his omnipotence and benevolence and give us Free Will that included the ability to cause harm to others, you have to understand more completely the nature of that Being.

It does not help to cherry-pick a few verses and say that they prove he is Evil. Eric, I cannot answer why God dealt with Pharaoh and the Egyptians the way he did. I must simply trust that he knew better than me. On the other hand, I have explained my understanding of the curses "to the third and fourth generation". That passage has always been deeply meaningful to me, and helpful as I described.

It would take a few chapters in the book to set the record straight on God's character, especially for modern atheists who are only looking for bad things to say about him. The picture you paint is completely misguided. It does not describe the true God, and it is not even fair to say that the Bible describes him the way you think it does. There is a bigger picture, and it is also a much better one. The God of the Bible is truly worthy of our worship.

As for our Science and intuition (or common sense) discussion, the point I really want to get past is that of Design. I made the point that the very use of the word "Wall" (when used properly, Eric), had the notion of Design built into it. If this is not true, then language is meaningless, and what in the world are we doing typing words into a computer for one another to read?

Thus, when we look at a pile of rocks and call it a Wall, we merely acknowledge our intuitive sense that there is order in the pile of rocks, and therefore someone must have arranged them that way. If you accept the validity of that type of intuitive knowledge, then you accept the basic premise of Intelligent Design.

If you want to stand this on its head, then you are using notions such as quantum physics to say in effect, that because it IS intuitive, therefore it is NOT scientific. So choose one, and maybe we can progress this conversation as well.

Gordon #96:

You're a lot better at dodgin' and duckin' than debatin', Gordon.

What makes you think your revelatory experience was anything other than a delusional episode?

Why, in this world full of empirical evidence, is there none at all for gods?

It is not my job to tell you which end is up.

And a good thing too, because you do not know which end is up.

“Fellas, ya can’t kiss yer girl and think about the kiss at the same time.”

Man, is he ever doing it wrong.

Thus, when we look at a pile of rocks and call it a Wall, we merely acknowledge our intuitive sense that there is order in the pile of rocks, and therefore someone must have arranged them that way. If you accept the validity of that type of intuitive knowledge, then you accept the basic premise of Intelligent Design.

Except that to infer design, you're comparing the wall rocks to nature, which you want to claim is also designed. If there's a divine watchmaker, then everything is a watch.

By Another Matt (not verified) on 24 Feb 2015 #permalink

Zebra @ #124

Yes, I agree. Most people come on forums like this to hash out their own beliefs. They are indeed trying to convince others in order to convince themselves. That perfectly described me up to about ten years ago.

Then I went silent. I stopped trying to convince others because I realized how little I understood, and how fast I was learning new information. The few things I wrote I found myself rewriting a few months later, and disposing of entirely a year or two later.

I have only been back online, sharing what I have learned, in the past 3 months. But now I have a coherent and complete belief system. Even others here acknowledged early on that my beliefs were more consistent than any other Christian they had met.

If you read my last posts to Jason, you can also see I have no intention of sticking around here. It has been fun, but I really have no need to convince myself, OR others. But I also acknowledge that you have no independent way of verifying that this is true of me.

So I will merely point out that the accusation (if that is what it is) sticks to everyone else here as well. Even to Jason. Even to you. In fact, it is almost certain that you are the one here most in need of convincing himself, since you are the one who raised the issue.

The difference between us is this: I will not doubt your sincerity unless you give me a good reason to doubt it. And even if I knew for certain that you were here to convince yourself, I would still not doubt your sincerity in voicing what made the most sense to you on the day you stated it.

Have a look at Walt Jones' post @ #117. This is all to the good.

Gordon,

I certainly don't (and I don't think anyone else does either) consider a proposition unscientific because it is intuitive. I merely pointed out to you that reality is often counterintuitive. Where you do make your mistake, though, is pointing to the pile of rocks, calling it a wall, and using the fact that you've called it a wall as evidence that it's designed. The word "wall" itself implies design. Calling the pile of rocks a "wall" is the same thing as calling it designed. You may be right calling a particular pile of rocks a wall, but you can't use the fact that you've chosen to call that particular pile a wall as evidence that it's designed. That's tantamount to meeting a man for the first time, calling him a bachelor and using the fact that you've called him a bachelor as evidence that he's not married.

Just to get further toward the essence of the question: what features define "design"? How can you look at one pile of rocks and say definitively "it's a wall" and look at another pile of rocks and say "it's just a pile of rocks"? It's not as clear cut as you might think. I'll give you an example. Suppose you look at your coffee table and you see ten pennies on it. 3 of the pennies are showing 'heads", 7 are showing "tails". Did someone intentionally place the coins that way or did someone just drop the coins on the table and they randomly happened to be arranged that way? Does your answer change if I tell you all ten are heads (or tails)? If so, why?

Another Matt @ #125 and Eric @ #126,

Look at this from a practical standpoint. I have discussed these things with my father and my grandfather. They told me what I know about my great-grandfather. Before then, I simply have no information to go on. So the "third and fourth generation" is more of a practical thing, in that you can see the ripple effects across the generations of people you actually know, or who they knew. In my case it just happened to be that I could see a pattern that continued through 4 generations. I believe Moses wrote it this way precisely because we would always find ourselves as the third or fourth generation of the people we could talk to or learn about. This makes it personal, not egocentric.

I can only believe by Faith that I am the end of this line. I have read the instructions, and followed them to the best of my ability, and I believe they will have the positive result that the Bible says they will. If there was no way to break this cycle, why would God bother telling us about it in the first place?

Eric, we are talking about the pernicious nature of sin here. Sin is bad precisely because it harms innocent third parties. This is a warning to parents: persist in disobeying God, and even your great-grandchildren will experience the negative effects here.

I think you view this as God intervening to make sure the next generation gets punished. But he does not have to intervene at all. It simply happens. The mistakes get passed down. Bad habits get learned and repeated, or worse. The point Moses is making here is just how bad it is when we disobey God, and how infinitely better it is when we obey him.

These are simple facts of life. If you are an alcoholic, your grandchildren WILL suffer. And you have no God to blame for it. It is simply a fact of life. The Bible explains these facts of life in such a way that it:

1) Warns us
2) Tells us not to do it
3) Tells us the negative consequences
4) Tells us how to fix it, and
5) Tells us the consequences of getting it right

You read the warning or the negative consequences, and conclude that I should be mad at God for the mistakes of my great-grandfather. They are my great-grandfather's mistakes, not God's. And the consequences were such that, with no intervention on God's part whatsoever, I still felt their effect 4 generations later.

Remember, God HATES sin. He doesn't love punishing anyone for it. He hates sin precisely because it wrecks our lives. This is what the Bible clearly teaches, so you must read these words of Moses within that context.

Gordon,

Also, to follow up on what I discussed with you previously. Human intuition is something that developed specific to our environment. One of the things that we find intuitive is pattern recognition. We are very good at looking at data and finding patterns in that data. That had obvious survival advantage for us early on in our evolution - a given pattern might mean a predator is nearby, another pattern might mean we can find food nearby, etc. The flip side of this, though, is that while we are good at seeing patterns that are present in seemingly chaotic data, we are equallly good at seeing patterns in seemingly chaotic data that really aren't there. The traditional constellations and seeing shapes in the clouds are a couple of good examples of this.

I would assume that one of the things you would use to detect design is whether or not you can see a pattern in whatever it is you are looking at. Given my first paragraph, I think it's important to be very careful using that method, though. It's not always true that there is an underlying order every time we thinik we see a pattern; we see patterns in essentially random data. It's just the way our brains developed.

Gordon,

I still think I come back to my main question to you. If what you say is true, why should we worship God? In reading your posts and to paraphrase you a bit, it is a natural consequence of sin that the sin affects those around us. If my grandfather is an alcoholic, it's just a natural consequence that his alcoholism will affect me negatively.

My question then is, could God do something about this if He chose to? Could He intervene and mitigate the negative affects that my grandfather's alcoholism has on me? If the answer is no, then no further discussion is needed. Obviously, I can't claim that God would have done something wrong if He wasn't capable in the first place. If the answer is yes, though, I would certianly be questioning why the heck we should worship such a God.

Another Matt @ #128,

You have described "fundie culture" to a tee.

In posts #190 and #196 of "Proving God Exists", I try to explain to Phil the very problem that you describe. They think it is all about "believing" the right notions, when this is not at all what the Bible actually says.

Gordon sez

Remember, God HATES sin.

Instead of begging the question, why don't you explain how I can test your claim that gods are real.

Instead of arguing from a false premise, why don't you establish the truth of your premise?

Everything else is specious sophistry.

Phhht @ #132,

Your reply is in itself, delusional.

Having already dismissed all evidence as not good enough for you, you then make the silly claim parroted by all atheists that there is "none at all".

You want empirical evidence? Look up at the stars. Consult your Science books. Billions of stars in billions of galaxies that would take lifetimes to traverse IF we were able to travel at the speed of light. By comparison, you are just a little speck. A nothing.

And yet, this little speck of nothing has a spark of the Eternal within him which enables him to contemplate the vastness of the universe even though he cannot possibly comprehend it. What is this connection to the Infinite that you possess? Where did it come from?

For millennia such empirical evidence was sufficient for mankind. Yet today we are so full of ourselves that we simply dismiss that, probably with a not-even-clever put-down, and then once again mindlessly mouth the first sacred tenant of the Atheist's Catechism: "There is NO evidence!"

Thanks for the compliment about the importance of open dialog, Gordon, I regret that the only other common ground I can find is your statement that atheists always use the same arguments. That's true, because all we want is evidence to support the assertion that there is a god. Your revelation is no more meaningful to me than your dreams.

You seem to have found a lot of wisdom in the bible, but what about it is evidence for a god? Testable evidence, of course. What about that Psalm is testable? The existence of angels?

Asking you - or Chesterton - for evidence is not evidence of insanity. Asking your opponents to defend their sanity instead of providing evidence is a red herring, bordering on ad hominen.

If your god exists, he should know what would convince us, and should be willing to share that knowledge with you. Try praying and asking for it.

By Walt Jones (not verified) on 24 Feb 2015 #permalink

In reply to by Gordon (not verified)

Gordon,

You apparently do not understand what "empirical" means.

It means "testable," Gordon.

How can I TEST your claim that the stars are evidence for gods? How can I TEST your silly claim that I have a
"spark of the Eternal" in me - whatever that nonsense means?

So you see, Gordon, your claims that stars and alleged divine sparks are nothing but bull, nothing but more baseless claims.

Unlike all the really testable evidence which fills the world to overflowing, there is not an iota for the existence of gods.

Why is that?

And I'll repeat my other question. How do you know your revelatory experience was anything other than an everyday, common-or-garden delusional episode? They happen all the time, Gordon. Why not to you?

VS: sounds like we agree on the basic result of a theodicy-style analysis: the Christian conception of God is apparently not benevolent. There may be some way in which all these apparent malevolent actions are truly benevolent, but if so we don't currently understand that way. Fair enough?

Okay, brass tacks time, then: do you think that there are people in heaven who do bad things?

Protestant heaven admits poeple who did evil (all the way up until the day they died) as long as they have faith. Catholic heaven admits the same, as long as you've done some unstated number of good acts and confessed the bad. What these faiths think these people do in heaven and how free they are to keep doing the bad things they liked doing in life is somewhat opaque to me. Frankly, I don't think there's going to be a single solid answer to that; you're going to get back as many separate answers as there are believers. But my point was about entry requirements, and AFAIK no major Christian sect agrees with your entry requirements.

Do you think that it is consistent with ANY definition of heaven to have people there who do immoral things?

It doesn't seem consistent with the common notion of heaven, no. Then again, personally I think the common notion of heaven is vague and somwhat self-contradictory, so it doesn't surprise me at all that major Christian sects say seemingly contradictory things about it (i.e., that sinful faithful people are in it, that they retain their free will without any mind-control from god, yet they have no desire or ability to sin).

And, for bonus points, do you think that there’s any way to, in fact, avoid my consequence for any view of heaven that would say that not having free will in heaven would be a BAD thing?

That's a bit of a convoluted question. Could you clarify it?

your claims that stars and alleged divine sparks are nothing but bull

should read

your claims that stars and alleged divine sparks constitute empirical evidence for gods are nothing but bull

VS @130:

My point wasn’t about the entrance criteria, but about what people do when they’re in heaven.

Then you phrased it very poorly indeed, because "As heaven by definition is supposed to filter for people of the proper moral character..." is precisely an entrance criteria.

Sean T,

Sorry, I picked up in the middle of a discussion I was having with Eric. In that discussion I had already established that the pile of rocks had a pattern or order to it that we recognized as a wall. My last point to him was that, by the use of the word "Wall", we were acknowledging Design.

In other words, because of the pattern or order we observed, we did not consider this just another pile of rocks, but considered that someone had taken the time to move them into this position so as to build a Wall - the intentional application of intelligence. By mere recognition of the pattern, we assumed human intervention and all else necessary to Design.

My answer to your question in #138, "Could God do something about this if he chose to?" is, of course he could. In my case he did choose to. He arranged circumstances such that I found the answers I needed, and those led me to a better life, free from depression and other problems that plagued me in my younger years. But I did follow his instructions. I sought him with my whole heart.

The real question is: Why doesn't he choose to do this for everyone? I only have a partial answer, and that is that most people choose not to do this for themselves. Even when given access to God's answers, they reject them. And I am not picking on Atheists here.

I live at present in Manila, the Philippines. A huge percentage of the population are Christians, but they are also poor. As a fellow Christian, I have come here to try to help them. To my amazement, I find that few of them actually want to be helped. Sure, they want handouts, but that is not real help. I am concerned with breaking the cycle of poverty, but this is something they are not about to give up. Their wrong choices keep them in poverty, but even when you try to explain how different choices, found in the Bible they claim to believe, would make their lives better, they are simply not interested. It can be very discouraging.

So if you are God, how do you give people Free Will, and then turn around and help them when they don't want to be helped? At this point I can only trust God's omniscience, that he knows who will be receptive to his help, and he will show his benevolence by helping them. That does describe my own experience, which is enough to make him worthy of my worship.

Gordon @131:

Eric, I cannot answer why God dealt with Pharaoh and the Egyptians the way he did. I must simply trust that he knew better than me.

And that is why I call what you're doing apologetics instead of standard reasoning.

The picture you paint is completely misguided. It does not describe the true God, and it is not even fair to say that the Bible describes him the way you think it does. There is a bigger picture, and it is also a much better one.

AFAIK, you have never accused me of misrepresenting the facts of the biblical stories; of getting the stories wrong. You and I both agree I get the stories factually correct. So if my picture is misguided, then the book is too. Isn't it?

I made the point that the very use of the word “Wall” (when used properly, Eric), had the notion of Design built into it. If this is not true, then language is meaningless, and what in the world are we doing typing words into a computer for one another to read?

Thus, when we look at a pile of rocks and call it a Wall, we merely acknowledge our intuitive sense that there is order in the pile of rocks, and therefore someone must have arranged them that way.

I understood your point, but I think its wrong. Humans use design and purpose language all the time to describe things we don't think have an intelligent design or purpose behind them. Our word choice does not imply design in fact. It doesn't even always mean that the speaker things something was intelligently designed. Thus we may speak of the a termite city, or carpenter ants. The American southwest is dotted with mesas (spanish for "table"), but nobody suggests they are intelligently designed tables. The 1970s description of the earth as the Big Blue Marble did not imply everyone thought it was designed. This whole "if I use a design word, I am intuiting intelligent design" is just wrong.

If you accept the validity of that type of intuitive knowledge, then you accept the basic premise of Intelligent Design.

Well, I obviously don't. I don't think English word choice is a reliable guide to metaphysical reality. In fact I have a difficult time understanding how anyone who gives that notion serious thought could believe it.

I love it when christian believers harp on free will and the bible, because those two beliefs open a Loophole to evade the extortion inherent in the heaven-hell myth.

If I have free will, and since the bible says eternal life is a "gift," I can freely and willingly reject that gift. No eternal life for me, thanks, and thus neither heaven nor hell.

Here in reality I'm going to die anyway, finally and completely, so I have nothing to lose except that attempt at coercion of belief.

You're getting pretty close to prosperity gospel territory, there, Gordon.

By Another Matt (not verified) on 24 Feb 2015 #permalink

Phhht @ #142,

Empirical: "based on, concerned with, or verifiable by observation or experience rather than theory or pure logic."

But that's just the first definition that Google pulls up.

So this is empirical evidence, detailed in Psalm 8:

O Lord, our Lord,
How majestic is Your name in all the earth
...
When I consider the heavens, the work of Your fingers,
The moon and the stars, which You have ordained;
What is man that You take thought of him,
And the son of man that You care for him?
Yet You have made him a little lower than the angels,
And You crown him with glory and majesty!
You make him to rule over the works of Your hands;
You have put all things under his feet

What do you want instead? Do I need to put God on a leash for you, so you can lead him around like a dog and show him to people? The god you demand would be nothing more than a magic trick. Demons can be summoned, so I hear. God cannot. Funny that.

I am not sure where you got the idea that I had a single "episode" of some blinding revelation, such as the Apostle Paul had on the road to Damascus. I have had no such experience. Rather, the revelation is found in the Bible itself. But it is backed up by time and again being proven true in my life in very concrete, practical ways.

Read chapter II of Chesterton's "Orthodoxy" to see how close your own demand comes to insanity:

http://www.pagebypagebooks.com/Gilbert_K_Chesterton/Orthodoxy/The_Mania…

The above link will take you to page 1 of that chapter, but make sure you read all ten pages.

Eric @ #143,

You have got the Heaven issue exactly right, that Christians have got it mostly wrong. Opaque, vague and contradictory all apply.

The point that both Protestants and Catholics miss is what I explained to you earlier. Heaven is Heaven (whether on Earth or elsewhere) precisely because it is populated ONLY by people who choose to do good all the time, of their own Free Will.

Which begs the question: If they don't know how to do this here and now, how do they expect to be able to do so once they reach Heaven?

Thus the reason I find myself at odds with most Christians of any stripe. I can tell them how to achieve Heaven on Earth, here and now, and it's all in their Bible. But they don't want to know! They think they have their ticket to Heaven, and that is all that matters. Boy are they going to be surprised.

For their sake, I hope reincarnation is true!

Gordon:

So the “third and fourth generation” is more of a practical thing, in that you can see the ripple effects across the generations of people you actually know, or who they knew

No, I brought it up as a direct quote of Exodus 20:5 (NIV): "You shall not bow down to them or worship them; for I, the Lord your God, am a jealous God, punishing the children for the sin of the parents to the third and fourth generation of those who hate me"

That means that if your father worshipped a different god, God will choose to punish you and your kid for it. Why? Because he's jealous. He says so Himself. I find that malevolent. AIUI, your position on the matter is that since it is also true that if your neighbor worships God, her kids will be rewarded for it unto a thousand generations, this makes it morally defensible for God to punish your kid for your father's "crime" of worshipping the wrong god. The good is greater than the evil, which makes the whole package benevolent. We obviously disagree on that logic, but I think I have captured both our positions at least accurately.

This is a warning to parents: persist in disobeying God, and even your great-grandchildren will experience the negative effects here.

Then I would say that the bible did a quite horrible job in communicating God's message, because the words are pretty clear and they aren't a "bad decision warning." They are clearly a "I will punish you for doing it" warning. It also somewhat flies in the face of reality to claim that not worshipping the Christian God correlates with family troubles and worshipping the Christian God correlates with family happiness (unto a thousand generations, no less!). IOW even if we assume your interpretation is correct and this is simply a 'bad decision warning," statistically and empirically the decision doesn't appear to be bad; if you're right about the meaning of the message, then God's advice appears to be wrong.

Remember, God HATES sin. He doesn’t love punishing anyone for it. He hates sin precisely because it wrecks our lives. This is what the Bible clearly teaches,

That sounds like an abusive spouse to me. And frankly, if he's going to snap his fingers and alter the world, I would humbly offer that
[snap! Sinner gains understanding of empathy, consequences]
is a far more benevolent and effective alteration than
[snap! punishment for sinner]

Gordon:

The god you demand would be nothing more than a magic trick. Demons can be summoned, so I hear. God cannot. Funny that.

ID theory combined with Lenski's experiments would indeed imply that God can be summoned and made to dance. Since ID maintains that positive traits cannot evolve, then every time Lenski's group takes an older generation of e coli out of the fridge, puts it in a petri dish with citrate, and watches it independently develop the citrate-eating mutations, he is effectively summoning God and making him dance. So ironically enough, it is Intelligent design that turns God into a dancing bear while evolutionary theory does not.

The point that both Protestants and Catholics miss is what I explained to you earlier. Heaven is Heaven (whether on Earth or elsewhere) precisely because it is populated ONLY by people who choose to do good all the time, of their own Free Will.

Well Gordon, I suspect you're in for a very lonely afterlife.

Eric,

You go back and forth between sensible and being silly. And I go back and forth between writing you off and responding to you.

On the Design issue you just want to argue simply because you cannot be brought to concede the obvious.

A "termite city" is called that because of the order recognized in what the termites have done.

When I apply the word "Wall" to a pile of rocks, I am speaking directly to the order I detect in how the rocks are arranged. I am claiming to see a Design. And I thereby import all other presumptions surrounding the Design, such as the need to move the rocks to that location, and to place them one on top of the other, etc. I claim that some man, even though unknown to me, applied his intelligence, his will and his strength to the intentional building of that Wall.

All this is communicated the moment I point to some rocks and say, "That's a Wall."

Now, I assume you to be a well educated adult. So please don't make me spell things out in excruciating detail again, as if to some child in special education.

If I came here trying to prove how illogical atheists can be at times, I have succeeded. But I actual much prefer it when you all give intelligent answers and make me have to think. You have proven yourself very capable in this regard, and so I hope we can progress this issue from here.

Another Matt @ #149,

Wow! That came out of left field, Matt.

Perhaps you can give me an example of what sounds like Prosperity Gospel, because I couldn't get farther from that if I tried.

If you read all of my exchange with Phil, you would find me recommending such biblical admonitions as, "Sell all you have, give the money to poor, and come follow me."

If that is what Prosperity Gospel preachers are teaching these days, then perhaps I need my own television show, as my beliefs are more popular than I thought.

@kettle - Gordon has a message for you.

The use of "wall" means design? And you say eric is being silly?

By Walt Jones (not verified) on 24 Feb 2015 #permalink

In reply to by Gordon (not verified)

Walt Jones,

What atheists want is certain very limited types of evidence. Which is funny in light of some of the other issues I have argued here. For instance, there seems to be no problem waiting around for 300 years for some scientist to prove abiogenesis. However, obeying God's commands for say, 10 years, and looking at the results that has on your own life would be unthinkable.

And no one has even asked you to do that. Rather, folks such as myself have testified that we have done just that, and that the results have matched with what we read in the Bible. And we are happy to explain this all to you in detail. But that is not the type of evidence that you want.

Jesus already told me 2000 years ago, via with a parable in Luke 16, what would convince you. The punch line is:

"If they do not listen to Moses and the Prophets, they will not be persuaded even if someone rises from the dead."

In short, there is no "evidence" that will satisfy you guys. But what you can also see is that my God did in fact provide me the knowledge of what would convince you. I didn't even have to pray to him for it! But I'm also sure that the fact of God answering your question before you asked it does not impress you either.

Now I do not claim that any of you atheists are insane, with the possible exception of Phhht. And he is only suspect because he can't get away from using the word "delusional". But you really do owe it to yourself to read that chapter from Chesterton. It puts the repeated demand for "evidence" of God in perspective.

Chesterton is not calling atheists insane either. What he is really pointing out is summarized in the excerpt I quoted in post #111 to Sean T. There is a need for mystery in our life, and Chesterton devotes the entire chapter to laying out exactly why from start to finish.

Now, having to wrestle with the Problem of Evil, and attempt to answer it to a forum of mostly atheists, has challenged me and expanded my thinking and deepened my faith. That is why I complimented you for recognizing the good that comes from exchanging opposing views. I have definitely benefited from it, but it appears to me that most of you have not. I can only say that it is your loss if you fail to allow Chesterton to challenge you, or if you fail to understand my points such that they challenge you.

By Gordon (not verified) on 25 Feb 2015 #permalink

In reply to by Walt Jones (not verified)

Gordon @150,

In other words, you cannot supply any test whatsoever to back up your claims. You cannot say how I can confirm the evidence you claim exists. You cannot say how I can determine whether you are correct in your claims, or whether you are just as loony as you seem.

Thanks for the clarification about the revelatory experience. I did indeed think you had had a delusional episode. Now I see that instead, you have had a long, extended period of delusion.

It's easy to counter a charge of delusion, Gordon. All you have to do is to say, I am not deluded, and here is the testable evidence which backs me up.

The world is chock full of empirical evidence, for everything from apples to zebras, but there is not a scintilla of such evidence for the existence of gods.

So you have no such evidence. What, then, makes you think gods are real?

Hmmm. So saying a cliff is a "sheer rock wall" is a statement of design instead of a convenient description. Who knew?

That's what happens when you are used to twisting words to defend religion: you twist them in everyday usage too.

"Empirical: “based on, concerned with, or verifiable by observation or experience rather than theory or pure logic.”"

So are you saying you prefer this to the definition presented to you, or are you dismissing this one in favor of a definition you don't state?
If you are presenting this as an alternative - you are ignoring the "verifiable" portion - as many have pointed out. Internal biblical references aren't worth the paper they are printed on, and your beliefs are worse less.
If you are dismissing this one - likely because of the "verifiable" portion - you haven't given an alternate definition. Answering a simple question seems to be very difficult for you.

And - next time you bring up Chesterton - it would be highly amusing to see you attempt to wash away his anti-Semitism.

Gordon:

Which begs the question: If they don’t know how to do this here and now, how do they expect to be able to do so once they reach Heaven?

The traditional answer is theosis: http://en.wikipedia.org/wiki/Theosis_(Eastern_Orthodox_theology)

It's also why RCC doctrine has Purgatory -- it's precisely because this world is so fallen (according to the doctrine) that everyone except the most saintly saints needs to be purged of sin. When Jesus says "go and sin no more," it's likely that he was not giving a commandment but rather announcing a healing, as he might with "take up your mat and walk."

Perhaps you can give me an example of what sounds like Prosperity Gospel, because I couldn’t get farther from that if I tried.

Here's the passage I found toeing the perimeter of prosperity gospel territory:

I live at present in Manila, the Philippines. A huge percentage of the population are Christians, but they are also poor. As a fellow Christian, I have come here to try to help them. To my amazement, I find that few of them actually want to be helped. Sure, they want handouts, but that is not real help. I am concerned with breaking the cycle of poverty, but this is something they are not about to give up. Their wrong choices keep them in poverty, but even when you try to explain how different choices, found in the Bible they claim to believe, would make their lives better, they are simply not interested. It can be very discouraging.

There is more than one way to read this, but the "prosperity gospel" interpretation would be, "if you're the right kind of Christian, you will no longer live in poverty: God will reward you for living the right way." Another way to read it, which is probably more charitable, is more like the argument put forth here:

http://www.patheos.com/blogs/slacktivist/2014/12/10/ignorant-christians…

In any case, some of the pushback you're getting might be due to the fact that is galling to have someone who is not a cultural insider come and try to tell you how to live, and to be told what does and does not count as "help."

By Another Matt (not verified) on 24 Feb 2015 #permalink

In any case, some of the pushback you’re getting might be due to the fact that is galling to have someone who is not a cultural insider come and try to tell you how to live, and to be told what does and does not count as “help.”

It is just as likely to believe that no offer of help is forthcoming unless they agree to follow his god in his way.

Wow. I posted yesterday at #66, now we’re up to #162.

cipher, I engage believers in these debates because I learn from the effort. If changing their minds was the goal, this would be futile, but that’s not my goal.

For instance, Sean T wrote that “The POE is simply a ‘let’s assume for the sake of argument’ kind of thing.” Exactly right, and an obvious phrase I was looking for. Sean T also asked Gordon “why should we worship a moral agency that has the power to reduce suffering but refuses to do so? I’m not being ungrateful or blaming God here; we could still be grateful to God for creating us” and still acknowledge Gods many sins. Great question.

On the topic of “intuition” Sean T wrote that “any argument relying on intuition or common sense will be immediately suspect to those of us with a modicum of scientific knowledge [because] one of the main themes of science since the start of the 20th century is precisely that reality is counter-intuitive. ... I might even suggest that the entirety of scientific history is precisely the overthrow of intuition.” I’m going to totally steal that!

See Noevo wrote that “We’re told that the random, non-rational process of evolution continually advances a biological robustness which includes longer life ...

I don’t know who told you that, but they were wrong.

When Walt Jones questioned that, See Noevo replied, “Just about every pro-evolution piece I’ve read.

Strange, because I’ve never read any biologist’s claim like that.

Zebra asked me, “Has anyone asked of the people who claim to be Believers how we can determine which among them are sincere and which are not? ... there should be a metric ...

Sincerity is a quality, so any determination of sincerity is necessarily a qualitative assessment. At best some pseudo-quantitative ranking could be assigned, but it would be pure conjecture.

For myself, I take someone’s claim of sincerity at face value unless they do something that casts doubt. At the end of the day, it’s not critical to these discussions; only claims that can be examined are worth bothering with. If Gordon, Phil, or anyone else testifies to their sincerity, I don’t question it. I reserve critique for claims of reasonableness.

Zebra wrote to Gordon that “One of the most obvious reasons for me to doubt your ‘belief’ is that your behavior looks very much like that of someone who needs to convince others in order to convince himself.

In deed. And given his bizarre concept of free will in which Information destroys Freedom, and Knowledge is oppression, Gordon’s regarding his personal revelations and spiritual experiences sound unreliable.

Verbose Stoic wrote that “So the whole question — and not only half of it — is whether not preventing someone else’s suffering when you can do so is necessarily immoral.

Failing to prevent someone else’s suffering when you could is immoral so long as preventing that suffering does not inflict a greater harm and does not cause needless suffering for third persons. The totality of the circumstances do need to be considered, but GENERALLY, Failing to prevent someone else’s suffering when you could is immoral.

If some deity is in any meaningful sense omnipotent, then failure to prevent suffering is ALWAYS immoral (because they have the power to prevent it without inflicting collateral harms). Protecting our “free will” does not suffice to justify this immorality because there’s always a way to protect the free will of the one who is suffering.

Apparently eric wrote that, “The theist is making the claim that this entity exists, has done certain actions, and is benevolent. The theodicy theorist is pointing out that the actions the theist claims this entity has done don’t look benevolent.

I would rephrase this as “The theist is making the claim that this omnipotent entity exists, has done certain actions, and is benevolent. The theodicy theorist is pointing out that the actions the theist claims this omnipotent entity has done causes unnecessary and unjustifiable suffering.

Verbose Stoic wrote that “... the theist is defending against an assertion, not demonstrating one. So they don’t need to prove the theoretician false to show that their argument isn’t as good as they claim it is.

Not quite. If the theist asserts that the world is consistent with the claimed benevolence of their omnipotent deity, so they do have a burden of proof there. At the end of the day, neither claim for or against the benevolent deity is actually provable, but the theist does end up making bizarre claims justifying their position (like Gordon does), or they acknowledge that they cannot prove their point. The POE does make the notion of a benevolent and omnipotent deity incoherent or incomprehensible.

Gordon wrote to Jason, “just like Mr. Fry, you want to discuss a hypothetical God so far as necessary to prove that he must be Evil or not exist. But you cannot and will not import the full set of presumptions necessary to actually hold a meaningful discussion about the real God.
Your ‘god’ is a straw man who indeed does not exist and who certainly has no connection with reality. So of course your argument works. I have just admitted that he does not exist.

... in otherwords, first we have to agree with Gordon’s religion before we have a conversation about it. Of course, the problem is that unless one is crazy, they can’t swallow all the contradictions and elided-over gaps.

And of course, if we simply agreed with Gordon, a conversation would be purposeless.

Gordon also commented that non-believers such as myself and others have beliefs. I cannot tell whether he means “religious beliefs” or something else, but on another thread I wrote that the difference between belief and knowledge is that knowledge is belief justified by evidence or conclusive logic. To be clear, knowledge is a subset of beliefs. I stand by that. I have beliefs about many things and knowledge about a few things. About whether deities exist, I have nothing.

Gordon wrote that “I still believe that the 3 omnis are true of God. But he chooses to limit two of them ... ” and yet “God cannot stop the suffering of innocent third parties”.

This is foolish. If some God chooses to not exercise His full abilities, then that God is fully capable of stopping “the suffering of innocent third parties”. That God sins by omission.

If some God possesses the “three omnis” and chooses not to exercise them fully, that does not EXCUSE that God, it implicates Him in manifest evil. To be able to prevent suffering and not do so is evil.

sean s.

By sean samis (not verified) on 24 Feb 2015 #permalink

Thinking more about the design argument - we can infer human design because only some things are designed by humans, but isn't everything supposed to be designed by the biblical God? Shouldn't dust show just as much design as a human?, molecules and compounds as much design as dust, atoms as much design as molecules and on up and down the lines? How can one see design if there is nothing to differentiate design from non-design?

By Michael Fugate (not verified) on 24 Feb 2015 #permalink

Another Matt,

“Here’s a very crude analogy that might help”

Analogies usually don’t help very much. If enzyme evolution is a fact, data and experiments would help.
-
“RNA can do similar things as proteins, which means that they could have gradually been replaced as catalysis factors and coding factors by structures that work better for those two functions”

And how did these more functional structures evolve?

===

Michael Fugate,

“Diversionary tactics Phil – when are you going to produce any positive evidence for creation?”

Well, you were quick to announce that evolution can explain things. But the explanations really sound like sappy, unrealistic religious convictions. Have you ever noticed that every incremental developmental detail in your faith calls for something extremely unlikely to happen for no reason at all?
-
“Phil’s problem is that he can’t generalize – he thinks that every protein, every gene in every living thing needs to be explained in detail”

Well not necessarily every single one. But enough to establish a reason to believe that accidents can result in things that are extremely complex and functional, since all biological things are. You’re problem is that you want to generalize before you’ve ever shown any specifics at all.

I don't know where the omni-benevolent idea comes from. The Messiah who personally enabled general atonement is the same one whose anger is a winepress. Benevolence and wrath are both target specific.

Phil,

I don’t know where the omni-benevolent idea comes from.

Man, neither do I! The christian gods are such blatantly malevolent monsters that the very notion is ludicrous!

Well, you were quick to announce that evolution can explain things.

Of course the theory of evolution can - and does - explain things. But, again of course, it does not explain everything. Only religious nuts would demand that it should do so.

And when the theory has no explanation, those religious nuts think they've hit the jackpot! Why, they argue, if the ToE cannot explain, say, the origin of life, gods must have done it! And the entire theory is worthless!

That's the god-of-the-gaps fallacy, Phil. Just because we do not know something does not mean that gods had a part in it. Ignorance does not entail the supernatural.

And to try to refute the ToE because there are things we do not know is like trying to refute the theory of internal combustion because we cannot make a car that runs on water. It's just silly.

The theory of evolution is perhaps the most powerfully explanatory scientific theory ever devised. If you want to refute it, you can't simply point out that there are things it doesn't explain. Instead, you must offer an alternative explanation - a single, clear biological mechanism- that explains the persistence of sickle-cell anemia, the changes of the beaks of the Galapagos finches, those speckled moths in British coal country, the nested hierarchy of all genomes, acquired resistance to pesticides in weeds and to antibiotics in bacteria, trilobite and dinosaur fossils, hominid fossils, vestigial organs, snake and whale legs, Hox genes, why you look so much like your mailman, speciation, blind cave fish, vestigial mouth parts in insects, the recurrent laryngeal nerve in giraffes, the route of the vas deferens, the phylogeny of North American horse fossils, the continental distribution of species, lactose intolerance in humans, and literally - literally - tens of thousands of other facts.

Your god-of-the-gaps arguments, in addition to being logically fallacious, are negligible compared to what the ToE does explain. You and your fellow evolution deniers have your work cut out for you. Stop wasting everybody's time with your stupid gotchas and do something productive!

phhht,

I appreciate your enthusiasm, but I have a limited amount of time, and your arguments and evidence are either juvenile, presumptuous, or antiques.

Thanks for your interest, but I think you are right where you belong.

Phil, no answer about how God did it? I am surprised; you seem so sure, but I guess you were just bluffing.

How many times have we explained evolution and natural selection for you? Do you have amnesia and can't remember any of them? I don't want to intrude, but you might want to see a doctor about memory loss.

By Michael Fugate (not verified) on 24 Feb 2015 #permalink

Phil,

In other words, you got nothing.

I wouldn't talk much about limited arguments if I were you. It's clear you cannot address even a single one of my points.

What, Phil? Only unsupported dismissals of pffft's points on evolution?

By Walt Jones (not verified) on 24 Feb 2015 #permalink

Phil,

You talk a lot about what the theory of evolution can't explain.

How about the bible?

How do you explain the workings of the reanimation of a corpse by magic?

Of course you cannot. You're no better than those Haitian voodoo priests.

And if you can't explain that, why should anyone believe it, Phil? Just because you say so?

Gordon:

When I apply the word “Wall” to a pile of rocks, I am speaking directly to the order I detect in how the rocks are arranged. I am claiming to see a Design. And I thereby import all other presumptions surrounding the Design, such as the need to move the rocks to that location, and to place them one on top of the other, etc. I claim that some man, even though unknown to me, applied his intelligence, his will and his strength to the intentional building of that Wall.

All this is communicated the moment I point to some rocks and say, “That’s a Wall.”

Okay, that's what you mean by "wall." But remind me of your larger point on the matter: I thought you were trying to make a point supporting intelligent design, and it was that larger point I was arguing against. But now you seem to have backed up to making a simple semantic point that when Gorden says "wall," he means a physical wall containing many separate pieces built by intelligent agents. If that was your point, why mention it at all? It seems a bit of a nonsequitur without any sort of deeper argument on your side.

Michael Fugate,

“Phil, no answer about how God did it?”

No, I gave you an answer. Complete, integrated, functional designs; created from nothing, and spoken into existence. No space particle miracles, no DNA replication failure miracles, no natural selection fairies, but all kinds of wonderful chicken/egg nightmares for the suckers to wrestle with.

So how did those replication enzymes appear?

===

phhht,

“It’s clear you cannot address even a single one of my points.”

No, I’m sorry but I can’t. There are lots of venues if you want to be argue, but the best adversary is your own mind. If you’re just collecting horse shit, you’ll never really actually be involved in a fight.
-
“How do you explain the workings of the reanimation of a corpse by magic?”

The things you accept without question make resurrection look like a parlor trick. You’re the product of endless cultural support groups. Start all over…seriously, push the reset button, clear your head, forget what you like and rethink everything in terms of evidential weight. Slap promises off the table…demand evidence.

Phil,

There are lots of venues if you want to be argue...

No thanks, I'd much rather be debate.

But you're right, resurrection looks like a trick. It looks like all the other zombie stories in the bible. It looks like The Walking Dead. It looks like zombie stories were as popular then as they are now.

You have the gall to talk about evidence. There is not the slightest evidence that your zombie god is anything but fiction.
You actually believe in a magical man who creates stuff out of nothing, with nothing but his magical voice?

You're a loony.

Phil,

So how did those replication enzymes appear?

I don't think anybody knows for sure, Phil.

So what? God-of-the-gaps? Do all those things the ToE DOES explain suddenly evaporate?

What's your point?

Not an answer. What words, what language? If someone were there what would one hear and what would one see? How do words form atoms, molecules, cells and the like? Sound waves? I need more.

By Michael Fugate (not verified) on 24 Feb 2015 #permalink

Let me make it easier for you Phil. Pick any one thing mentioned in "creation week" and takes us through the conception, design, and manufacture of that one thing by God. It needs as much detail as any thing made by a human - say a watch like Paley. You can do it, can't you?

By Michael Fugate (not verified) on 24 Feb 2015 #permalink

Gordon--

So let me express my appreciation for the generally polite spirit of the discussions on your blogs. I hold a genuine respect for those posters who have engaged me in lengthy and vigorous exchanges.

I appreciate that. Though it seems to annoy cipher, I am a big believer in allowing people to say pretty much whatever they want, as relentlessly as they want, in response to my posts. If I dish it out I have to be willing to take it.

However, you then rather spoil the effect by writing this:

In this very thread I have shown that not one atheist will take a serious look into how the Bible explains the Problem of Evil. They simply won’t go there, even hypothetically. The train of thought is stopped dead in its tracks by a presumption of already having all the answers.

Really?

I'm the one who spent quite a few years, and spent quite a lot of money, socializing with YEC's, attending dozens of conferences, specifically so they could tell me themselves what they believe and why they believe it. I then wrote a book about my experiences, in which I gave one of the most sympathetic portrayals of YECs ever written by an academic. Todd Wood, a prominent YEC himself, reviewed my book very favorably, specifically praising my fairness in presenting the YEC side.

That's who I am. That's how seriously I take the task of understanding the viewpoints of others, even those who hold views radically different from my own.

You, by contrast, are the one who declares he just knows his take on God is true, and then derides anyone who disagrees as blinkered and ignorant.

So I think I won't be accepting any lectures from you on closed-mindedness.

Though it seems to annoy cipher, I am a big believer in allowing people to say pretty much whatever they want, as relentlessly as they want, in response to my posts.

I didn't say you shouldn't allow their comments. I said I don't understand why people bother engaging them, as they're intractable.

However, that being said, I actually do tend to think it's a mistake to allow right wing (politically and theologically) ideologues to participate on liberal blogs. They don't change, they learn nothing, they're incapable of stepping outside of their self-imposed (and most likely genetically-imposed) boxes, and in general, they derail comment threads and conversations without contributing anything. They are an obstacle to rational discourse.

Yes, cipher, and what's really annoying is when 74 comments later, they claim either that they didn't write or didn't mean something like, I don't know, maybe, "if I had a blog, I’d ban them outright."

By Walt Jones (not verified) on 25 Feb 2015 #permalink

Cipher:

I said I don’t understand why people bother engaging them, as they’re intractable.

Well, IMO there are several good reasons to do so. I'll mention two. First, its always possible I could be blinkered or biased on some subject and so reasoning incorrectly. Discussing my beliefs or reasoning with people who don't accept them is far more likely to turn over an error than discussing these things with people who share my beliefs and reasoning. If you've got a math problem and you don't know whether you did it right, you'll likely learn far more by comparing your solution to someone else's different solution. Comparing your solution to someone who did exactly the same thing isn't going to tell you much about whether you did it right or not, is it? Now, do I think I'm wrong and the creationists are right? Obviously not. Am I certain every argument I make is sound or at least valid? No.

Secondly: lurkers. In many cases a discussion between two intractible people can still serve to help lurking tractable people change their minds or refine their ideas.

cipher,

Funny, it used to be that we believed "I don't agree with what you're saying, but I'll defend to the death your right to say it." Far too often now it's turned into "I don't agree with what you're saying, so shut up!" I personally am thankful to Jason that, at least in this small corner of the internet, the former is more the rule than the latter.

Yes, cipher, and what’s really annoying is when 74 comments later, they claim either that they didn’t write or didn’t mean something like, I don’t know, maybe, “if I had a blog, I’d ban them outright.”

Yes, and if I had a blog, that's what I'd do.

Eric, if you make a mistake, these are not the people who'll be able to detect it. These are the people who think the reasoning process works this way: "Here's the conclusion; what evidence can I find to support it?" Of course, in an ironic turnabout, that is what they are constantly accusing liberals and atheists of doing. Churches and seminaries apparently don't teach the concept of projection.

Sean: “I don’t agree with what you’re saying, so shut up!”

It's more along the lines of, "You have nothing of value to say, so go say it somewhere else."

Phil is an excellent illustration of what I'm talking about:

The things you accept without question make resurrection look like a parlor trick. You’re the product of endless cultural support groups. Start all over…seriously, push the reset button, clear your head, forget what you like and rethink everything in terms of evidential weight. Slap promises off the table…demand evidence.

Apparently, they don't have mirrors in those churches, either.

To my comment at 181, I'll add equivocate. Thanks, cipher, for demonstrating my omission.

By Walt Jones (not verified) on 25 Feb 2015 #permalink

eric,

VS: sounds like we agree on the basic result of a theodicy-style analysis: the Christian conception of God is apparently not benevolent. There may be some way in which all these apparent malevolent actions are truly benevolent, but if so we don’t currently understand that way. Fair enough?

This is phrased a bit oddly, making it look like you're trying to sneak in some commitments that I don't agree with:

1) You talk about "malevolent actions", but there are more choices than benevolent or malevolent. Apathetic, for example. So I don't want to agree to something that implies that God's actions look malevolent, since they clearly DON'T look malevolent. If there's anything we can say at that level it's that God doesn't have eliminating our suffering as a main goal, which it might seem is required for benevolence.

2) I don't agree that the result of any theodicy-style analysis is that, because I think that theodicy is the philosophical examination of the issue, and that when we look at it philosophically then it isn't apparent that God is not benevolent, at least in the right way. I only concede it for the first blush examination, not the detailed one.

To make this clearer, let me give you some examples. Let's take the old "Can God create a rock that even God can't lift?" question. At first glance, this seems like a problem for omnipotence. But when we look at it philosophically, we can see that it ends up requiring God to do at least one thing that's logically impossible. Either God can create a rock that it is logically impossible to lift -- at which point He couldn't lift it -- or it is logically impossible to create a rock that can't be lifted. Either way, God ends up having to do the logically impossible, and a much more sensible view of omnipotence is that it can't do the logically impossible. So no real issue there.

To take a less religious example, let's take the "If humans evolved from apes, how come there are still apes?" question. At a shallow level, this looks like a real conundrum, but once you go into more detail on what evolution actually you can clearly see that this is an oversimplification, and that in the context of evolution it's not really a challenge. So on deeper reflection we can see why this isn't an issue, even though it looks like one at first glance.

I'm not saying that when we look deeper the problem goes away completely. It doesn't. But when we look deeper, we can see that things are a lot more complicated than they appear at first glance, for a number of reasons:

The problem was obvious when you could talk about evil as a real thing in the world. But that's not philosophically tenable any more. So the Problem of Evil translates to the Problem of Suffering. This means that you have to consider it morally wrong to not prevent the suffering of others if you can. But there are a lot of cases where we reject that. Few people think that someone is morally obligated to forgo going to the movies instead of giving that money to a homeless person to buy a meal. Peter Singer's big push is to argue for us to take that idea on MORE, but even THEN it's not that stringent. So it isn't clear that you are morally obligated to relieve someone else's suffering just because you can.

There's also the issue of how God is differently situated than we are, which leads to the point I keep pushing at you and you keep side-stepping. Even if we concede that we are morally obligated to try to eliminate diseases like, say, smallpox, we are not in a position to be able to eliminate all of them any time soon. God, on the other hand, would be. So if we argue that God is morally obligated to eliminate smallpox, how do we then avoid concluding that God is not also morally obligated to eliminate measles and the common cold? If we argue that God is morally obligated to eliminate bone cancer and childhood leukemia, how do we then avoid concluding that God is not also morally obligated to eliminate leukemia, cancer ... and ingrown toenails? Ultimately, it's hard to see how these arguments avoid turning into arguments that this world should not have any suffering at all, but that's not a tenable position in any Christian religion because of the explicit push that this world WILL have suffering, and it's only heaven that won't.

Even IF we can avoid that, we also have reason to think that one is not morally obligated to try to eliminate those diseases if one can, because someone going into medical research is not morally obligated to research cancer as opposed to, say, that disease where people sneeze when they go out into the sun, and someone can give money to the latter and not the former without being considered to be acting immorally.

Finally, to tie into something that I think Gordon was talking about, Goodness does not require God to be nice. If God has a reason for us to have to struggle in this world, even if He could achieve that end with less suffering, that doesn't mean that God has to do it that way. To tie back to what I thought the explicit comment was, God can indeed decide that Justice trumps Mercy without impacting his overall Goodness. So that tangle needs to be unwound before we can really settle anything here.

Essentially, to make this argument work we need to know that God is acting immorally, and that requires us to know what is and isn't moral. It turns out that the concepts of moral and benevolent are more complex than they look at first glance.

But my point was about entry requirements, and AFAIK no major Christian sect agrees with your entry requirements.

We've lost the thread again ...

This started from you insisting that there was an issue with people having free will in heaven and yet not having there have to be evil. My argument was that people were still free to do bad things in heaven but that once there they wouldn't. When you asked how that happened, I pointed out that the people who get into heaven are people who won't do bad things. You argued that that doesn't seem true theologically. At the end, though, what is true is that people in heaven will not do bad things and that there is indeed a criteria for entry. This ranges from they are moral people -- ie they won't deliberately act immorally -- to they are people who won't act immorally without the temptations of the flesh to they are people who believe in God and so won't deliberately disobey his commands to they are people who believe in God and/or are moral and will be buttressed by God after having made the free choice to strive to meet the entrance criteria to their free will will be taken away because it's no longer needed once they've been judged. All of these mean that in heaven there will be no need for more tests or growth, because the selection is already done, to whatever criteria you need.

Which leads to my question, which is this:

Do you think that there is a Christian sect that says that people in heaven, if given free will, would act evilly that doesn't ALSO insist that they won't have free will in heaven or that they never had free will, and considers not having free will in heaven to be an issue?

By Verbose Stoic (not verified) on 25 Feb 2015 #permalink

Thanks, Gordon, for the G.K. Chesterton.

Here’s another from GKC (paraphrase):
‘Christianity has not been tried and found wanting. It has been found difficult and left untried.’

By See Noevo (not verified) on 25 Feb 2015 #permalink

Thanks, Gordon, for the G.K. Chesterton.

Here’s another from GKC (paraphrase):

‘Christianity has not been tried and found wanting. It has been found difficult and left untried.’

By See Noevo (not verified) on 25 Feb 2015 #permalink

To my comment at 181, I’ll add equivocate. Thanks, cipher, for demonstrating my omission.

No, Walt. It really isn't that difficult. I know you have reading comprehension skills, so I have to assume you're just being stubborn.

That is why I complimented you for recognizing the good that comes from exchanging opposing views. I have definitely benefited from it, but it appears to me that most of you have not.

I have benefitted by realizing that those who claim to know the most about gods, know absolutely nothing about them. The best they can do are tropes like "God's ways and thoughts are not our ways and thoughts." Which is equivalent to saying I know nothing. You demand infinite detail from scientists and yet nothing from creationists, theists and even your God.

Isn't it a bit bizarre to spend so much time trying to refute evolution and no time trying to find out how, if your God really did it, it was done? Couldn't your God resolve the whole issue by being more forthcoming? Why give us a little bit and claim no more can be given? Why not leave the whole thing as a mystery?

I also have benefitted in that it is clear if you and phil are representative, that engineering programs are very poor at transmitting how science is done. Anecdotes are at best single data points with very little value for determining cause and effect. You claim that a God made you happier, but of course, that is an opinion based on a sample size of 1. What you would want is a independent study of people living with and without and some way to measure happiness - easier said than done. I am much happier without religion than I ever was with it - and don't try to claim that I wasn't really religious; you don't know anything about it. One v. one - who wins?

By Michael Fugate (not verified) on 25 Feb 2015 #permalink

Eric,

You have to keep in mind that the Bible was not written to a culture steeped in the scientific method as ours is. It was written to a people who probably saw God hiding behind every bush.

So in describing the generational consequences of Sin, it made sense for Moses to personify it in terms of God's punishment or wrath. Those people understood things this way. Really, we are probably only a century or two away from a time when virtually everyone still thought this way.

So all I am attempting to do is read what I see the text saying, but to translate it in terms relevant to a scientific society. Thus, where you see God deliberately intervening to make sure the innocent are punished (which is how the text reads, and to us does sound malevolent), I step back and see that the negative consequences being spoken of are going to effect the later generations in any event. In fact, it is probably only God's intervention that would stop this.

It is no different than if I called your car "jealous" because it only liked gasoline. You could say it was a very unreasonable or even malevolent car, for refusing to get you to work when you only had water available to fill the gas tank. Or you could realize that there is in fact only one way to make the car work. All other liquids are guaranteed to give you a bad result.

The fundamental truth is the same. The most effective way to communicate it to a people, can change. You see this change even between the Old and New Testaments. Christians often fail to understand this as well, and then come to the preposterous conclusion that God changed. "In the OT he was a God of Wrath, but in the NT he is a God of Love."

"Worshiping the Christian God" does not fully communicate what is required. What must happen is obedience to his commandments, and not mere lip service. Even Jesus was quite clear in making this distinction. Most Christians today have not gotten beyond the lip service, so the blessings simply do not apply to them, and the curses still do. Find any Christians living out Acts chapter 4, and you will see a different story entirely. I know, they're hard to come by.

Phil wrote to phhht, “your arguments and evidence are either juvenile, presumptuous, or antiques.

phhht, take heart. Notice that Phil did not say they were wrong.

Cipher wrote, “I didn’t say you shouldn’t allow their comments. I said I don’t understand why people bother engaging them, as they’re intractable.

We should never expect to change minds set in stone, the point is to try, then when we meet someone whose mind is not so set, we’ll know what to say.

I actually do tend to think it’s a mistake to allow right wing (politically and theologically) ideologues to participate on liberal blogs.

That’s an error for two reasons. FIRST, We all need to escape from our own echo chambers. We need to hear opposing ideas no matter how foolish. That’s essential to keeping an open mind. SECOND, as already mentioned, trying to convince others (even the intractably foolish) is a good way to discover how to present and defend the truth.

As eric this is also the process where our own blinkers and deficiencies are best found and fixed. You responded that these others will never catch a meaningful error, but my experience is different. They may be intractably foolish, but that does not make them stupid.

Banning the Other from a blog just creates an echo chamber, a temple dedicated to confirmation bias.

And if you think someone in the conversation is contributing nothing of value, ignoring them is easy.

Sean T wrote, “I personally am thankful to Jason that, at least in this small corner of the internet, the former is more the rule than the latter.

I second that.

Gordon wrote “In this very thread I have shown that not one atheist will take a serious look into how the Bible explains the Problem of Evil. They simply won’t go there, even hypothetically. The train of thought is stopped dead in its tracks by a presumption of already having all the answers.

We don’t take the biblical explanation for the POE seriously because, after giving it serious consideration, it does not add up. I have myself posted several objections to it, to which your responses can be summarized as “but the Bible (or Chesterton) says ...”.

What atheists want is certain very limited types of evidence.

I am not an atheist, so I won’t presume to speak for them, but all kinds of evidence can be considered by science. However, not all evidence has equal weight. Biblical evidence is valid, but has little to no weight because we don’ know it’s provenance and cannot replicate it’s claims or resolve internal inconsistencies.

In short, there is no “evidence” that will satisfy you guys.

Sure there is, but it’s evidence YOU DON’T HAVE.

what you can also see is that my God did in fact provide me the knowledge of what would convince you.

Um, no. If your God did that, you would convince us. No, that is exactly what you don’t have, and why you don’t convince us. We cannot see what your God may have given you, we weren’t there when this giving supposedly happened. If your God actually gave you the knowledge of what would convince us, then you’ve yet to use that knowledge.

You appear to believe you have been called to be a prophet. Matt 22:14.

sean s.

By sean samis (not verified) on 25 Feb 2015 #permalink

So I don’t want to agree to something that implies that God’s actions look malevolent, since they clearly DON’T look malevolent.

Threatening to punish someone's innocent grandkid if that person believes in another God, "because I am jealous" seems pretty malevolent to me (Ex 20:5). That is not indifference to suffering, that's actively propagating it. Or don't you agree?

Re: 2. The problem with comparing theodicy to logical paradoxes is that, at some point, you must either concede that some evil or suffering is not a logical paradox, its contingent and therefore not necessary, or concede that God and the world are both locked-in on every single act because any other option is just a logical paradox human's can't see. The first is the classic theodicy problem: if you concede it, your comparison gives no defense of theism. It adds nothing. But the second is, AFAIK, roundly rejected by theists because it eliminates meaningful free will. Moreover, it sure seems to be the sort of surprising and remarkable claim that would bear the burden of proof; "there are no philosophically contingent events, ever" is not something I think one can insist others must disprove or it stands; its instead the sort of claim that requires positive evidence before anyone should accept it.

So the Problem of Evil translates to the Problem of Suffering.

Only in part. It also includes asking why God created humans the way we are and continuously allows us to be the way we are, when it certainly seems to be the case that many of us could be more peaceful, empathetic creatures without violating our free will. Just make the more violent of us have the free-will-including predispositions of the least violent of us.

So if we argue that God is morally obligated to eliminate smallpox, how do we then avoid concluding that God is not also morally obligated to eliminate measles and the common cold? If we argue that God is morally obligated to eliminate bone cancer and childhood leukemia, how do we then avoid concluding that God is not also morally obligated to eliminate leukemia, cancer … and ingrown toenails? Ultimately, it’s hard to see how these arguments avoid turning into arguments that this world should not have any suffering at all, but that’s not a tenable position in any Christian religion because of the explicit push that this world WILL have suffering, and it’s only heaven that won’t.

You're right in your first part that a suffering-filled world is not consistent with the tri-omni God posited, and you're right in your second part in pointing out that Christians must premise or axiomatically declare the world must include suffering to make any sense out of it. Where I think you go wrong is in seeing the latter as a "legal move" in reasoning rather than being a form of post-hoc apologetics. It is not good reasoning to say that since the world includes suffering, the concept of a benevolent omnipotent etc. god must somehow be consistent with a world that includes suffering. There is no "must somehow," that's an apologetic assertion or presumption.

This started from you insisting that there was an issue with people having free will in heaven and yet not having there have to be evil. My argument was that people were still free to do bad things in heaven but that once there they wouldn’t. When you asked how that happened, I pointed out that the people who get into heaven are people who won’t do bad things. You argued that that doesn’t seem true theologically. At the end, though, what is true is that people in heaven will not do bad things and that there is indeed a criteria for entry.

The first part of your last sentence is pure assertion on your part, with no reasoning or logic or evidence behind it. This is again a form of apologetics. No, it is not shown to be true that 'people in heaven don't do bad things.' What is shown is that the three ideas (1) Christian entry criteria (2) Christian insistence of free will in heaven, and (3) Christian insistence that bad acts don't occur there produces a paradox. They don't rationally seem to all be able to be true at the same time. While perhaps not strictly logically contradictory, they are at best underpants gnomes logic, missing a crucial step: you cannot derive 3 from 1 and 2. Instead, 3 seems to be taken as a premise, from which they work back to try and find some argument that will justify it. Which is, again, apologetics.

Do you think that there is a Christian sect that says that people in heaven, if given free will, would act evilly that doesn’t ALSO insist that they won’t have free will in heaven or that they never had free will, and considers not having free will in heaven to be an issue?

I'm again having trouble parsing your statement, but I think my answer is: I don't know of any sect that posits the first part of your 'also' so your question is moot. AFAIK the standard christian doctrine across many sects, probably unstated but here's my most educated guess, is that everyone has free will in heaven and yet there is no evil performed there.

Sorry cipher, I was assuming that the suppressed premise in your statement was that because you would ban people for these types of comments, Jason should consider doing that, too.

If you're saying that you were just telling us your feelings, and not trying to convince anyone of your position, I believe you - and wonder why you think anyone cares what you think.

@Gordon:
What makes you think I hadn't obeyed God's commands for 10 years? Or 20? Or 30? Well, not all of them. I didn't stone anybody, and I've met quite a few Wiccans whom I've suffered to live, and I've even let my wife instruct me!

I know plenty about Chesterton, and I'm more interested in finding some evidence than your argument from authority about my motives. I'm open to a god creating the original life, and the only reason I lean toward other hypotheses is that they include only things that I know exist (such as chemicals and energy).

By Walt Jones (not verified) on 25 Feb 2015 #permalink

In reply to by eric (not verified)

Another Matt @ #161,

Funny that the Protestants do not see this problem, as they deny Purgatory altogether. So I guess only only the Catholics and Eastern Orthodox will ultimately make it to Heaven, and the Protestants will wait forever on the platform for a train that never arrives.

The question of perfection in this life is a discussion for another day, but there is plenty of support for the notion in the New Testament. The present theology, with or without Purgatory, basically has us wasting our time here on planet Earth, with nothing to learn and no possibility of improvement. What was God thinking? Or have we just been ignoring his clear commands?

"The right kind of Christian" assumes that I am here to convert people over to my pet set of dogmatics. While that is indeed the motivation of most missionaries, I came with a very different mindset. First, the people here are predominantly Catholics. I converted to Catholicism here. (And not because of Purgatory!)

Second, the thinking I am trying to change is really basic stuff like, "Thou shalt not steal." They cannot see how ignoring this command hurts them, and how teaching stealing to their children extends the poverty down through the generations. Stealing, which is against God's commands in every stripe of Christianity on the planet, is one of the big things keeping these folks in poverty.

It is fairly universal the people don't like to change, no matter how bad their lives are. This cuts across cultures and educational levels, etc. No one from my own American Christian culture wants to be told by me that they are failing to obey the commands of Jesus either. If I showed up in most churches preaching what was in that article you linked to, I would be shown the door before the first hymn was finished.

Gordon wrote, “So all I am attempting to do is read what I see the text saying, but to translate it in terms relevant to a scientific society. Thus, where you see God deliberately intervening to make sure the innocent are punished (which is how the text reads, and to us does sound malevolent), I step back and see that the negative consequences being spoken of are going to effect the later generations in any event. In fact, it is probably only God’s intervention that would stop this.

... in other words: he is selectively reinterpreting the Bible to suit his needs. If so, then he cannot complain if others do likewise, even if they come to manifestly unlikable conclusions. Fair for one is fair for all.

What must happen is obedience to his commandments, and not mere lip service. Even Jesus was quite clear in making this distinction.

That’s fine, but if one is supposed to testify to God’s goodness, then to avoid mere lip service, that word “good” needs to mean something; it needs to reflect the application of a moral system

However, Gordon has repeatedly asserted that his God exists and operates entirely outside all moral systems. So calling his God “good” or attributing any moral virtues CANNOT BE OTHER THAN MERE LIP SERVICE. If we cannot judge his God then any conclusion about Him is MERE LIP SERVICE.

True Christians “are hard to come by.

In deed.

sean s.

By sean samis (not verified) on 25 Feb 2015 #permalink

Eric @ #173,

The larger point was that we all recognize Design when we see it. We see things arranged, be they complex like a watch or simple like some rocks stacked on top of each other, and we instantly know that the intervention of some intelligent being was required in order to give that result.

So we can look at something like our own hand, and recognize Design. That is the "evidence" of the Theist that there is a higher intelligence behind Life itself.

Now Evolution has provided a story whereby all complex organisms were not designed, but rather evolved from simpler organisms, thus, in Jason's view, nullifying this proof of the Theist. But if you go back to the simplest living organism, and put it under a microscope, you once again see something so complex that it still speaks of Design.

So the Design argument is not dead at all. It is just glossed over, and then deflected with, "Just give us another 300 years to prove abiogenesis, and then your Design argument will have finally been disproven."

Whether this faith in the ability of science to create Life proves true or not, today it is very far from being proven. So the announcements of the death of Intelligent Design are premature.

And don't forget the other elephant in the room, carefully being ignored at present, is intelligence itself. When you can train a monkey to write poetry, I will personally write the article announcing the demise of Intelligent Design.

You had a humorous point @ #154 that I did not feel needed a response.

And unlike most Christian looking forward to the afterlife, I know that the whole point of the exercise is about making friends in THIS life. Focus on that, and everything else takes care of itself.

Correction to #199: I assumed cipher's suppressed premise was, "because I do something, other people should do it, too." What I wrote was actually my inferred conclusion.

By Walt Jones (not verified) on 25 Feb 2015 #permalink

In reply to by Gordon (not verified)

Michael @ #177,

Right now, and every day, your life is controlled by words.

You just are not aware of it.

Were you aware of it, you would see something profound in the statement, "In the beginning was the Word", even if you did not see just how deep it goes.

But don't worry, most Christians read right over that statement as well, and haven't a clue about what they've just read. I'm speaking from experience of some 30 years of reading or hearing that statement on a regular basis, and not going past the surface meaning of it. And no one else told me it went any deeper either.

If you want some clues, do some reading about what the Jews say about the Torah. Every name has a meaning. Every letter has a numeric value. Every verse can be interpreted at four different levels. It is mind boggling how much they pull from the first few chapters in Genesis alone that is simply lost once you translate it to English.

If you want to know how the world runs, study the Words.

Gordon wrote, “we all recognize Design when we see it. We see things arranged, be they complex like a watch or simple like some rocks stacked on top of each other, and we instantly know that the intervention of some intelligent being was required in order to give that result.

That’s not knowledge. That’s intuition, and an unreliable basis for conclusion. Look at the Giant’s Causeway sometime; it’s in northern Ireland.

we can look at something like our own hand, and recognize Design.

... but we cannot know that it was purposefully designed. That verb (“designed”) may only be metaphoric. Gordon confuses two things: useful for some things is entirely different from intentionally designed for some things. The first does not imply the second. Found objects can be useful.

the Design argument is not dead at all.

... because even bad ideas don’t die, ideas like the flatness of the Earth or the moral superiority of some races or genders or creationism. Ideas can’t die, they never lived.

When you can train a monkey to write poetry, I will personally write the article announcing the demise of Intelligent

He wouldn’t; because that would not suffice. It would not actually be meaningful, and Creationists would only point out that an intelligence trained the monkey, so an intelligence must have “trained” us.

I am especially sure of this because Gordon wrote in the past, “My faith is independent of what Science does or does not find. I don’t require a certain result in order to ‘prove’ me correct.”

I literally laughed out loud when I read this. So even when the result proves him wrong he still thinks he’s right.

It is well known that to be PROVEN correct REQUIRES particular results. Whether a theist or not, that rule holds. No reasonable person’s opinions are invulnerable to facts.

Gordon advised, “If you want to know how the world runs, study the Words.

That’s bad advice. If you want to know how the world runs, observe how the world runs.

sean s.

By sean samis (not verified) on 25 Feb 2015 #permalink

Jason @ #179,

Yes, of course I believe that my take on God is true. Because I can back it up from many angles, I am a bit more certain of myself than most Christians. Perhaps you expected all Christians to be bamboozled by the Problem of Evil?

It is a favorite tactic of many atheists on other forums to simply accuse anyone who disagrees with them of being ignorant. They do this as their initial and only retort. I have not done that to anyone here, because I try to engage in discussion, not shut it down.

There have been a few times when someone has demonstrated obvious ignorance about the Bible, and I have called them on it. If there is someone here I have derided who you think did not deserve it, I would be interested to know who.

There seems to be a general notion that I expect everyone to agree with me. I do not expect that at all. I am hopeful that some will be willing to follow the logic and see where it leads. If they want to use the Bible to question the morality of the God of the Bible, then they need to take the Bible as a whole, and not just a few selected out-takes.

Unlike previous threads where the issue came up, it was your choice to make this one specifically on the Problem of Evil. So I was frankly a bit miffed when no one, including the host, seemed interested in exploring the possibility that the Bible does indeed provide a coherent answer.

Atheists are forever accusing Christians of being closed-minded, and not without good cause, I admit. It just turns out, possibly to your surprise, that Christians do not have a monopoly on this trait.

Walt,

The large part of my initial comment in this thread was about the motivation and probable genetic preconditioning of conservative Christians. I also expressed an inability to understand why intelligent, thoughtful people would engage them in these fruitless arguments. My statement that I would ban them (which I meant) was to emphasize this. I was neither telling Jason what to do nor expressing an opinion apropos of nothing (although I've no doubt you'll disagree).

Here's another opinion I don't think is apropos of nothing: you're a pompous ass.

Go ahead and have the last word. I'm sure you need to.

Cipher @ #185,

We understand the concept of projection, but I was probably not taught it during a Sunday sermon.

Why do you think I pointed Phhht to a chapter on insanity?

He couldn't stop saying, "delusional". Of course, he was applying it to me. Projection.

Thanks for the clarification, cipher, I read your statements about Christians as support for banning those commenters. I was wrong.

I, and others, have already explained the value of engaging them, so I assume you understand our position, even if you don't agree.

By Walt Jones (not verified) on 25 Feb 2015 #permalink

In reply to by Gordon (not verified)

Gordon:

The larger point was that we all recognize Design when we see it.

We obviously don't, or archaeologists wouldn't need training. You could just grab a random person off the street, ask them "hand axe, or stone?" and be assured of getting the right answer. If we could do as you claim, then (to use Sean's example), humans would not have made up myths to explain things like the Giant's Causeway. Or lightning strokes, floods, plagues, etc... If we could recognize design when we see it, nobody would have been burned at the stake for being a witch.

Witch-burnings (and these other things) are a result of false-positives in our pattern recognition: we see a pattern or intelligent cause that isn't actually there.

Because we know humans do this, we cannot rely on intuition to tell us when something is designed and when it isn't.

if you go back to the simplest living organism, and put it under a microscope, you once again see something so complex that it still speaks of Design.

There is a common misunderstanding that (the TOE asserts) the smallest and simplest single-celled organisms today represent some ancient form. That is incorrect. The smallest and simplest single-celled organisms today are just as much the result of billions of years of evolution as humans or any other organism. The TOE does not assert or assume they resemble the replicators that existed at the dawn of life. So yeah, prokaryotes of 2015 are complex. Why wouldn't they be? They descended with modification too. Hopefully we can draw some insight from them, but expecting them to be simple replicators that could arise from nonliving organic reactions is an idea that belongs more in Edgar Rice Burroughs' The Land that Time Forgot novel than in any serious discussion of the theory of evolution.

So the announcements of the death of Intelligent Design are premature.

As a scientific hypothesis, announcements of its birth are premature. ID proponents have not given enough hypothesis details on the when, where, how etc... that the idea can be tested. Dembski has outright said he has no need of a mechanistic theory - basically, declaring that ID isn't even going to bother to try and be a scientific hypothesis. Its PR for biblical creationism, nothing more. Always has been. That is why the DI's textbook used the same definition for both creationism and design and why they felt a simple word substitution of design for creation would be appropriate.

See Noevo @ #190,

That is my favorite GKC quote.

Not enough Christians are aware that the arguments they are having today, with atheists and others, are over ideas and issues raised over a century ago. G K Chesterton answered most of those issues decisively from a Christian perspective, yet we seem to think we have to reinvent the wheel all over again.

At least 50 of his books are now in e-book format. I've just finished Heretics, and am now reading Orthodoxy. Mind-altering stuff!

Gordon:

However, obeying God’s commands for say, 10 years, and looking at the results that has on your own life would be unthinkable.

This is a joke, right? Almost every atheist I know comes from a Christian background where they spent 20+ years trying to live "Christianly" or "biblically." To a person – and this expresses my own experience as well – they describe escaping from Christianity like waking up from a nasty nightmare and feeling the sheer relief of it being over. These are people from several denominational backgrounds, and each one of us has been told by Christians from many denominational backgrounds that we were just doing it wrong, that we weren't really Christian, that if we would just adhere to true Christianity surely God would reveal himself and change our hearts. I personally went through three church approaches (fundie, Mainline, Eastern Orthodox), fervently seeking, and found nothing – actually in all the cases I found some of the commandments beneficial and even vital, while others seemed to be designed to hurt other people. I don't know if you've ever felt this way, but it's morally paralyzing – it makes your instincts to be kind and decent seem like temptations.

And aside from the moral quandary it is simply the case that there are many of us for whom "spiritual feeling" is totally absent. I'm willing to believe that I'm missing some kind of "God antenna" (though why God would build anyone that way and then blame them for it is beyond my comprehension), but it just can't be the case that the Bible is a universally applicable instruction manual for life. Humanity is vastly larger than the sum of what is found in its pages, and you'd understand this if you stopped to actually listen to people instead of presuming to tell them what you think your god wants from them. And then this:

Funny that the Protestants do not see this problem, as they deny Purgatory altogether. So I guess only only the Catholics and Eastern Orthodox will ultimately make it to Heaven, and the Protestants will wait forever on the platform for a train that never arrives.

This succinctly expresses so much of what eventually drove me away completely. First is the dread certainty that people have about their religious beliefs and about who will and who won't spend eternity in conscious torment; and the nerve it takes to implore atheists to be more "humble" – we aren't the ones claiming that the creator of the universe has given them the authority to speak on its behalf in telling others how to behave (and in some cases to enforce it). Second is the fact that no ecumenism has been or ever will be capable of resolving the utter fragmentation of doctrine and praxis that exists in the Christian world. It is manifestly apparent that as a group you have no reliable method whereby to resolve disagreements, especially in the last four hundred years, and this is just within Christianity itself – the problem is obviously starker when you consider the vast differences among world religions. Third is this damned obsession with "getting into Heaven" as a goal. How self-centered. It's turned into a kind of capital that Christians hoard – "I've got my reward, and you don't get any if you fail to act like I tell you" – and is so damaging when that kind of mindset gains worldly power. Huck Finn says "All right then, I'll go to Hell" because getting into Heaven mattered less than the wellbeing of this other human. In that moment his turn away from the commandments of the True Christianity of his time and place and reasoning ethically and empathetically was the only thing that could save him from being a monster. To hell with "getting into Heaven." Burn it with fire.

By Another Matt (not verified) on 25 Feb 2015 #permalink

Michael @ #193,

"No one can show you what the Matrix is. Unfortunately, you have to see it for yourself."

We have a book full of clues, and people at various stages of walking the path, but in the end you have to decide to follow it yourself. I can tell you where it has led me. That is all.

I do not doubt that you are happier without religion. I have little time for religion. I live a life of Faith in my Creator that has almost no connection with religion at all. I go to church occasionally, but I also know that my Creator is not keeping score. If there were more people at the church who saw in the Bible what I do, I would spend more time there. Otherwise, I am under no compulsion to go to church, or to attend to any other religious rituals.

Sean S @ #195,

What evidence would satisfy you?

And you missed God's answer. Jesus said that nothing will convince you. That IS the answer, just not the answer you were expecting.

God does indeed work in mysterious ways. ;-)

Gordon,


He couldn’t stop saying, “delusional”. Of course, he was applying it to me.

There is a simple refutation to the charge of delusion, but that's not it.

Walt Jones @ #199,

Of course I'm only surmising from what you have not said. Perhaps you did sell all you had, gave the money to the poor and have been following Jesus for the past 10 years or more with no discernible positive result. But I doubt it.

The part I doubt is the "no discernible positive result", because it has taken me far less than ten years to see positive results every time I learned how to follow the commands of Jesus more closely.

Incidentally, for reasons that I won't detail here, those are not the 613 commands found in the Law of Moses, so only the instructing from your wife might be leading you off the straight and narrow.

Gordon wrote, “I was frankly a bit miffed when no one ... seemed interested in exploring the possibility that the Bible does indeed provide a coherent answer [to the POE]”

There are some of us (me included) who are still willing to explore the possibility that the Bible has a coherent answer to the POE, but so far as I can see, Gordon thinks the Bible’s “coherent” answer is that God exists and operates outside of all moral systems, and we humans are not permitted to question God’s actions. Perhaps I’m mistaken, but that seems to be Gordon’s take on the biblical answer.

In other words: the biblical answer to the POE is “don’t ask.” Although this answer may be in and of itself coherent; within the larger context of a truth-seeking life, it is not. Unsurprisingly, many of us find that answer unacceptable.

If I’m wrong about his position, I’m sure Gordon will answer.

Gordon wrote, “It just turns out, ... that Christians do not have a monopoly on [close-mindedness]”

This is true, and no surprise.

Then Gordon asked me, “What evidence would satisfy you?

Wait a minute. You claimed your God told you what would satisfy us. If you have to ask me, then your God has not told you what I (or we) need.

So that claim was a lie, was it not?

Or does your God need my help? Does He need me to give you hints?

If your God is real, and if your God hears prayers, then your God heard my prayers. He knows what I need, and He’s clearly not told you.

sean s.

By sean samis (not verified) on 25 Feb 2015 #permalink

Gordon,
First, how did you determine which parts of the bible are God's commands? Are they the ones that make sense in the context of your life experience?

Second, if we're going to argue by dead author, how about concisely answering Christopher Hitchen's challenge:

Name one ethical statement made, or one ethical action performed, by a believer that could not have been uttered or done by a nonbeliever.

By Walt Jones (not verified) on 25 Feb 2015 #permalink

In reply to by sean samis (not verified)

Another Matt @ #209,

No, I was not joking, but I also was obviously not clear enough.

But you are really going to think I'm joking when I tell you this:

I absolutely understand you AND agree with you!

I consider that the modern church, across all denominations, has failed us. You cite many good examples, but the kicker is this one taught by Jesus:

"By this will all men know that you are my disciples, if you love one another."

How can someone found yet another denomination and be true to this teaching of Jesus? How can one church condemn other churches? Yet they do it all the time!

So I do not blame you or other atheists at all. The church has indeed failed you, just as it failed me.

The only difference I suppose is that the church didn't completely fail me. I learned certain valuable truths that transcended all the mistakes I saw around me. The more I saw the church deviating from the Bible, the deeper I dug into the Bible for answers. Eventually I left the church altogether. But because I did find answers in the Bible, I never lost my Faith.

From what you write it is obvious that you came away understanding what really matters. "Love your neighbor as yourself" and "Do unto others as you would have them do unto you" are both encapsulated in your telling of the Huck Finn story.

There really isn't much else to know. But there are some challenges to putting this fully into practice in our lives, and a better understanding of the teachings of Jesus will enable us to overcome those challenges. Unfortunately, you will not hear those teachings taught in most churches.

Once you understand that it really is very simple (it is the churches who make it complex), you can then see that all the necessary information can indeed be found in the Bible. In fact, Genesis chapter 3 and Matthew 5, 6 & 7 tell us 95% of all that we need to know. One chapter on the problem and 3 on how to fix it.

And no, you are not defective, and missing some "God antenna". I have had a wide variety of spiritual experiences. Most of them did not include warm, fuzzy feelings or the like, although some did. But my connection to the spiritual is not at all dependent upon feelings. While for most people it is, I do not believe that is genuine spirituality.

So to clarify my original point, it is specifically the teachings of Jesus, and not the doctrines of any church, that will change your life positively. Those are the teachings that have been found difficult, and left untried.

Matt, I want to thank you for your forthrightness here. I think you have answered the issue of "evidence". You have listened to the claims of the churches, and diligently so, and yet not seen the results that they promised. That is the evidence you are working off of. You have tested their claim, and it has been falsified. So you ditched the hypothesis, which is the perfectly reasonable thing to do.

In light of that strong evidence, it is going to take a whole lot more than some theoretical argument to get you to change your mind again.

You've really got me thinking now.

Gordon,
You still haven't explained how words cause these effects you claim. How do they work? You do understand that assertions are not explanations? Of course words having meanings - talk about obvious - but do do need to know the language employed. I do enjoy the way you change the meanings of words to suit your purposes - handy thing for winning arguments - but not for engaging in meaningful discussion.

By Michael Fugate (not verified) on 25 Feb 2015 #permalink

Sean S @ #215,

Well Sean, now you've gone and dug yourself a really big hole. But don't worry, I'm not the type who would bury you in it.

If you read Another Matt's post @ #209, and my response @ #217, you will see that the "evidence" issue has now been answered definitively.

Matt has confirmed exactly WHY there is no evidence that will convince most of you.

So God has indeed heard my cry, and answered me, even before you wrote your little taunt to challenge me.

Now there is only one way for you to prove me wrong here, and that is to AGREE that God has trumped you on this one!

Because then you would be saying that this evidence of God helping me was good enough to convince you.

But I'm sure that my original answer will stand: Nothing will convince you. Your reply confirmed it, and Matt's response pretty well set it in stone.

Just keep in mind what I said to Matt: I do not blame you at all for this. I blame the churches.

This whole thing has got me wondering which parts of the Bible are true. Is it only the things Jesus "says?"

And why should we believe anything you say, Gordon?

By Michael Fugate (not verified) on 25 Feb 2015 #permalink

Gordon @214:

Of course I’m only surmising from what you have not said. Perhaps you did sell all you had, gave the money to the poor and have been following Jesus for the past 10 years or more with no discernible positive result. But I doubt it.

Wow, the guy who claims the moral high ground and posts many posts to the internet per day is complaining someone else didn't give up all he owned. Better check for the PC or Mac in your own eye, Gordon, I don't think even the slimmest iPad mini is going to fit through that needle.

The larger point being this: both Walt and Another Matt have pointed out that current atheists are often people who have done (in the past) all the steps you claim are needed to find God. They've done all the sincere soul-searching and religious thought you claim we need to do, and they've done it for multiple decades running. GKC is simply wrong about the majority of atheists in the western world today: Christianity has not been left untried. Any claim that this doesn't count because they didn't give up all their belongings and walk barefoot between cities like Jesus and his followers misses the glaringly obvious point that in their Christian life, they were at least as sincere in following the commands as you and others have been, given that you haven't done that either.

@202;

If you want some clues, do some reading about what the Jews say about the Torah. Every name has a meaning. Every letter has a numeric value

Oh lord, you're a numerologist too?

Sean

There are some of us (me included) who are still willing to explore the possibility that the Bible has a coherent answer to the POE, but so far as I can see, Gordon thinks the Bible’s “coherent” answer is that God exists and operates outside of all moral systems, and we humans are not permitted to question God’s actions. Perhaps I’m mistaken, but that seems to be Gordon’s take on the biblical answer.

Two more points he's made:
1. Because God created us, it is not immoral for God to kill us.
2. Collective punishment of the innocent for the crimes of the guilty is morally justified, if you also collectively reward the 'innocent' for the good acts of a few to a (much) greater extent.

Like you, Sean, I do not see these as a coherent answer. Neither of these answers is, IMO, satisfactory, and in addition to that, they are not even internally consistent! The first argument would imply that the second argument was unnecessary and untrue; one would not need to give a reason why collective punishment is moral for God to commit, if one truly thought murdering innocent people was okay in the first place.

Gordon, this may come as a surprise to you, but I have never given Matt (or anyone) a license to speak on my behalf. No matter what he wrote, he cannot confirm anything about what would convince me. He does not know. Clearly, neither do you, even though you’ve claimed to have gotten information from your God.

You claim your God has already told you what will convince me, but now that claim is revealed as a lie. You don’t have a clue.

I don’t need to prove you wrong; you cannot prove you are right and you have proved yourself a liar and unreliable. You have destroyed your own credibility. We all know now that anything you claim is likely to be a lie.

And, No: I’m never going to let this one go, not so long as you make claims about what you want us to believe. You are an unrepentant liar.

You complain about others not engaging you in discussion, and then run away and hide when the step up to do so.

That's harsh, but at least it's honest.

The only things I need to know are some things that I DO KNOW:
I KNOW I have no credible reason to believe in any deity.
I KNOW I have no credible reason to deny any deity.
So I KNOW I have good reasons to remain a doubter.

sean s.

By sean samis (not verified) on 25 Feb 2015 #permalink

Michael @ #218,

Yes, I know I have only made an assertion. The reason is because an explanation would take too long. I don't even know where to begin, nor how to summarize my thoughts on such a profound subject in a way that would be meaningful to you. So I only placed the seed of thought, as ideas like this have a way of confirming or disproving themselves once they are in our heads.

I try to be very precise in how I use words. Can you give me an example of where I "change the meanings of words"? If you are talking about "empirical", I cited the dictionary definition for you.

"verifiable by observation or experience" is not exactly the same as testable in a laboratory. You can observe the stars, and you can experience the same sense of wonder, whether from looking at them, or from trying to get your head around the sheer scale of billions of billions. That is real-life experience, based on observation, that leads most men to conclude that there is something greater than them.

There is no theory or logic here. I can repeat the test every night if I wish. So far, looking up at the stars on a dark, cloudless night has never failed to fill me with a sense of awe. Perhaps you live in a city and just have not tried it recently. I would be surprised if you told me it did nothing for you.

@ #220

I take the Bible as a whole. The modern Evangelical church tends to focus on the teachings of Paul, and avoid the teachings of Jesus, and to discount the Old Testament altogether. Thus the reason I mention the centrality of Jesus' teachings in all this.

As for believing me, I assume you are referring to my take on the Bible. You don't have to believe me. All I am trying to do is show the connections between various aspects of Bible teaching. I feel the church has done a bad job, and I have had to go find answers for myself. I am just sharing those answers. They either add up for you or they don't.

By Gordon (not verified) on 26 Feb 2015 #permalink

In reply to by sean samis (not verified)

eric,

I think you and I are on the same page.

sean s.

By sean samis (not verified) on 25 Feb 2015 #permalink

The only difference I suppose is that the church didn’t completely fail me. I learned certain valuable truths that transcended all the mistakes I saw around me. The more I saw the church deviating from the Bible, the deeper I dug into the Bible for answers. Eventually I left the church altogether. But because I did find answers in the Bible, I never lost my Faith.

You have a twofold historical problem, then. First is that the Christian Church predates the Christian scriptures, and was responsible for deciding which scriptures became canon. The idea that the scriptures (plural) could be regarded as The Bible (singular) is quite recent. Not only that, but there were doctrinal reasons why there had to be four gospels (because there are four cardinal directions), and why the four we have now were chosen in favor of others. It's not that they thought they were the best historical account or had the greatest insight, but that they were orthodox, and orthodoxy is something decided by the church. It was not derived from the Bible. The radical sola scriptura stance you have taken would have been unthinkable prior to the Reformation, which is the second part of your historical problem:

It takes a lot of chutzpah to claim that, 2000 years into Christianity, that you have found the true, obvious meaning of the scriptures, which was simply ignored by 16 centuries of Christian communion and tradition and only found its first glimmer of existence when Luther nailed his theses to the church door. And yet there is nothing you could say that would convince the Pope he is wrong to proclaim the Magisterium because he believes that the preponderance of Christian history is on his side. And actually, I think he has a better case than you do, however much I loathe RCC doctrine. But notice that this kind of crass cultural imperialism is how early Christians treated the Jewish people and their scriptures – because they thought they had God on their side, they claimed the authority to explain the sacred writings of a culture and time they cannot possibly have understood, to the very people from whom they were co-opting those writings. For your next trick you may as well read the Mahabharata and then presume to tell Hindus what it really means, that they haven't picked up on in all the centuries of its existence.

By Another Matt (not verified) on 25 Feb 2015 #permalink

Gordon, this may come as a surprise to you, but I have never given Matt (or anyone) a license to speak on my behalf. No matter what he wrote, he cannot confirm anything about what would convince me. He does not know. Clearly, neither do you, even though you’ve claimed to have gotten information from your God.

Thank you. I hope it's clear to everyone that I'm speaking only for myself in this forum.

By Another Matt (not verified) on 25 Feb 2015 #permalink

Another Matt;

It’s no problem. I’m sure Gordon doesn’t have license to speak for you either. The difference is you care about that, Gordon appears not to.

sean s.

By sean samis (not verified) on 25 Feb 2015 #permalink

<Gordon @219 sez
So God has indeed heard my cry, and answered me...

I've prayed to your gods, Gordon. I asked to be converted to a believer. Three times. In public. In writing. Know what happened?

NOTHING!

So either your gods did not hear my cry, or they did not answer.

Or both, since they do not exist.

To Gordon #208:

Besides “Orthodoxy”, make sure you check out “The Everlasting Man”.

By See Noevo (not verified) on 25 Feb 2015 #permalink

If there was just a way to do it, I’d bet an extraordinary sum of money that
One day, evolution will be revealed to all to be perhaps the greatest embarrassment and shame in the histories of science and of rational thought.

That’s the truth.

By See Noevo (not verified) on 25 Feb 2015 #permalink

See No, sez, @231
One day, evolution will be revealed to all to be perhaps the greatest embarrassment and shame in the histories of science and of rational thought.

But nowadays, evolution stands as the most powerfully explanatory theory ever devised. It explains a myriad biological facts, apparently unrelated, and all using the same relatively simple mechanism.

Of course your wish may come true, but if I trusted you even a little bit, I'd take your money.

See Noevo;

If I could, I'd happily take that bet.

sean s.

By sean samis (not verified) on 25 Feb 2015 #permalink

That’s the truth.

That's your opinion and, based on your demonstrated absolute ignorance of evolutionary theory, it is a waste of electrons.

By Michael Fugate (not verified) on 25 Feb 2015 #permalink

Michael Fugate,

“What words, what language?”

Well, the real activity happens with one word, hayah הָיָה It a command that says ‘Be”.
=
“If someone were there what would one hear and what would one see?”

I guess that would depend on what was being created. I’ve wondered whether light means from UV to IR, or whether it included everything in the EMS from gamma rays to AM.
=
“How do words form atoms, molecules, cells and the like?”

Oh, it wasn’t the words themselves. The words expressed the mandate to instantly exist, with separate orders to the animal kingdom to replicate.

How does evolution account for that? Did the DNA replication errors that provided separate male/female hardware, software and upholstery occur in tandem in the same LUCA?

I have another question. I can’t think of anything that is quite like human scalp hair, and in males, facial hair. What’s weird is that they don’t have a natural stop-growth mechanism. You don’t see apes with beards or really long hair, so I’m guessing that humans must have picked up some oddball mutation somewhere along the line. When do you suppose that happened? With Australopithecus, or a lot later on?

So Phil, about those replication enzymes.

I don't think anybody knows how they originated.

So what? God-of-the-gaps? Does all the stuff the ToE DOES explain just evaporate?

What's your point?

Oh and Phil - when people say you are a loony, they mean bull like this:

Oh, it wasn’t the words themselves. The words expressed the mandate to instantly exist, with separate orders to the animal kingdom to replicate.

You cannot offer any sane explanation for how your gods' magical powers work, because the powers are not real. They are fiction, not reality, and thus are not constrained by reality.

Phil, And you know all of this how? If I say those same words, can I make a universe or a human?

By Michael Fugate (not verified) on 25 Feb 2015 #permalink

Phil has both kinds of argument: from ignorance AND personal incredulity.

By Walt Jones (not verified) on 25 Feb 2015 #permalink

Phil, if you want to learn about the genetics of hair, look up all kinds of recent work on dog genetics. Scientists have been able to locate homologous genes in humans because dog breeds are highly inbreed and all kinds of weird things pop out. Did you know that dobermans often suffer from narcolepsy? Border collies - something resembling OCD. But I digress, you can get long hair, short hair, kinky hair, straight hair, furnishings like ear hair and eyebrow hair and mustaches. Dwarfism seems to be a different gene, but the affects are the same. Dog genetics - the place to start -dogs are mammals and share common ancestry with humans.

By Michael Fugate (not verified) on 25 Feb 2015 #permalink

The words expressed the mandate to instantly exist, with separate orders to the animal kingdom to replicate.

I don't know about you guys, but Phil reminds me of someone who cannot tell that Harry Potter is a fairy tale.

Harry too can express a mandate for something to instantly exist. He can give orders to the animal kingdom - he can talk to snakes, just like Adam and Eve!

Yes, look! Up in the sky! Superman can fly, the Shadow can cloud men's minds, and Spiderman does what a spider can!

Superpowers like those Phil ascribes to his gods do exist, and are quite common - in fiction. Phil just can't tell fact from fantasy.

Michael Fugate,

"Phil, if you want to learn about the genetics of hair, look up all kinds of recent work on dog genetics."

No, not dog hair....human scalp and male facial hair. And tell me the recent work on the DNA replication failures that resulted in sexuality.

Phil,

I don’t think anybody knows how sex originated.

So what? God-of-the-gaps? Does all the stuff the ToE DOES explain just evaporate?

What’s your point

Phil,

I don't think anybody knows how human facial hair evolved.

So what? God-of-the-gaps? Does all the stuff the ToE DOES explain just evaporate?

What’s your point?

phhht,

Evolution seems to only have the appearance of thundering explanatory power.

eric,

Threatening to punish someone’s innocent grandkid if that person believes in another God, “because I am jealous” seems pretty malevolent to me (Ex 20:5). That is not indifference to suffering, that’s actively propagating it. Or don’t you agree?

But as already discussed, this isn't what I'm talking about here, and so I STILL have no interest in pursuing this. But as an aside, disproportionate punishment doesn't indicate malevolence. Malevolent beings cause people to suffer for no other reason than that they like to see them suffer. The case you cite is an example of disproportionate punishment, perhaps, and perhaps an example that would be morally wrong, but not of malevolence.

Let's not let the way the argument is phrased be the thing that does the heavy lifting as opposed to the actual arguments.

Re: 2. The problem with comparing theodicy to logical paradoxes is that, at some point, you must either concede that some evil or suffering is not a logical paradox, its contingent and therefore not necessary, or concede that God and the world are both locked-in on every single act because any other option is just a logical paradox human’s can’t see.

Which I didn't. There were two examples there, and both were comparisons where at first glance it looks like there's a major issue or inconsistency to be resolved, but when we look closer we can see that there isn't any real issue at all. Which I was clear about. After all, the evolution example CLEARLY wasn't about a logical paradox, and yet I still considered it an example, said why, and pointed out that for this case I wasn't even saying that on reflection it became obvious that there WASN'T a problem there. Again, eric, do you read entire comments before replying?

Because of that, I don't understand what the rest of what you say has to do with anything, other than potentially just restating the debates that we've been having, which is irrelevant to settling just what we agree on about how apparent the Problem of Suffering really is. So I'm going to move on.

Only in part. It also includes asking why God created humans the way we are and continuously allows us to be the way we are, when it certainly seems to be the case that many of us could be more peaceful, empathetic creatures without violating our free will. Just make the more violent of us have the free-will-including predispositions of the least violent of us.

Except that only comes into play when you deal with the considered response that at least some of the suffering is caused by the free choices of people to cause others to suffer, sometimes through violence, sometimes through neglect, and sometimes through other means. So it is, indeed, still the Problem of Suffering, as I said. Your reply here is to question whether we really did have to have the free will to hurt others, which is an additional argument on your part, and far, far past the "apparent" issue that we are starting with ... and is actually quite controversial for a number of reasons, including just how much of our predispositions determine that and whether there is a problem in giving people different predispositions -- or temptations -- to commit immoral acts. For me, the big trump here is that someone who overcomes a strong predisposition to do something immoral is more morally praiseworthy than someone who overcomes a weak one or not predisposition at all. Without us being able to want to do bad things, there is no moral praise in avoiding them. Only by us either conditioning our wants to the Good or overcoming wants to do bad can we be considered morally praiseworthy in any way.

So, again, this is already at the complicated level and isn't apparent itself, so it doesn't in any way impact my comment about what happens when we look beyond the first blush, shallow examination of the topic.

You’re right in your first part that a suffering-filled world is not consistent with the tri-omni God posited, and you’re right in your second part in pointing out that Christians must premise or axiomatically declare the world must include suffering to make any sense out of it. Where I think you go wrong is in seeing the latter as a “legal move” in reasoning rather than being a form of post-hoc apologetics.

Since that statement is in Genesis and is thus explicitly in the source text, it's certainly a legal move to say that it's part of the definition of the philosophical position and so that if people are trying to claim that the intent was for this world to be suffering-free then that seems to be an invalid move without extra support. You CAN still push for there being an inconsistency, but you will have issues logically with insisting that conclusions from derived attributes must trump direct statements ... especially since it still isn't clear that your conclusions are required, and that we have independent reasons to think that a world with no suffering is not necessarily a great world (think of the sense of well-being people who run a marathon get, which is fostered at least in part by overcoming the suffering they went through to get there.)

It is not good reasoning to say that since the world includes suffering, the concept of a benevolent omnipotent etc. god must somehow be consistent with a world that includes suffering. There is no “must somehow,” that’s an apologetic assertion or presumption.

But when the counter is "That God said that there ought to be suffering here in this world for reason X", then you are not dealing with a "must somehow" but with an explicit "That being wants that". You might conclude that then it can't be benevolent ... but that's the argument we're having, and the one that I've said isn't actually as clear and apparent as you think it is, for multiple reasons that I outlined in my comment and that you've mostly ignored.

The first part of your last sentence is pure assertion on your part, with no reasoning or logic or evidence behind it. This is again a form of apologetics.

No, as even you agreed it's part of the definition of what heaven is: no one thinks that people in heaven do bad things. That's what heaven means. You are trying to argue that another argument implies that people in heaven WOULD do bad things, which then causes a contradiction in the definition and so then causes an issue. You cannot then turn around and insist that I can't assume that the definition of heaven is as it is when you do that to make your objection.

No, it is not shown to be true that ‘people in heaven don’t do bad things.’ What is shown is that the three ideas (1) Christian entry criteria (2) Christian insistence of free will in heaven, and (3) Christian insistence that bad acts don’t occur there produces a paradox.

Actually, you haven't shown that. At all. You've argued that based on ANOTHER argument that in this world, at least, we need to have the capacity to act in bad ways to have free will, and so if we maintain free will in heaven we'd need to have that capacity as well, and so essentially argued that if we can have the sort of free will that doesn't produce bad actions in heaven then we could have it here as well, and so that that argument doesn't do anything about the Problem of Suffering. There are numerous ways your argument fails, but the one that's most relevant for this combination is this:

The main reason why we need free will in this case is to be able to freely choose to shape ourselves and act so that we meet the entrance criteria for heaven. Once that's done, free will directly doesn't matter as much, leading to the range of theological options that I outlined and you ignored, reproduced here:

This ranges from they are moral people — ie they won’t deliberately act immorally — to they are people who won’t act immorally without the temptations of the flesh to they are people who believe in God and so won’t deliberately disobey his commands to they are people who believe in God and/or are moral and will be buttressed by God after having made the free choice to strive to meet the entrance criteria to their free will will be taken away because it’s no longer needed once they’ve been judged. All of these mean that in heaven there will be no need for more tests or growth, because the selection is already done, to whatever criteria you need.

With people who've passed the tests, there are no more reasons for tests. Free will is the base requirement for us to even be able to take the tests and develop appropriately.

Instead, 3 seems to be taken as a premise, from which they work back to try and find some argument that will justify it. Which is, again, apologetics.

Or, rather, presuming it in replying to an argument that PRESUMES it or else doesn't work as any kind of criticism.

’m again having trouble parsing your statement, but I think my answer is: I don’t know of any sect that posits the first part of your ‘also’ so your question is moot. AFAIK the standard christian doctrine across many sects, probably unstated but here’s my most educated guess, is that everyone has free will in heaven and yet there is no evil performed there.

Since there are sects that deny free will -- I think Calvinism is used as an example of one typically -- this is just plain false. So, again, either the view thinks that there is free will in heaven and so that the entrance criteria must filter out those who won't do bad things in heaven, or they don't think there is free will in heaven and/or that it's not required there and so don't have the problem you cite.

By Verbose Stoic (not verified) on 26 Feb 2015 #permalink

Walt Jones @ #216,

This is what I determined regarding the biblical commandments:

The 613 commandments comprising the Law of Moses are necessary to regulate man's fundamental mistake. That mistake was the concept of possession or ownership, which then led to barter exchanges and finally to money. So, if you want to keep the mistake, then you need to follow the Law of Moses in order to minimize its negative effects.

If instead you eliminate this initial mistake, as Jesus taught us to, then you only need a few simple commands instead. Most of the problems regulated by the the Law of Moses simply disappear. For example, "Thou shalt not steal" serves no real purpose in a world where no one claims ownership.

Christopher Hitchens' challenge is rigged, by the word "could". What it means is that if I knew of some example that met the challenge, you could prove me wrong by immediately doing that action (or teaching it), assuming you had established your credentials as an unbeliever.

But a related observation can be made that at least points towards the sort of ethics we are choosing. When you argue with Christians, on the whole they are polite towards those who disagree with their position. The same cannot be said of atheists. This forum is a refreshing exception, as I have already noted, but if you go to any YouTube video where folks are debating evolution or Christianity or a similarly divisive topic and simply look at who is being polite, the percentage of polite Christians will be about equal to the percentage of impolite atheists.

So while anyone can or could utter or do any ethical behavior as a one-off, what we observe in practice is that Christians tend to take ethical advice such as, "Love your enemies" a bit more seriously.

Perhaps politeness is in the eye of the beholder. While you have been nothing but polite in our exchanges (and I have tried to respond in kind), it is my personal experience that the majority of Christians that are involved in these discussions tend toward the opposite. Of course, that may also be my perception of it clouded by my own personal biases, but (just as one example) I've been told countless times by Christians that I deserve to be tormented for all eternity because I didn't listen to them. I just don't consider that to be polite behavior. Even if it's something you sincerely believe, it's not something that is polite to say to another human.

Gordon,
So your answer to the Hitchens challenge is no. I resent your accusation that I would argue unfairly - that is hardly polite. A more intellectually honest approach would be to give an answer, and then if I respond as you imply I will, the burden of proof shifts to me to prove that atheists can and do that action, or explain why it is not an ethical statement or action.

By shifting the argument to my character, you are dodging the question.

By Walt Jones (not verified) on 26 Feb 2015 #permalink

Sean S @#215 and Eric @ #222,

You list some side issues that have taken the main discussion off track. To just lay the PoE issue to rest, here is an article where the author nicely summarizes the "Free Will Defence" (FWD), and then refutes it:

http://infidels.org/library/modern/niclas_berggren/theodicy.html

Section 3 contains his arguments against the FWD, which are based upon the Christian doctrine of Original Sin he has summarized just above.

I have to concede that he has done a fair job of summarizing the standard doctrine, both Catholic and Protestant. And based upon those, his rebuttals are correct.

So I find myself with the unenviable task of having to go correct established Christian doctrine with the Christian world first. There is where my real argument lies.

So I will only say this in closing: The author's own argument in section 3.2 absolutely destroys the Protestant theology of Original Sin, and especially Calvinism. No wonder atheists are having such a heyday using the PoE against fundamentalist Protestants! The Catholics, unfortunately, do not fare too much better.

Perhaps you can guess from my frequent references to Adam and Eve that I have my own ideas on Original Sin. It is my personal take on the issue that does stand up to the author's argument. But at the same time it essentially makes me a heretic. So this is an argument for me to have with the church, not with non-believers.

Section 3.1 is also a good argument. I have an answer to it as well that is consistent with the Bible, and that I'm sure will not have me branded a heretic. But it is pointless to even discuss before I've sorted out the main problem above.

3.4 and point 3 of 3.5 have also got me thinking more seriously about omniscience as it relates to time. Is it possible that God cannot see all of the future? I have already had some interesting ideas along these lines, but again, they are for me to take up with Christians, because they are a departure from the standard theology.

Finally, if you read Another Matt's post @ #209 and my reply @ #217, you will see what really brought home to me that I am debating with the wrong people. I need to win this argument with other Christians first, and get them to mend the errors of their ways. If I succeed at this, I suspect there will be far fewer atheists for me to argue with!

So I thank all of you for helping me understand both your own thinking, and where the church has got things wrong, and for being patient and polite through the whole process. But my plan from here is to tidy up a few last issues, and try to find some Christians who will engage me at least as thoughtfully as you all have.

Slight correction - if you say something that requires belief of something supernatural, you have the burden of proof to show that it exists.

By Walt Jones (not verified) on 26 Feb 2015 #permalink

Sean S @ #223,

Sorry Sean, I seem to have hit a nerve. I was really only trying to have a bit of fun, not to ruffle your feathers.

Let's recount what got us here, so you can see what I was trying to communicate:

I was asked to provide evidence of God that would convince you all, and the suggestion was made that I pray to God and ask him what would convince you.

I answered that nothing would convince you, and cited a parable of Jesus.

You then repeated the above challenge.

I asked you what would convince you, and also repeated my above answer.

You jumped all over my question, as proof that God had not told me what would convince you. But you either ignored or missed my answer. You did not say that I was wrong, and that there was something that would convince you. You misread my statement that "God told me" what would satisfy you. You assumed that God told me "something" that would satisfy you, when I had given Jesus' answer that "nothing" would satisfy you. So things were a bit confused at this point.

Meanwhile, something profound had happened. God really had answered me, but not in the way that I expected. Because while you were writing your taunting yet confused reply, I was replying to Another Matt. His post had told me what I had failed to understand about the mindset of many of you whom I was debating. It changed my perspective significantly.

Now I am not saying that Matt speaks for you or that your situation is even similar to his. I don't know. But what was clear from his reply was that there really was nothing I could say that was going to be convincing to you all. The problem is not the lack of a rational argument. It goes deeper than that.

And from your own words, I believe it applies to you as well. You wrote: "I KNOW I have no credible reason to believe in any deity." While your circumstances are no doubt very different, I can still say that this is because the church has failed you. I don't know how or when or why they failed you. But even the failure to communicate a credible reason to believe in God is a failure. After all, you were not raised in the Steppes of Central Asia, having no exposure whatsoever to Christian teaching.

The funny part for me was the timing of it all. For as soon as I posted my reply to Matt, I then read your reply @ #215 where you wrote: "If your God is real, and if your God hears prayers, then your God heard my prayers. He knows what I need, and He’s clearly not told you."

I read this right after writing, "Matt, I want to thank you for your forthrightness here. I think you have answered the issue of “evidence”." So you see, embedded in my response to Matt was my admission that I had received the answer I needed.

So this was the only difference. God, who I believe in, had indeed answered me in a direct way. But it was not the answer that you needed, nor was it the answer that I expected. But I knew it was THE answer. My debating was over. The problem was not the lack of convincing evidence. The problem was bigger than that.

Especially important for me was that this answer showed me that the problem was not with a bunch of "hard-hearted atheists", as other Christians might view you. It was a problem within the churches themselves. I was directing my energies in the wrong place.

So, for what it's worth, you really did get to witness God answering me. It was not the answer I expected or the answer you wanted, but it should be pretty hard to deny, after reading my last three paragraphs to Matt @ #217, that I had taken it that way. And this was before I saw your reply to me @ #215. They had "crossed in the mail", as it were.

That's all I was trying to communicate, in a lighthearted way, in my reply to you @ #219. It was late and I was off to bed after that, so I did not put much thought into that brief reply. Hopefully I have explained things better to you now.

To phhht #236, #243, #244:

Regarding replication enzymes and sex and human facial hair, your response to Phil is the same:

“So what? God-of-the-gaps? Does all the stuff the ToE DOES explain just evaporate? What’s your point?”

I think Phil’s point is that you’re a gapper, too. YOU invoke an EVOLUTION OF THE GAPS, over and over and over.

By See Noevo (not verified) on 26 Feb 2015 #permalink

Another Matt @ #225,

I'm still tracking with you Matt.

I was raised with the radical Sola Sciptura notion, which gives away that I was raised in the Protestant church. That is the church that I left, as Sola Scriptura took me places that no Protestant church was teaching.

I spent a number of years thinking that no church taught what I now believed. Then I found, to my great surprise, that the Catholic church did teach it, but just not in the way that I expected. For example, Protestants make no distinction between serving God and serving money. They figure if you put 10% into the offering plate on Sunday, all else is forgiven.

The Catholic church also does not demand anything drastic from the laity. But if you want to become a Priest or join a religious order, i.e. explicitly devote your life to serving God, then you must take a vow of Poverty, which removes all possibility of serving money. So there is Jesus' teaching, front and center in the Catholic faith, but just not where I expected to find it.

After overcoming my Protestant prejudices, and further exploring Catholic teaching, I became a full member of the Catholic church several years ago. So I am no longer outside of the church, and I do acknowledge the reality that it was the Church that made the Bible, and not the other way around. The Protestants have simply got that wrong.

So Sola Scriptura and the guidance of the Holy Spirit led me back to the mother church. A bit ironic, eh? But I am not the only one who studied the Bible enough to become convinced that the Catholics really had it right all along.

But while I hold great respect for the body of theology developed over the last 20 centuries, I still believe there is room for the church to clean up its act. If you read my post #251, you will see that the article I link to on the Problem of Evil cites the Catholic Catechism on Original Sin. The author then goes on to use this definition to prove his PoE argument. And I agree that he gets the logic right. But the problem is not with his logic, it is with the Catechism.

So there is still room for improvement. And for whatever its worth, I see a better way of explaining Original Sin that doesn't alter the fundamental concept, but that does make big changes in present doctrine. But it then also answers the PoE. And even though I want to make Christian theology more consistent, on the surface most churches will assume I am just another heretic. So I do indeed need a bit of chutzpah.

Phhht @ #229,

As I have said to others here, the church has done a bad job of guiding sincere people such as yourself. God works through other people, so when they fail us, God, in effect, fails us.

But this is a temporary problem. God has heard you, and he will answer. Probably when you least expect it.

I will just tell you one of my own experiences. In my younger years, as a devout believer, I battled serious depression. I prayed about that and related issues for years and years. And I wondered why God wouldn't fix the problem.

It took over ten years, but one day I realized that I hadn't been depressed for the last few months. Another decade later a psychiatrist was questioning me about my depression, and I told him that I had never once been depressed since the problem had ceased. Moreover, I told him with absolute confidence that it was impossible for me to become depressed again.

Well, I have had some things happen since then that would cause depression in most people, yet I have not known a single day of depression in all this time. So, when God finally did answer me, the answer was definitive. It can be Hellish while waiting for his answer. But once it comes, you will find that it was well worth waiting for.

"Ask and ye shall receive" is one of the most powerful principles in this world. But unlike magic, you don't just wave a wand a see the result in the next moment. Sometimes the answers come quickly, but in the early years of my faith I found that most answers were very slow in coming.

You have asked, so you have set something in motion that cannot now be stopped. And when the time is right, it will happen.

Welcome Home, Gordon!

By See Noevo (not verified) on 26 Feb 2015 #permalink

Sean T @ #248,

You are right Sean, that is not polite behavior. And I know that far too many Christians fall back on petty stuff like that when they know they are losing an argument. But few of them start out that way. I debated some atheists on a few YouTube videos and soon got sick of being constantly insulted. And petty insults, such as calling me ignorant simply because I did not accept the Theory of Evolution.

I think we both agree on what constitutes good behavior, and we both wish that our respective camps would do a better job at it! I have a plan for cleaning up my side of the aisle, even if I think they get higher marks already. Their marks are not high enough, as far as I'm concerned.

So I think the only thing I can claim is that Christianity does a better job on the whole of communicating a decent set of ethics. But it still does not live up to its promises.

Phil says, @245

Evolution seems to only have the appearance of thundering explanatory power.

And yet it actually DOES explain literally tens of thousands of superficially unrelated biological facts. Remember the list @167? Those are only a few of those facts.

Of course the ToE doesn't explain everything; nothing does. But that is a limitation of human intelligence, not the ToE.

See No says @254

YOU invoke an EVOLUTION OF THE GAPS, over and over and over.

Nope, you're wrong.

In order for there to be a similarity with the god-of-the-gaps fallacy, I would have to say "We don't know, so it must be that evolution did it."

But I don't say that. I just say "We don't know."

I repeated myself, word for word, because Phil was repeating his empty criticism of the ToE almost word for word: "Here's something the ToE does not explain," with the implication that the ToE is therefore worthless. But that's just silly.

Walt Jones @ #250,

Correct Walt, my answer is "No" to Hitchens' challenge.

I was not trying to accuse you of anything. I was just using you as an example to show that, due to the word "could", anything that had not yet been done "could" be done in the very next moment after the deficiency had been pointed out.

I don't really see what point Hitchens even makes with this. It is not like anyone teaches that moral behavior is only possible once one becomes a Christian. Some lone backwoods preachers might teach that, and many Christians might think that way, but find me one statement out of any church's catechism that says as much, and I will be very surprised.

Where I shifted the argument to was the general behavior of atheists as a group when compared to Christians. See my response to Sean T @ #258 for more on that.

Gordon:
I'm sure Jessica Ahlquist disagrees with your opinion of Christians.

Yes, Hitchens's challenge is to the idea that morality comes from religion, so I probably did take it out of context.

BTW: I probably missed it - here or an earlier thread, so feel free to give me a rough location if you have answered this: why do you believe the bible is the word of God?

By Walt Jones (not verified) on 26 Feb 2015 #permalink

In reply to by Gordon (not verified)

Gordon,

You might be interested in the work of an anonymous blogger I have spoken with in other forums. Check out his series on Original Sin:

https://turmarion.wordpress.com/2012/07/03/legends-of-the-fall-index/

and also on universalism, which is related:

https://turmarion.wordpress.com/2012/11/04/universalism-what-the-hell-i…

The sub-series are also interesting.

He also comes from the Catholic tradition, and writes very well about all we have been talking about here, however much I disagree with him. Theology is still interesting to me as a kind of anthropology, and it's fun to engage on it (that and sports is all my family really wants to talk about), even I'd much rather be talking about art and science. I'm in classical music, and theology plays an obviously big role in music history as well, and so it's important to be able to understand why e.g. a Bach Cantata is the way it is.

As an aside, I'm glad my story made you think a bit differently about atheists, but I'd appreciate it if you didn't use it against the other members of the forum here. I don't think anyone needs to have my background to come to similar conclusions, and I really wouldn't want to wish anyone a half a lifetime of being trapped in religion just so they could say they tried it.

By Another Matt (not verified) on 26 Feb 2015 #permalink

And scientists are actively working on filling the gap, while the ID folks are looking for more gaps, instead of producing reproducible research into the existence of a designer.

By Walt Jones (not verified) on 26 Feb 2015 #permalink

Gordon @256

So what's the problem, Gordon? It's already been years and years. Are your gods just behind on the paperwork? Are they, as Tom Waits says, away on business? Or is this just another one of those mysterious ways of theirs?

I see no reason whatsoever to think your gods will ever answer my prayers. As far as I can tell, gods have no tangible effects on reality at all. As far as I can tell, they do not exist. I expect to go to my grave holding to that conclusion.

See Noevo @ #254

EVOLUTION OF THE GAPS

Precisely!

@ #257 - Thanks. 30 years in the making

One of the really encouraging things is finding out how many Catholics are deep thinkers. Here is a great perspective on the Problem of Evil from a Catholic blog:

http://www.patheos.com/blogs/inebriateme/2014/05/against-theodicy/

He basically makes the point that I brought up early on. "Yes, there is Evil in the world. Now, what do we propose to do about it?" Our Faith should move us beyond the argument to this critical question. And from there to an answer. Otherwise we have nothing to offer but dry bones.

Phil, that's how evolution works - dog hair and human hair are coded by the same genes. You can study either and get the same answer. It is easier to study dogs - because we have few moral qualms about breeding them and many lines are heavily inbred. This makes it easier to find genes.

You do know that when we study biology - we don't teach a different biology for each species? Physiology, Genetics pretty much identical across the taxonomic spectrum.

By Michael Fugate (not verified) on 26 Feb 2015 #permalink

Another Matt @ #262,

Thanks for the links Matt. It looks like a lot of reading, but I will definitely check out what he has to say.

Sorry about the misunderstanding with Sean S. I was not trying to use you against him in any way. I attempted to clear things up with him in post #253. We'll see if I succeeded. I'm not sure how you feel about being God's answer to me, but you were.

Gordon,
Um, like "design" and "wall"...

Paul never did anything for me - I could never see any connection between the Gospels and the rest of the NT.
I was never quite sure what to do with the OT either - I would probably chuck the whole thing; we know it is wrong scientifically and historically. Christians and most Jews ignore most of the rules and regulations. Most are specific to a time and place that no longer exists.

By Michael Fugate (not verified) on 26 Feb 2015 #permalink

One of my favorite things about winter is being able to see Orion the Hunter in the night sky before I go to bed. He is moving on to the west now and will soon be gone. The last winter storm we had produced a great rainbow too - double in parts. What does this have to do with God?

By Michael Fugate (not verified) on 26 Feb 2015 #permalink

Gordon @256

I too have bouts of serious depression, sometimes lasting for years. I too have experienced remission.

But there were no gods involved.

That's just the way the disease works.There is no supernatural Prozac, nor is there any reason at all to assume one.

Walt Jones @ #265,

I love the hypocrisy of the whole notion of one person imposing their will on everyone else, and then everyone is just supposed to take it. Imagine Jessica was a Christian in a school predominantly of atheists, and could use the power of the State to force Bible verses to be posted all over the school. Would she still be alive today?

Yes, it goes against Jesus' teaching of turning the other cheek, but America is populated by the descendants of precisely those Christians who would NOT turn the other cheek. Their cousins who did, don't exist any more. And this attitude of not taking things lying down is common of all Americans, not just the Christians.

Atheism is now the only officially State-sponsored belief system in America. Shoved down everyone's throat, whether they like it or not. Religion, no. Belief system? Absolutely. Hypocrisy in the extreme. A neutral atheist would simply walk past religious displays, allowing others to practice what they believed, so long as he or she was not forced to participate.

The Bible is unquestionably the wisdom of the ages; a sacred text since Moses handed the first five books of it to Israel at Mount Sinai. The Bible calls itself the Word of God, which is a claim that it speaks to us today. And it does. Even were it nothing more than a handbook for morality, it contains a complete set of rules for life, at least as good as any other religious book out there.

But it is more than that, because God does use it to speak to us today. So often when I am looking for answers, a Bible verse will come to mind or be brought to my attention, and it will speak directly to the issue I need answering. And I am talking about reliable answers that produce the results I want, consistently. For those of us who take it seriously, it fulfills its promise of being, "Living and active and sharper than any two-edged sword."

By Gordon (not verified) on 27 Feb 2015 #permalink

In reply to by phhht (not verified)

Yes, I too suffer from fairly serious depression. The worst of it subsided when I left Christianity for good. The rest has been handled mostly with medication and treatment of an autoimmune disease I just found out about a couple of years ago.

By Another Matt (not verified) on 26 Feb 2015 #permalink

"One day, evolution will be revealed to all to be perhaps the greatest embarrassment and shame in the histories of science and of rational thought."

You See a lot See Noevo. I also foresee a collective "oops!" on the part of proponents of this theory and a greater "OOPS!!!" on the horizon for atheists. Skimming through these threads, I see the persistent demand for believers to prove God (when proof is actually EVERYWHERE), but where is the proof for this (Creator excluding) science fiction? We often hear that the burden of proof is on believers to prove God, but it seems to me that the burden should be on anyone who insists God doesn't exist, to disprove Him.

By Lt. Wassabewabee (not verified) on 26 Feb 2015 #permalink

Lt @ 273 says
I see the persistent demand for believers to prove God (when proof is actually EVERYWHERE)...

Name it. Tell me how I can test it to see if it is real, or just the empty rambling of a deluded mind.

Michael Fugate,

My question is not about hair/fur in general. It is about human scalp and facial hair. Why does it not stop growing?

I have another question. I can’t think of anything that is quite like human scalp hair, and in males, facial hair. What’s weird is that they don’t have a natural stop-growth mechanism. You don’t see apes with beards or really long hair, so I’m guessing that humans must have picked up some oddball mutation somewhere along the line. When do you suppose that happened? With Australopithecus, or a lot later on?

If I remember correctly, human hair has basically the same phases of growth and cessation as other mammals; it's just that the growth cycle for humans is longer. An individual hair will only grow for so long before it stops growing and falls out. Why the human cycle is longer is another question, but it's not the case that human hair has no stop-growth mechanism.

By Another Matt (not verified) on 26 Feb 2015 #permalink

Phil,
So animals like dogs can have continuously growing hair like humans. Many equids do also on the mane and tail. It has to do with the length of the growth phase of the individual hair. In humans head hair is 2-7 years, arm hair 1-2 months. No doubt there are regulatory genes involved.
In dogs with two copies of the recessive allele long hair and the allele for furnishings - hair grows like human head hair.

By Michael Fugate (not verified) on 26 Feb 2015 #permalink

Matt beat me to it - thanks Matt. The important thing here is that selective breeding in dogs can achieve the same result that shows up in humans. A pattern of longer hair on some body regions which shows in other mammals as well. This is the beauty of common ancestry in biology; we can study one organism and predict that the same genes will be involved in another organism with the same features. Dog studies have allowed scientists to make primers from dog genes and probe human DNA to find homologs. This is great for studying genetic diseases.

I would imagine that mate choice goes into the switch, but it is not easy to reconstruct selective regimes when there is no variation left.

By Michael Fugate (not verified) on 26 Feb 2015 #permalink

Another Matt,

"it’s not the case that human hair has no stop-growth mechanism"

There appears to be no such thing. Think Rapunzel, Crystal Gayle, or this girl: http://www.worldslongesthair.com

There is obviously a difference between human hair (or beard) growth and that of apes. I’m supposing that you think our genes were altered at some point so that we have this peculiar distinction. Would this have happened early after the supposed divergence, or later on?

phhht,

“Remember the list @167? Those are only a few of those facts.”

You made a list, but whether or not those things support evolutionary theory is another matter. Why don’t you pick out one or two of your favorites and explain why you think they are evidence?

===

Walt Jones,

“…why do you believe the bible is the word of God?”

In a work, prophecy.

Phil,

No thanks. In my view, you are unable ever to accept the theory of evolution, no matter how many facts you are presented with. Your judgment is impaired by your religious disorder.

phhht,

But you seemed so focused and confident in your catalogue, and I was gonna let you pick the champions. Did you get tired of typing "I don’t think anybody knows..."? You could always just copy and paste it if one of your irresistible facts unravels, right?

Prophecy? The vague and post hoc ones, I assume, since the clearest one, delivered by Jesus himself, and including a time metric, didn't come to pass. (Or have you met the Wandering Jew?)

By Walt Jones (not verified) on 26 Feb 2015 #permalink

In reply to by Phil (not verified)

Phil, you've lied about reading and considering every other reference people have given you. They've gotten tire of your little game.

Phil

You're welcome to investigate any of those examples yourself.

God-of-the-gaps cuts no ice, Phil. It's a loon's trick.

Rapunzel? Come on, you can't just invoke fairy tales as evidence.

No, scalp and facial hairs do stop growing. Don't hold me to this because I don't have time to research, but I think I remember learning that about 15-20% of the hairs on your scalp are in the non-growth stage, where they've reached the longest length they will ever get, and will eventually fall out. Most of us would not be able to grow our hair out for 30 years and have it keep getting longer. It's certainly possible that the woman with the world's longest hair has a gene variant that doesn't regulate growth in the usual way.

By the way, are you just trolling on hair because of the dispute between Darwin and Wallace, the latter of whom had some qualms with the concept of sexual selection in animals, and who (again, if I am remembering correctly) thought human hair was proof of God's existence?

By Another Matt (not verified) on 26 Feb 2015 #permalink

Matt said, to Phil

Come on, you can’t just invoke fairy tales as evidence.

Ha!

Another Matt,

“No, scalp and facial hairs do stop growing.”

That isn’t the issue Matt. Humans have scalp hair and facial hair that will continue to grow for several years, and apes do not. This means that, from your point of view, somewhere along the line, human hair genes were altered. My question is this:

When do you suppose the alteration occurred? How long ago? 20,000 years? A million?

===

Walt Jones,

“Prophecy? The vague and post hoc ones”

Not many are what you could really call vague. Some didn’t make sense in their contemporary context, but became clear after the fact. Lots of the 1st advent prophecies fall into this category. Some are quite stunning, like Psalm 22 and Isaiah 53.
-
“the clearest one, delivered by Jesus himself, and including a time metric, didn’t come to pass.”

You probably mean Matthew 16:28

“…There be some standing here, which shall not taste of death, till they see the Son of man coming in his kingdom”

This is a preterist favorite, and I’ve heard some pretty good reformed theologians wonder out loud about it as well. The answer is really pretty easy. A much clearer one (Luke 21:24) looked towards the destruction of Jerusalem and beyond, actually to our time and current events. But a significant percentage of the Bible is devoted to prophecy, many closed out, and many pending. These are not intended to prove anything to unbelievers, and won’t. They are not for you. You have your own space particle and DNA replication error stories. But in answer to “why do you believe the bible is the word of God?”, that is one of the things that authenticates the book. It takes no less than God to steer history towards a conclusion forecast many centuries ago.

“No, scalp and facial hairs do stop growing.”

That isn’t the issue Matt. Humans have scalp hair and facial hair that will continue to grow for several years, and apes do not. This means that, from your point of view, somewhere along the line, human hair genes were altered. My question is this:

When do you suppose the alteration occurred? How long ago? 20,000 years? A million?

A geneticist would be a better person to ask about this. I don't have the data in front of me, and if I did I wouldn't have time nor the training to do the statistical analysis. These kinds of things are inferable in principle, though. See for instance:

http://en.wikipedia.org/wiki/Molecular_clock

and

http://en.wikipedia.org/wiki/Molecular_phylogenetics

By Another Matt (not verified) on 26 Feb 2015 #permalink

So let's see, a book of legends and campfire stories from thousands of years ago contains the words of an invisible, immortal superhuman with magical powers who steers history towards some conclusion forecast in that book.

Uh huh.

timetree.org will give you splits for almost any pair of taxa.
Their estimated time for the chimp/human split is 6.3 million years.

By Michael Fugate (not verified) on 26 Feb 2015 #permalink

Phhht,

"Name it. Tell me how I can test it to see if it is real, or just the empty rambling of a deluded mind."

Just look in the mirror Phhht. If I hear another atheist demand proof while overlooking EVERYTHING, I'm gonna go phhhuckin' crazy!

By Lt. Wassabewabee (not verified) on 26 Feb 2015 #permalink

Lt@292

I've looked in the mirror, and while it is a very pleasant experience, I see no evidence whatsoever that gods exist.

Could it be that you're hallucinating the existence of gods?

Because there certainly is no testable evidence for their existence.

"Could it be that you’re hallucinating the existence of gods?"

No. There's no way (in hell) I'm hallucinating the existence of this Person phhht. I'm absolutely sure of the existence and it's not up for debate (but feel free to do so).

By Lt. Wassabewabee (not verified) on 26 Feb 2015 #permalink

It is interesting that Fry immediately speaks about a personal God with apparent omniscience when asked to imagine that atheism is false. The typical Jewish-Christian view of God seems to be the default alternative for Fry, even if he does not actually think there is any evidence for it. (I mention Judaism since Fry has described himself as "technically Jewish".)

Maybe God is very benevolent and powerful but rather dim and could not find a perfect solution where there was no suffering. Or maybe God is not actually all powerful, only having some limited ability to intervene to change things.

A common fantasy version of theology is that the god(s) actually need human worship to have any power. Maybe that is why God have not cured bone cancer, people are praying in the wrong manner. If I were to start a religion I think I would include this doctrine.

I think that atheists who want to spread atheism should be careful to point out the many versions of religions, actual or imagined, that look nothing like Christianity. By presenting it as a binary choice, believing or not believing in God, I think the theistic alternatives gain unwarranted credibility in the Western world. We should make it clear that even if atheism is false it implies nothing about the truthhood of traditional views of God.

And when I say "ABSOLUTELY SURE", I mean it phhht. Now if you're ABSOLUTELY SURE in your beliefs and you're committed to your atheism, disprove God. I know you can't, but I like serving that tennis ball considering how much believers are asked the opposite - and considering everything God has done to grab attention on His own.

By Lt. Wassabewabee (not verified) on 27 Feb 2015 #permalink

Michael @ #269,

If you'd like to provide the dictionary definitions that you believe I have mangled, I will be happy to consider them. It is you atheists who seem confused about what a "wall" and a "design" actually are. You look at them and say, "No, that's not it."

I was reared on Paul, in Bible-thumping churches who only seemed to understand his letters and not much else. It took me years to come to terms with the Old Testament, and a little bit longer to finally start understanding Jesus. My head had literally been programmed full of "interpretations" that completely negated what he taught.

Kierkegaard neatly summarized the average Christian's response should he take the words of Jesus literally:

"The matter is quite simple. The Bible is very easy to understand. But we Christians are a bunch of scheming swindlers. We pretend to be unable to understand it because we know very well that the minute we understand, we are obliged to act accordingly. Take any words in the New Testament and forget everything except pledging yourself to act accordingly. My God, you will say, if I do that my whole life will be ruined. How would I ever get on in the world? Herein lies the real place of Christian scholarship. Christian scholarship is the Church’s prodigious invention to defend itself against the Bible, to ensure that we can continue to be good Christians without the Bible coming too close. Oh, priceless scholarship, what would we do without you?"

To their credit, the churches I was in did take Paul's teaching seriously, and try to follow it. But once they started reading out of the Gospels, the words of Kierkegaard applied perfectly.

From our discussion in the previous thread, you might find this blog post interesting. It details the many ways that the principles of Hume are at odds with the New Atheism:

http://www.patheos.com/blogs/inebriateme/2015/01/david-hume-against-the…

This is what I'm talking about. Lt. Wassabewabee claims to have a perfectly well-working God Detector built in to his/her person. I don't have one, and grew up thinking I was broken. Can you imagine a reason why God would create someone like me? This isn't like blindness, which has physical explanations and has nothing to do with whether or not I am supposed to undergo an eternity of conscious torment.

By Another Matt (not verified) on 27 Feb 2015 #permalink

Phhht @ #271,

18 years of remission, and counting?

I could list for you those things that led to my overcoming depression, and some of them WERE supernatural. No drugs, all God and good things given by him.

"Atheism is now the only officially State-sponsored belief system in America."

Gordon, you've said a lot of incredibly stupid and false things, but that goes right to the top of the list.

Another Matt @ #272,

Yes, I can believe that much of your depression was caused by religion. Same with mine. Although it was combined with a perceived need to please my mother, who was the one who had dragged me to church in the first place. So I was caught in a cycle of despair where it seemed impossible to please God because it was impossible to please my mother.

How I learned that my mother was the root of the problem involved an answer to prayer that I can only describe as supernatural. An hour before that prayer, I was completely unaware that I had spent my entire life til then trying to please a mother who could never be pleased. I had no idea that she was the lens through which I viewed God.

Lt Wassabeewabee,

It seems likely that you're just a troll, but in case you aren't just on here to troll: you are a bit confused. Check the URL for this website. In case you can't read well, it's SCIENCEblogs.com. That tells us that this is, in fact, a science blog. Thus, the rules of science are what applies here.

One of the rules of science that we have found useful is something called Ockham's razor. This rule simply states that we should not needlessly multiply entities. That is, if there are two candidate theories to explain a set of observations and one of them requires some new particle, force, god, etc. to work and the other doesn't, we should prefer the one that doesn't require the new entity.

Now, we as scientists are making the claim that none of our scientific theories requires the presence of a deity to work as an explanation. We have provided quite a bit of evidence that this is so. Evolution is certainly not unique in that regard. Quantum mechanics, relativity, germ theory of disease, and pretty much all other scientific theories share that feature. None of them needs God to work. Why then, considering Ockham's razor should we believe in God based on scientific evidence? Answer: we should not, unless some new observation comes along that cannot be explained without invoking a deity.

Now, what would the nature of such an observation be? It would have to be an observation that admits of no natural explanation; that much is obvious. What is not obvious, though, is that if you want to invoke a deity as an explanation, you must also propose limits on what you might expect to observe if that deity actually exists. That is, it must be a testable hypothesis. All scientific theories must be testable. There must be some observation that it is possible to make that would have a different result depending on whether or not the theory is true. For instance, WRT evolution, we expect that all life would share a common system of coding for proteins, we would expect that the offspring of all organisms will show minor, but only minor, differences from their parent, we would expect that the fossil record would show certain patterns, such as the lack of fossils of anatomically modern organisms in Precambrian rock strata, for instance, and we expect that certain patterns of genetic mutations could be observed while others could not. For instance, we observe retroviral insertions that are shared by chimps, gorillas and humans. We observe others that are shared by only chimps and humans. What we do not observe is retroviral insertions shared by gorillas and humans, but not chimps.

If we did observe this, our model of human evolution would be wrong. If we observed things counter to the other examples, we would also question evolution. Now, what similar observations are there that would cause you to change your belief in God? If you are like most Christians, there aren't any. In fact, postulating omnipotence for God renders any such observation impossible; God could always alter the results of our experiments so ANY result is consistent with this omnipotent God.

The burden of proof is definitely on you, not on those who disbelieve. I have given you a template for what you need to do. Make sure your observations are repeatable so that any skeptics out here can check your work and confirm your observations. No "God revealed Himself to me", or "It's in the Bible" will count here. We need reproducible, objective evidence. Good luck.

Gordon - read the First Amendment of the United States Constitution and the Supreme Court decisions interpreting it. The U.S. was formed as a secular nation, and the Constitution was written to protect the minority from the tyranny of the majority.

Atheism is not a belief, much less a belief system. Atheism is the lack of belief in the claim there is a god.

You seem to be big into thinking about what people might do, and then use that hypothetical situation to justify real actions. I don't find that polite.

I was hoping that you'd have something more thoughtful to say about why you believe the bible is the word of God other than because it says it is. That's as compelling as Lt. W's proof - why should we believe some guy on the Internet who is absolutely sure, but won't explain why?

Phil,
Prophecies from the Old Testament predicting things in the New Testament? That might be credible if the writers of the latter weren't aware of the former.

By Walt Jones (not verified) on 27 Feb 2015 #permalink

Walt Jones,

“Prophecies from the Old Testament predicting things in the New Testament?”

There were people who had expectations about the Messiah based on what they could glean from the OT. But lots of the details of the first advent were obscured in their presentation. It would have been impossible for anyone to see the whole drama and stage any of it. Psalm 69:21 says:

“They gave me also gall for my meat; and in my thirst they gave me vinegar to drink.”

There are no stories about David, gall and vinegar. This was a projection of two events, both of which happened during the crucifixion. All four gospels mention the vinegar, but only Matthew records the offering of the gall.
-
“That might be credible if the writers of the latter weren’t aware of the former.”

There is no way that four accounts could be woven by four writers at different times, no reason for any of them weave a gigantic collection of lies, and no historical exposé of a conspiracy. The Gospel writers were obviously sincere and dedicated believers, but they were not that shrewd.

Gordon, perhaps you realized that there might be a difference between "looks designed by an intelligent something or other to me" and "is designed by an intelligent something or other"?

One is opinion the other is fact. Opinion is all you have. Hume didn't support intelligent design - period.

By Michael Fugate (not verified) on 27 Feb 2015 #permalink

Let me add, I have no idea whether there are gods or not - only that, if there are, they are nothing like the descriptions in religious books. Jesus or at least those who wrote on Jesus provide food for thought, but the miracles and god on earth business is nonsense. It is time to move on.

By Michael Fugate (not verified) on 27 Feb 2015 #permalink

Verbose Stoic wrote to eric

disproportionate punishment doesn’t indicate malevolence. Malevolent beings cause people to suffer for no other reason than that they like to see them suffer. The case you cite is an example of disproportionate punishment, perhaps, and perhaps an example that would be morally wrong, but not of malevolence.”

To cause or allow disproportionate suffering, or suffering of innocent third parties is malevolence or indifference, either of which amount to evil.

I’ve always wondered:
If Heaven is a paradise, do people there have free will?
If not, then FW cannot be all that valuable.

If there is FW in Heaven, do people in heaven sin?
If ‘yes’, then sin must be part of the deity’s plan.
If ‘no’ then why don't they sin? FW is supposed to cause sin, how can there be FW in Heaven without sin?

Answer: the missing things are knowledge and empathy. But if God can give us the tools in Heaven to avoid sin even while we have FW, then that same God could give us those tools here on Earth; all without impairing our Freedom.

That this God does not give us these tools would mean this God wants evil in our world, and therefore is evil.

Regarding, “With people who’ve passed the tests, there are no more reasons for tests..”

I understand why humans test things; this is necessary for us to gain the information we need to understand things. We are only human.

Why would a deity test us? Only if they in fact do not know us. So all the claims about God knowing me, knowing my innermost secrets, etc. would be false. Is God only human?

If there is a God in any meaningful sense of the term, there is no reason for tests in the first place. He already knows what He'll find. ‘Testing us’ does not explain or justify either evil or suffering.

sean s.

By sean samis (not verified) on 27 Feb 2015 #permalink

Lt @295 says
There’s no way (in hell) I’m hallucinating the existence of this Person phhht. I’m absolutely sure of the existence and it’s not up for debate (but feel free to do so).

@297 he says
And when I say “ABSOLUTELY SURE”, I mean it phhht.
Now if you’re ABSOLUTELY SURE in your beliefs and you’re committed to your atheism, disprove God.

Well, Lt, I am not "ABSOLUTELY SURE" in my conclusion (rather than belief) that there are no gods. I am only human, and I could well be mistaken. It's highly revealing that you arrogate to yourself that superpower of ABSOLUTE CERTAINTY. That sort of certainty belongs to pathology, and it's symptomatic of your religious disorder.

I cannot disprove the existence of gods, of course, but I don't have to. All I have to do is to understand that there is no reason to conclude that they are real. Gods are fictional characters, like vampires, zombies, and superheroes. In this world so chock full of empirical evidence for the existence of everything from apples to zebras, there is not a whit of such evidence for the existence of gods. In all of science, technology, engineering, and mathematics, there is no sign of a god. No one can detect a god. Gods have no unambiguous tangible effects on reality. Even those who insist on the existence of gods cannot agree on what they mean.

So why should I conclude that gods are real, Lt? What makes you think they are?

Folks, I’d think by now it’s pretty clear that believers cannot prove their Gods exist to others, and deniers cannot disprove any deity to those who believe in one (or more). So all this “Prove it!” “Disprove it!” does nothing but generate heat.

There’s boat-loads of evidence, but it all points in every direction imaginable. Unless you’ve had some direct encounter with something you believe is a deity, all that evidence leads nowhere. Everyone who has reached a conclusion did so by rejecting some of the evidence and embracing other evidence; for whatever reason suited them.

What matters to me is that believers (like Lt. and Phil and others) cannot demonstrate why the rest of us should believe them.

To be clear, I’m not talking about reasons to believe in some God, or to believe in the truth of some religious text, I am referring to the testimony of these people telling us to believe in their God or their Scripture. Their testimony is the ONLY evidence of any God. There is nothing else (apart from that direct encounter).

And their testimony is too confused and contradictory to convince. This is where the POE is so critical: it demonstrates the contorted logic of many believers and discredits their other testimony about their Gods.

Given this, I remain a doubter to all assertive conclusions about deities (for and against).

The faith I had once was killed, not by science but by silence; the silence of whatever deities might be real. Iff they exist, they don't think about me, and in that case I return their complement.

sean s.

By sean samis (not verified) on 27 Feb 2015 #permalink

I am curious how a mono-omni god, let alone a trip-omni one is compatible with the Bible? If the Bible is true, then god is not omni-anything. If god is trip-omni, then the Bible is not true. A case in point, if a moderately smart human in 0 CE wanted to save humans from sin by preaching the Gospel, would he or she send a person to a single place and have him or her walk around a 150km area preaching for three years? Humans were in Australia and the Americas tens of thousands of years earlier. Most humans at the time didn't survive childhood and if they did lived for 50-60 years. 500-1000 years to reach western Europe, 1500 to reach Australia and the Americas?

By Michael Fugate (not verified) on 27 Feb 2015 #permalink

It’s all about the Resurrection (and the witness, thereof).

“Then some of the scribes and Pharisees said to him, "Teacher, we wish to see a sign from you."
But he answered them, "An evil and adulterous generation seeks for a sign; but no sign shall be given to it except the sign of the prophet Jonah.
For as Jonah was three days and three nights in the belly of the whale, so will the Son of man be three days and three nights in the heart of the earth.”
……….
“For if the dead are not raised, then Christ has not been raised.
If Christ has not been raised, your faith is futile and you are still in your sins.
Then those also who have fallen asleep in Christ have perished.
If for this life only we have hoped in Christ, we are of all men most to be pitied.
But in fact Christ has been raised from the dead, the first fruits of those who have fallen asleep.”

By See Noevo (not verified) on 27 Feb 2015 #permalink

See Noevo;

You tried this one on us before:

but no sign shall be given to [them] except the sign of Jonah.

My answer has not changed: I would take the sign of Jonah in a heart-beat. Better than nothing, which is all I’ve gotten so far.

It’s interesting that when we ask for a reason to believe in your Good and Holy God (Perfect in Every Way!) the only response you can come up with are biblical threats and intimidation. So much for "good" reasons!

By now you must realize that cherry-picked Bible verses are not persuasive to those who do not believe in the biblical God. I can only conclude you bother with this because you’re trying to deal with your own doubts.

sean s.

By sean samis (not verified) on 27 Feb 2015 #permalink

See Noevo@313,

Your story book is full of zombie fiction, See. For example, according to Mark, I think, there were dozens of dead priests popping up out of their graves all over town about the same time Jesus did. He wasn't even unique.

It's interesting to me that zombie fiction remains as popular today as it was then. A great trope is immortal.

The most noble thing Jesus ever does in the Bible is to let Thomas touch his wound to be sure. Here was someone who was incapable of believing without evidence, and Jesus gave him the evidence. He didn't give him a book and say "read the prophecies." Jesus's actions in this story can be construed either as a warning: "don't be a Thomas," or as an indication to other Christians that not every skeptic is a Judas -- at least as many are Thomases, and your judgment of the Thomases is problematic at best. Belief is not something I can just will myself into; I spent many futile years trying.

By Another Matt (not verified) on 27 Feb 2015 #permalink

"It’s all about the Resurrection (and the witness, thereof)."

But as has been pointed out, there is no external reference to this event - no evidence for it, or even the crucifixion, at all. It (the resurrection) is just a story written to close the story of jesus.

phhht,

“For example, according to Mark, I think, there were dozens of dead priests popping up out of their graves all over town about the same time Jesus did.”

Matthew 27:52-53 This was a sideshow to the liberation of OT saints from paradise. The veil being split in the temple was a signal that the atonement was “finished” at the crucifixion, and there was no longer a barrier between dead men and God.
-
“It’s interesting to me that zombie fiction remains as popular today as it was then.”

Not quite the same thing, but ideas definitely do get recycled. Look at how spontaneous generation has become all the rage again.

Phil,

I'm pleased to see you acknowledge that the reanimation of your demigod was not even unique.

Of course there are lots more zombie stories in the bible, too.

Another Matt @ #300,

I think this "God antenna" does have an easily understood explanation. Not physical, but mental.

It is a challenge for those of us who have overly-analytical minds. Our minds are so good at figuring stuff out, that we conclude that all truths can be derived simply by thinking about things. But life does not actually work this way.

The distinction was brought home to me in the form of a quote from an old Bible college professor. He used to tell his class:

"Fellas, ya can't kiss yer girl and think about the kiss at the same time."

In other words, analysis (thinking) destroys a very real and good experience (the kiss). That's also why we all close our eyes when we kiss. You can think about the kiss or you can experience the kiss, but you cannot do both at the same time. In the same way you can think about God or you can hear from God, but you cannot do both at the same time.

As an Engineer, it was a new concept for me. I was used to analyzing everything. Now I had to turn off my analytical brain and just experience God. Not emotionally though, as you might assume. It is a lot like Eastern meditation, in fact. But instead of just focusing on nothing, the point is to clear your mind of your own analytical thoughts and to expect something to happen, for something else to fill the void.

I have learned to do this on a regular basis now. To allow in thoughts that are not really my own, and then not to destroy them by over-analyzing them. Of course I do consider whether they are good and useful thoughts, or silly ones, or actually bad. But it is amazing how consistently good and helpful thoughts come in this way.

So there really is no magic antenna. Just the ability to tune out my own analytic mind and to listen. And because I get the answers I need, I continue to know that I am in touch with my Creator.

I suspect that people with lower IQs simply have an easier time tuning out their analytic mind (mine runs on overdrive most of the time), and thus have an easier time "hearing" God. This is what makes it seem like they have a God antenna and you do not. But it is not hard once you know what the problem is. You just rely too much on your innate ability to analyze things, which is normally a good thing. But when it comes to hearing God, you have to turn that off and just listen. And then you have to learn not to doubt that you are really hearing from God! Because this is the other thing the analytic mind does on the back end.

We smart guys tend to suffer from the paralysis of analysis.

Oh, and that eternity of conscious torment? Funny enough, it is to be found nowhere in the entire Old Testament. You know, that part of the Bible where God is always angry at everyone? Hebrew has one word, and one word only to describe where the dead go: Sheol - the grave.

And then Jesus comes along and tells us that he has paid for ALL our mistakes and we should just love one another, but we're going to be damned to Hell if we don't. Ummmm, right. Makes sense to me!

It helps to have a theology that starts at the very beginning - a very good place to start.

Gordon,

It helps to have a theology that starts at the very beginning...</i

It helps to have empirical evidence to back up your random thoughts and those pesky voices in your head.

Without that, you cannot be sure that what you have is the effects of supernatural communications, rather than plain old everyday delusion.

Walt Jones @ #305,

The Bible is a compilation of sacred texts that from the beginning have claimed to be what people heard from God. From God walking in the Garden with Adam and Eve, to God handing the Law to Moses on Mount Sinai, to the prophets hearing the voice of God, to Jesus being the Son of God, to John telling us what he saw in Heaven.

That is the Bible from start to finish. An awful lot of people claiming to have heard directly from God himself, in one way or another. All we can then do is read what they said, and see if it makes sense to us. At first I accepted it because others recommended it to me. But the more I studied in, and practiced what it taught, the more I found that it really did work.

Just take a simple example. The core of the Law of Moses is the Ten Commandments. Everyone agrees that they are a good set of basic rules. Now look up the Code of Hammurabi, a similarly ancient set of laws.

What you will find is that Hammurabi had a thing about killing thieves. So ask yourself, which do you think is better, the death penalty for stealing, or "an eye for an eye"? Law #2 of Hammurabi involves jumping in the river to decide between accused and accuser. If they drown, they were guilty. Do you find that reasonable? Now you can argue about stoning someone for adultery, but find me a law of Moses that advocates superstition as a way of determining the truth.

Hammurabi says he heard from his gods also. Funny that I don't hear anyone promoting the Code of Hammurabi as being the Word of God. Over 3000 years later, most people still consider the Law of Moses to be fundamentally sound, even if we no longer follow many provisions of it.

I'm an Engineer. I like Instructions. I view God as the Designer/Builder and the Bible as the Instruction Manual. If you follow the instructions and they don't work, you pretty soon lose faith in the manual. But if you find that time and again the instructions you needed were there in the manual somewhere, well you keep referring back to it. And you accept the premise that it was in effect dictated by the Creator himself.

I hope that answers your question.

Michael @ #312,

Jerusalem/Palestine sits at the crossroads of three continents. It is about as central as you can get to anywhere on the planet, but especially to the "Old World", where the bulk of the people were and still are.

So long as you see this as a long term plan, it would be hard to choose a more perfect starting location.

Another Matt @ #316,

Yes, Matt. Jesus did not judge Thomas in any way. He made no implication that skepticism was a bad thing. What he said AFTER giving Thomas the certainty he wanted, was:

"Because you have seen Me, have you believed? Blessed are they who did not see, and yet believed."

Gordon,

Everyone agrees that they are a good set of basic rules.

No they don't. Lots of people believe in other gods or no gods. Blows out a two or three right there. I think you could come up with exceptions to most of the others as well - most of them highlighted in Bible stories.

Why not a Jesus on every continent? or have him preach for 30 years, in 10, 20 30 different places? No it makes your god out to be none to competent if its goal were to save the world and it is also omnipotent and omniscient.

Also the "manual" is obviously wrong on how God designed/built the universe. All you need do is accept that many, many things that appear designed actually came about without being designed/built by intelligence. I think eric mentioned the Giant's Causeway in Northern Ireland or the Devil's Postpile in California for geological formations. Then of course there is life - the ultimate in design without a designer.
You really do make yourself appear a buffoon by dismissing evolution.

By Michael Fugate (not verified) on 27 Feb 2015 #permalink

Gordon,

So why are you so sure those voices in your head - the ones you're "hearing from God" - are real?

Delusional illness is very common. It makes people believe in things that do not exist. It makes people hear voices from gods.

As I understand you, you have no empirical reason to believe your experience beyond your subjective convictions. As a Smart Guy, you surely must realize that's not enough. You must be aware that pathology can twist the mind and make you believe things that just are not true.

What makes you so sure you're not sick?

So why are you so sure those voices in your head – the ones you’re “hearing from God” – are real?

A close relative once claimed God told him not to treat his chronic liver disease because God would heal it. He finally sought treatment when he was near death, but I wonder if he regrets not doing it sooner (and I wonder if he feels guilty for not trusting God to heal him after all). Another relative of mine recently told me that God spoke to him and revealed that ISIS was going to kill his family if he didn't start taking measures to arm himself... he is already many thousands of dollars in credit card debt, but God says he should go buy guns so that when ISIS successfully overthrows the US government and comes after his family, he'll be ready. Can anyone prove he's wrong? How can we tell? One family of relatives asks God what to do before every decision they make, and then whatever comes into their mind immediately they claim to be from God (God promises he'll answer any prayer if you have enough faith). They lost their primary income and are likely to lose their house. Maybe God wants them destitute? How do we distinguish what they claim is God talking from the normal internal narrative?

PS -- I'm gonna say it: if you don't think about your kiss while you're kissing, you're likely a lousy kisser. You have to think to be conscientious and responsive.

By Another Matt (not verified) on 27 Feb 2015 #permalink

Michael Fugate,

"You really do make yourself appear a buffoon by dismissing evolution."

But how can you be sure that it is not just the appearance of buffoonery?

Our faith only offers miracles as occasional events, and these just serve to distinguish the natural from the supernatural. Your faith in evolution, on the extreme other hand, believes in countless billions of absurd miracles. Yours is a much more profound faith.

<blockquote<“You really do make yourself appear a buffoon by dismissing evolution.”

But how can you be sure that it is not just the appearance of buffoonery?

Because you keep repeating the same stupid assertions over and over and over. You don't know the first thing about either faith or evidence. Hell, you don't even know what a miracle is. Nothing in evolution requires a suspension of physics or chemistry. Even after hundreds of posts, you have gained not a single bit of understanding of why biologists think evolution explains organismal diversity.

By Michael Fugate (not verified) on 27 Feb 2015 #permalink

Michael Fugate,

“timetree.org will give you splits for almost any pair of taxa.
Their estimated time for the chimp/human split is 6.3 million years.”

I was really rather disappointed, though not surprised, with the responses about human scalp and facial hair. You never appealed to your theory, but you should have wondered why such a high-maintenance imposition would ever be favored by natural selection. You should have at least been arguing for a very late date coincidental with brain mutations commensurate with flint tools.

Thank you very much for obviating the created distinction between man and the dominion. I couldn’t have done a better job myself.
-
“Because you keep repeating the same stupid assertions over and over and over. You don’t know the first thing about either faith or evidence.”

But I do know about mountainous evidence about mutations because I read the literature. Nobody is having parades to celebrate the heart and core of your theory. Quite the opposite, the researchers and the labs are committed to combatting the expectable results of DNA replication damage. The champion of your theory is a loser.

The champion of your theory is a loser

Oh, there's a loser here, all right. Guess who, Phil?

Phhht @ #326,

If my delusion led to the end of 18 years of depression, you can take your "reality" and shove it up your ... . I'll take my delusion any day of the week.

I've had a few visits to psychiatrists in the past 18 years. Only one said something was wrong with me. He said I was "bi-polar".

I thought about it and said, "Oh yeah, that used to be called manic-depressive. I know what depression is, and I used to suffer from it quite severely. I haven't been depressed for years. You can observe me for as long as you want, and you will never see me depressed."

He responded, "Fine! Then you're Manic!"

Mania: excessive or unreasonable enthusiasm

Yep, that's me. Easily excited about lots of stuff. I love my life. It's a tough job, but somebody's gotta do it.

When's the last time you were given the "all clear"?

Another Matt @ #327,

I spent years around Christians who "heard" from God in the way that you describe, although the one who's afraid of ISIS is a bit over the top. That actually DOES sound like mental illness!

All I can say is that if it makes no sense to you, then they are unlikely to be hearing from God. When I hear from God it is always helpful or useful. I do not choose between my rational thoughts and what I hear from God. They work together. Sometimes if I am torn between options, God will cast light on why one is a better choice. Other times, the answer I was looking for simply drops into my head, and I know it was not the result of some rational thought process I had been engaged in. But I also know it is the answer because it makes complete sense.

I had to unlearn the prayer methods I was taught in church before I could really learn how to hear God. If you don't get the kiss analogy, you will simply not get this.

Ais a technical writer, I can tell you that's one poorly written instruction manual. As mentioned earlier, the first two chapters contradict each other (the assertion that the second chapter provides the details is contradicted by the different order of creation described), and clear statements like "I'll be back within a generation" require extensive explanation (which itself depends on knowing what the person is thinking), and halfway through says that a lot of the first half doesn't apply.

By Walt Jones (not verified) on 28 Feb 2015 #permalink

Phhht,

Stop arguing with Christians. They are not the enemy.

"And the last enemy to be defeated is Death."

And when that happens?

No more zombies, and God will be All in All.

By Gordon (not verified) on 28 Feb 2015 #permalink

In reply to by Walt Jones (not verified)

I am not that concerned the anti-evolution posters here reject evolution, but I would at least like them to understand what it is they are rejecting. After months of patiently and impatiently answering questions, there seems to be no gain in knowledge. This is depressing.

We squelch the complaints about new or beneficial mutations with no effective comeback and then weeks later the same whine is back again - as if it had never been asked before.
I understand that more evidence is not the answer and it is more the source of the explanation than the explanation that is the barrier. Maybe if a knowledgable and sympathetic Christian were to be the giver it might work, but would probably disintegrate into a doctrinal dispute over some hermeneutical or eschatological esoteria. As my old calculus teacher used to say when maundering about aimlessly while trying to solve a complex problem, "When you go in drain the swamp and you get up to your ass in alligators, you kind of forget what you went in there for." Such is the nature of evidence-free belief.

By Michael Fugate (not verified) on 28 Feb 2015 #permalink

Gordon says

All I can say is that if it makes no sense to you, then they are unlikely to be hearing from God.

What you say you hear from gods makes no sense to me, Gordon.

Gordon wrote in #332,

All I can say is that if it makes no sense to you, then they are unlikely to be hearing from God.

phhht replied in #335,

What you say you hear from gods makes no sense to me, Gordon.

Several of us have been telling Gordon this for a long time. He always blamed it on us.

But now he sings a new tune; now he finally tells us that he’s unlikely to be hearing from God.

So why on Earth would we believe anything this guy writes? I can’t think of a reason. What he says makes no sense to me.

sean s.

By sean samis (not verified) on 28 Feb 2015 #permalink

I have had conversations with others who have responded exactly like Gordon - if an idea pops into their head that solves some problem, then it is declared to have come from God. If it doesn't or if it is considered a "bad" thought, then it didn't. It is all based on what he thinks God is like and what God wants and is entirely based on if something works or doesn't. No wonder science seems baffling.

By Michael Fugate (not verified) on 28 Feb 2015 #permalink

The other thing is we all have thoughts pop into our heads - especially when we relax and let our minds drift. I am sure I have as many as Gordon does and some of mine are good ideas too helping me to solve problems. The difference is Gordon claims these ideas are put into his head by a god - they are derived externally where as I think mine are derived internally. None of this has any bearing on the existence of gods - as it is not evidence of anything other than how human brains work.

By Michael Fugate (not verified) on 28 Feb 2015 #permalink

Gordon tries to refute the charge of delusion, and gets it half right.

He blusters that he is NOT deluded, NOT, NOT!

He fails to give the necessary subsequent step: "...and here is the testable evidence which backs me up."

So nobody, certainly not Gordon himself, has reason to claim that the goddy voices in his head are anything more than yet another case of religious disorder.

Sean T,

"It seems likely that you’re just a troll"
Uh no, I'm here because I actually give a shit but thanks anyway! I like strengthening my brain and faith while encouraging others to have some - so there you have it.
Why imply that I struggle with reading BTW? Were the comments I typed (and you responded to) unreadable giving you the impression that I can't read?
Speaking of, I read somewhere that Charles Darwin was an ill, grief-stricken, anxiously paranoid, depressed agoraphobiac with insomnia who felt TORMENTED by his own thoughts. Nevertheless, you bought the contents of his mind hook, line and sinker while many others (like myself, See Noevo, etc.) reject them as something other than truth. That being said, I don't yield when I hear about rules or "templates" held by atheists and/or members of the scientific community. The "if we don't see it, we don't believe it" standard is of no interest to me when testifying about God, so your Ockham's razor is of little interest. Don't get me wrong, I'm glad you brought the principle to my attention because I'm not a scientist and had never heard of William of Ockham before you mentioned his contribution - but I discovered (despite him being a fellow Christian) that his principle was not without it's criticisms. So, like Darwin's theory, I'd assume it's less than perfect in getting to the ABSOLUTE truth.

Also, after he engaged with me, I was dealing with phhht on a personal level trying to appeal to him after reading his confession about loss of faith/lack of belief. I have an abundance, so I aimed to share despite the potential rejection or outcome. I wasn't thinking about rules or the scientific method at the time of my last 2 posts, but phhht's personal struggle.

But since you jumped in (you were of course welcomed to), I got a rule to add to yours that "simply states that we should not needlessly" reject the living God remaining open to the possibility that He's the sole Master Mind behind everything that we observe and call science. Gordon is one of the more astute Christians I've encountered and he's burned quite a bit of brain fuel/time talking about watches, piles of rocks vs intelligently designed walls, etc., to no avail. As he said, Jesus warned that some would reject God despite our efforts and I presume you fall under that umbrella. What would do it for you though? Signs, wonders, a series of answered prayers? A one on one with Jesus doing miracles on your back porch?

"Now, we as scientists are making the claim that none of our scientific theories requires the presence of a deity to work as an explanation."

I'm making the claim that just because an attempt has been made to explain something without God doesn't make that particular explanation fact. "None of your scientific theories needs God to work", but you do.

"Making sure my observations are repeatable so that any skeptics here can check my work and confirm my observations" is out of the question as even you'll acknowledge that most God stuff falls outside the purview of science. I, however, assert that He's not outside the spiritual purview of people. "No 'God revealed Himself to me', or 'It’s in the Bible' will count here. We need reproducible, objective evidence" and there you have it. I ask though; if God exists and people are capable of knowing Him, how could a person (including maybe you one day) ever know Him if He's not revealed personally or via someone else? There was this Jesus who had this IMPACTFUL life that witnesses wrote about and a few people embraced perpetuating His echo. If the accounts and experiences aren't recorded and shared, you wouldn't be reading my spiel because I wouldn't be a Christian at this present time. That's how it works, human beings are the conduit. I'm telling you there is a God, you don't believe me - but if and when you ever do discover this Person (in your physical life), it will likely involve a collaboration between Him and human vessels. Be careful to not discard every sincere believer's testimony because we have something of IMMENSE VALUE as you'll discover one way or another.

Michael,

"Jesus or at least those who wrote on Jesus provide food for thought, but the miracles and god on earth business is nonsense. It is time to move on."

He's a thorn in the side of your Godless worldview huh? Your tired of hearing about Him and you want God out of the picture ONCE AND FOR ALL!?! Too bad. Jesus and those who wrote on Jesus provided more than "food for thought" (what particular piece of bread got you thinking?), but they actually changed the world!!! Can that be disputed in any way that's worth a listen? There is no "moving on", truth never dies as there are major reasons why He stays relevant. While you're "moving on", go ahead and rewrite history and the way we measure it while you're at it. "The miracles and god on earth business is nonsense." Okay, thanks for letting us know! When are you going to tell the rest of the world your proof and blow this scandal wide open?

phhht,

"It’s highly revealing that you arrogate to yourself that superpower of ABSOLUTE CERTAINTY."

I can only tell you what I know and I can't undo my life or experience with God. To make you more comfortable, would you prefer I say I'm not certain when I actually am?

"No one can detect a god."

I can, and I'm not alone.

"So why should I conclude that gods are real, Lt? What makes you think they are?"

I "detect" Him.

"I see no reason whatsoever to think your gods will ever answer my prayers. As far as I can tell, gods have no tangible effects on reality at all. As far as I can tell, they do not exist."

As far as YOU can tell. I have my own set of eyes, ears, and mind - and as far as I can tell, God exists and I'm ABSOLUTELY SURE He has a tangible effect on ("my", if we must) reality. I'm not special though, so be on the lookout.

"I expect to go to my grave holding to that conclusion."

Let's hope not.

By Lt. Wassabewabee (not verified) on 28 Feb 2015 #permalink

"But Peter and the apostles answered, "We must obey God rather than men.
The God of our fathers raised Jesus whom you killed by hanging him on a tree.
God exalted him at his right hand as Leader and Savior, to give repentance to Israel and forgiveness of sins.
And we are witnesses to these things, and so is the Holy Spirit whom God has given to those who obey him."
When they heard this they were enraged and wanted to kill them.
But a Pharisee in the council named Gama'li-el, a teacher of the law, held in honor by all the people, stood up and ordered the men to be put outside for a while.
And he said to them, "Men of Israel, take care what you do with these men.
For before these days Theu'das arose, giving himself out to be somebody, and a number of men, about four hundred, joined him; but he was slain and all who followed him were dispersed and CAME TO NOTHING.
After him Judas the Galilean arose in the days of the census and drew away some of the people after him; he also perished, and all who followed him WERE SCATTERED.
So in the present case I tell you, keep away from these men and let them alone; for IF this plan or this undertaking is OF MEN, IT WILL FAIL;
BUT IF it is of God, you will NOT BE ABLE TO OVERTHROW THEM. You might even be found opposing God!"
So they took his advice, and when they had called in the apostles, they beat them and charged them not to speak in the name of Jesus, and let them go.
hen they left the presence of the council, rejoicing that they were counted worthy to suffer dishonor for the name.
And every day in the temple and at home they did not cease teaching and preaching Jesus as the Christ." [Acts 5]

And in 2,000 years, the Catholic Church has not been overthrown. And I bet my life it never will be (cf. Mat 16:18).

By See Noevo (not verified) on 28 Feb 2015 #permalink

Michael Fugate,

“We squelch the complaints about new or beneficial mutations with no effective comeback and then weeks later the same whine is back again – as if it had never been asked before.”

If by squelch, you mean that you effectively repel the argument, no you do not. The collective response is always the same thing…an accusation of unreasonable incredulity. But it isn’t at all unreasonable because the data is in on what mutations actually do.
-
“Such is the nature of evidence-free belief.”

That would be your beliefs. There is no evidence that countless billions of DNA replication failures produced things like metamorphosis, bio-sonar, bioluminescence, much less…anything else. I asked many time for you to sketch a mutations outline for how anything at all could happen, and you never offered anything. Because you can’t. Any yet you believe that this occurred not just once, but billions of times.

If you find it depressing that questions keep coming up about mutations, I’m afraid you are going to be terminally discouraged. Anybody whose curiosity has not been squelched by your religion is going to notice something is wrong.

Lt

phhht’s personal struggle

If you mean my atheism, Lt, it's no struggle. It's as clear as the nose on your face.

I notice that you do not say how gods differ from vampires, werewolves, superheroes, and other fictional characters. You cannot say HOW to detect a god - all you have are your empty claims that you can do so.

I notice that you do not specify what tangible effects your alleged gods have on reality. I notice you offer nothing to me to convince me that gods are real, beyond your testimony. I do not see how your belief that you have the superpower of infallible certainty marks you as anything but a loony.

You can bluster and fulminate forever, but that cuts no ice, LT. What you need is testable evidence. Otherwise, you're just one more victim of religious disorder.

Well phhht, @310 you said: "Well, Lt, I am not 'ABSOLUTELY SURE' in my conclusion (rather than belief) that there are no gods. I am only human, and I could well be mistaken."

That doesn't seem "as clear as the nose on my face." It does imply, however, that you may have doubts about your doubts.

By Lt. Wassabewabee (not verified) on 28 Feb 2015 #permalink

Lt,

Of course I am not ABSOLUTELY SURE. Only loonies and fanatics are ABSOLUTELY SURE.

I recognize that I am human, and may well be mistaken. Any sane person will concede as much.

But there is no struggle involved, Lt. I'm entirely comfortable with my conclusion that there are no gods.

Have you got any testable evidence to the contrary?

No, of course you do not.

“What you need is testable evidence.”

Fair enough.

“Deuteronomy 30:3 That then the Lord thy God will turn thy captivity, and have compassion upon thee, and will return and gather thee from all the nations, whither the Lord thy God hath scattered thee.”

About 3300 years elapsed before regathered Jews established modern political Israel in 1948.

So how is this not testable? I’m sure that you are publicly educated, and therefore ignorant. And there is a lot to this, but how do you manage what you know so far?

Phil,

You maintain that modern Israel constitutes testable evidence for - for something, I'm sure. For what?

How, then, can I test it?

phhht,

"How, then, can I test it?"

By comparing the ancient forecast with the modern results. It isn't really that hard,

"I notice that you do not say how gods differ from vampires, werewolves, superheroes, and other fictional characters."

phhht,

None of those guys have the legacy of creating you and every other good and natural thing of value in the universe. You don't owe any of those characters a "thanks for making me", but you do ("potentially", to tickle your ears) owe that to God.

By Lt. Wassabewabee (not verified) on 28 Feb 2015 #permalink

Phil,

Why don't you tell me what "the ancient forecast" is? Why don't you tell me how I can test it against "modern results"?

Isn't it because you cannot? Isn't it because your ideas are so incoherent, so muddled, so insane, that no rational treatment is possible?

Lt,

But no god created me and every other good and natural thing. That's just baseless bullshit.

Well don't be so definitive phhht, especially after stating: "I recognize that I am human, and may well be mistaken. Any sane person will concede as much" 7 comments ago.

By Lt. Wassabewabee (not verified) on 28 Feb 2015 #permalink

Lt,

So how do your gods differ from vampires? From zombies (remember, your demigod is supposed to be a reanimated corpse)? From superheroes?

They do not. All are fictional characters. None have any presence in reality.

If you disagree - and I notice you cannot defend your baseless contention that gods created everything good - then do so. Otherwise, you're just dodgin' and duckin' . You got nothing.

Michael @ #338,

You all have the ability to hear from God. You don't have to join some club first. You described this same thing in an earlier post. The only distinction is a small dose of humility, which you also mentioned in that early post, I recall.

Sometimes those thoughts that just drop into our head are so out of place that we know they did not come from us. We can either reject them, or consider that they came from an outside source and were given to help us. If you take the latter approach, you find that they become more frequent, more reliable, and more helpful.

If you reject them, or just assume that it is some subconscious act of your own mind, then they still happen. But you do nothing that improves this method of prayer - of seeking and receiving answers.

Like anything else, practice makes perfect.

Walt Jones @ #333,

Yes Walt, it is a bear of a book to figure out. There are a number of reasons for this.

The first, and overarching reason, is that mankind has, from the very beginning, not been all that keen to follow the message of the book. That is the fundamental problem.

Adam and Eve made a bad choice, and justified it to themselves and their children. All that could be done was to leave a record on what went wrong, just in case someone wanted to fix it in the future. But in general, no one is looking in the book seeking that answer.

Instead, we are left with the second problem, which is that a bunch of people tell us what the book means before we ever read it for ourselves. We tend to open the book with our head full of cultural presumptions from our own life, and especially from how others have already "interpreted" the book.

Third, we read a translation, which embeds the cultural presumptions of the translators. Fortunately, a Greek and Hebrew dictionary is sufficient to solve this problem in most cases.

Fourth, it is written in a style that differs markedly from how we write books today. As an aside, it is about as far from the style of a technical manual as you could possibly get.

Fifth, it is a big book, and thus hard to figure out where to start and what parts are more important. It took me decades to figure out that Genesis and Matthew really were the two most essential books, and it would be better to read no further until you have a good grasp of those two.

phhht,

“Why don’t you tell me what “the ancient forecast” is? Why don’t you tell me how I can test it against “modern results”?”

I pasted the early one above. There are many, many others. You can figure out how prophecies work if you put your mind to it.
-
“Isn’t it because your ideas are so incoherent, so muddled, so insane, that no rational treatment is possible?”

How does this work? Do you consciously decide to be obtuse, or is it some sort of reaction that you have no control over?

we are left with the second problem, which is that a bunch of people tell us what the book means before we ever read it for ourselves. We tend to open the book with our head full of cultural presumptions from our own life, and especially from how others have already “interpreted” the book.

You said you were Catholic. Scripture interpretation is one of the many functions of the Magisterium. There's some latitude for personal speculation, of course, but if you're talking about coming up with interpretations that have significantly different doctrinal implications than what is taught by your Church, you may have a problem.

By Another Matt (not verified) on 01 Mar 2015 #permalink

Phhht @ #372,

Phil is the wrong one to ask about Mary. It is us Catholics who you think are worshiping her as mother God. But we do no such thing.

It is obvious now that you have spent very little time in any church. Phil, were he in a nasty mood, could probably make the exact same taunt to us as you did. Protestants suffer from the same misunderstanding of Catholics, but at least they know that their own teaching on Mary does not remotely fit your accusations.

As a recent convert I was taught that Mary is the chief of the Saints. We pray to her for the same reason that we pray to any Saint, which is the same reason that we might ask another member of our church to pray for us. We ask Mary to pray for us. She is part of the "great cloud of witnesses" in heaven. Nothing more spooky than that.

Recent Popes have written extensively about the qualities of Mary that you are questioning. Why not read the answer from the horse's mouth?

By Gordon (not verified) on 01 Mar 2015 #permalink

In reply to by Another Matt (not verified)

Phil,
Do mutations occur? Yes.
Do some mutations increase fitness? Yes.
Does natural selection increase the frequency of beneficial mutations in the population? Yes.
Are mutations inherited? Yes.
Are mutations shared across generations? Yes.
Are mutations shared across populations? Yes.
Are mutations shared across species? Yes.

Which part(s) don't you agree with? And if not please explain using evidence why you don't agree.

Gordon, not one single reason I should conclude that ideas in my head come from another source.

By Michael Fugate (not verified) on 01 Mar 2015 #permalink

Phil,

Presumably your "prediction" from the bible is intended to constitute evidence for the existence of gods.

If that is what you think it means, then of course it is fallacious. It assumes what it is meant to demonstrate.

Gordon,

Stop condescending to atheists.

You stand naked in your religious disorder. You cannot defend your claims in any testable way; all you can do is to repeat them endlessly.

Free yourself of your delusions.

Michael Fugate,

“Do some mutations increase fitness? Yes”

Fitness is barely the issue. The sickle cell/malaria showcase does not equate with the type of replication errors that would be necessary to slowly build anything complex, and everything biological is extremely complex.

Once upon a time, any biological thing you want to notice, did not exist. Science didn’t discover that countless billions of random DNA replication failures are responsible for everything that exists now. This is just a sappy, desperate, materialist tenet.

===

phhht,

“Presumably your “prediction” from the bible is intended to constitute evidence for the existence of gods.”

No, it is evidence for a particular God, the one who had the prophecies recorded. They call them prophets for a reason. Authentic prophecies mean that someone capable of controlling human history is in control.
-
“If that is what you think it means, then of course it is fallacious. It assumes what it is meant to demonstrate.”

Nonsense. The scattering and regathering of the descendants of Abraham, Isaac and Jacob is described numerous places in the Bible. Isaac Newton wrote extensively about the books of Daniel and the Revelation. He pondered the (second) return to the promised land well over two hundred years before it actually occurred:

“Now while such a return from captivity was the expectation of Israel, even before the times of Daniel….the commandment to return and to build Jerusalem, precedes the Messiah the Prince 49 years; it may perhaps come forth not from the Jews themselves, but from some other kingdom friendly to them, and precede their return from captivity….the return from captivity and coming of the Messiah and his kingdom are described in Daniel 7, Rev. 19, Acts 1, Matt. 24, Joel 3, Ezek. 36, 37, Isa. 60, 62, 63, 65 and 66 and many other places of scripture. The manner I know not. Let time be the Interpreter.”

The prophecies were indisputably written in both Testaments many centuries ago. Claims of vagueness, coincidence or fraud cannot be seriously entertained. There is no rational place to hide from this.

As a side note, there are severe penalties associated with people and nations who are on the wrong side when it comes to the chosen people. There is a good chance that our president and his naive international dalliances, is unwittingly flirting with giant danger. This will be an interesting week.

Phil says
Authentic prophecies mean that someone capable of controlling human history is in control.

How can I test your outrageous claims to see if they are true?

Must I simply accept your baseless assertions and your crazy interpretations about an ancient book of legends, no matter how preposterous?

No, it is evidence for a particular God, the one who had the prophecies recorded. They call them prophets for a reason. Authentic prophecies mean that someone capable of controlling human history is in control.

So let's say a baseball player says "I think God will let us win tomorrow." Is he a prophet if they win? No, because you'd still have to show that their winning could be attributable to God instead of the team, and asserting it is just assuming what it is supposed to be evidence for. The Israel question is just like this -- a prediction of events can be correct even if it is for the wrong reasons. I can predict that the sun will rise tomorrow because a cosmic rooster is towing the earth in circles, but I can't use the sunrise as evidence for that rooster. It's a logical fallacy called "affirming the consequent." You can say "if there's smoke, there's fire," but you can't use an instance of fire on your stove as evidence that your kitchen is filled with smoke. Likewise even if "If God causes there to be a state of Israel, then there will be a state of Israel" is fine, you can't say anything about antecedent just because the consequent is fulfilled.

Or say a girl says to Jake, "God, please make Jake have to walk home with no clothes and a black eye." Is she a prophet if a bully who happens to like her overhears her say it, and so beats Jake up and takes his clothes? Is the fact that the bully carried out the "prophecy" an indication that it was a real prophecy, and that since she invoked God, that the prophecy's fulfillment is therefore attributable to God? No, of course not. For the prophecy about Israel to be a real prophecy, you'd have to have people who were unaware of it fulfilling it. If I take it upon myself to fulfill a prophecy, I'm treating it as a commandment or an entreaty, not a prophecy.

By Another Matt (not verified) on 01 Mar 2015 #permalink

And this all assumes that the Biblical Israel and the modern nation of Israel (or worse, the representatives of its government) have identical extensions. My whole family treats Netanyahu as a kind of super-pope -- anything he wants must be granted so that the US can't be seen as "standing against Israel." It's preposterous, and really dangerous.

By Another Matt (not verified) on 01 Mar 2015 #permalink

No answers Phil no explanations. How typical!

By Michael Fugate (not verified) on 01 Mar 2015 #permalink

"There is a good chance that our president and his naive international dalliances, is unwittingly flirting with giant danger. This will be an interesting week."

Wow - it is always scary when delusional people (like you, sn, gordon, and vs) comment on complicated issues. It would be interesting to know what your fermented mind spewed when President Bush sent the middle east into crap by going into iraq with no real plan and for no real reason.

What's funny too is sickle-cell anemia is much harder for creationists to explain. Either their god is an incompetent bumbler who can't come up with a malaria cure without enormous side effects or it is degeneration after the fall, where those who have degenerated the most, i.e. no malarial protection, are healthier than those degenerating less. I think the bumbling god is a better fit to the Bible, though. First off the crown of creation humans the one's most like god immediately disobey him and continue to do so until he tries to kill them all off. The one he chooses as the best turns out to be a drunk. Its like that all the way through - trial and error - not a hint of a long range plan anywhere in sight. Evolution or an idiot for a god - take your pick.

By Michael Fugate (not verified) on 01 Mar 2015 #permalink

Phil@363 says No, it is evidence for a particular God...>

Which god is that, Phil? I presume you mean a catholic god, and not one of the millions of others. Do you mean the father god? The demigod? The mother god? The ghost god?

And don't give us that pitiful nonsense about three, three, three gods in one. That's just silly, and anyway it entirely omits the mother god.

phhht,

“How can I test your outrageous claims to see if they are true?”

You’ll just have to watch.
-
“Must I simply accept your baseless assertions and your crazy interpretations about an ancient book of legends, no matter how preposterous?”

No, you can go back to sleep.

===

Another Matt,

“For the prophecy about Israel to be a real prophecy, you’d have to have people who were unaware of it fulfilling it.”

In view of history, starting with the birth of Zionism around 1870, that idea doesn’t hold up at all. When the UN General Assembly adopted resolution 181, I don’t think they had prophecies in mind. The restoration of Israel is
-
“And this all assumes that the Biblical Israel and the modern nation of Israel (or worse, the representatives of its government) have identical extensions.”

It isn’t about the political nation of Israel. It is about a four thousand year old covenant, a specific people, and specific geography. There is no rational reason for the world to be preoccupied with such a tiny piece of land, and so few people.

===

dean,

“it is always scary when delusional people (like you, sn, gordon, and vs) comment on complicated issues.”

There is a lot about this that warrants your fear, but hardly my comments. There are some really nasty reports coming out:
http://www.israelnationalnews.com/News/News.aspx/191966#.VPPJnzid0b0

Were I you, I’d be more concerned at having a CIC who would say that “The future must not belong to those who slander the prophet of Islam”. It makes you wonder exactly who he does think the future belongs to.

Phil,

So which God was it?

And I'm especially interested in the mother god. She's like Spiderman: she does what a deity can, including invisible presence, listening to thoughts, the answering of prayers, the bodily ascension to heaven, and - correct me if I'm wrong here, Phil - eternal virginity, even after having sex and giving birth.

That's pretty cool stuff, especially as I am sure you are eager to insist that Mary is not really a god at all. She just plays one on TV.

I've always wondered, Phil: How did the ghost god get his sperm in there? And why wasn't it the father god on the job, instead of getting cuckolded by one of his fellow divines?

I do feel fairly certain, as a matter of faith, that however they managed it, they stayed in the missionary position.

Try to link somewhere that isn't a religious nut job conspiracy site phil. Your unsupported crap is bad enough.

By the way, in case there are still people who think you have a shred of integrity, it is worthwhile to give the comment you reference in context.

"It is time to leave the call of violence and the politics of division behind. On so many issues, we face a choice between the promise of the future, or the prisons of the past. And we cannot afford to get it wrong. We must seize this moment. And America stands ready to work with all who are willing to embrace a better future.

The future must not belong to those who target Coptic Christians in Egypt — it must be claimed by those in Tahrir Square who chanted, "Muslims, Christians, we are one." The future must not belong to those who bully women — it must be shaped by girls who go to school, and those who stand for a world where our daughters can live their dreams just like our sons.

The future must not belong to those corrupt few who steal a country's resources — it must be won by the students and entrepreneurs, the workers and business owners who seek a broader prosperity for all people. Those are the women and men that America stands with; theirs is the vision we will support.

The future must not belong to those who slander the prophet of Islam. But to be credible, those who condemn that slander must also condemn the hate we see in the images of Jesus Christ that are desecrated, or churches that are destroyed, or the Holocaust that is denied.

Let us condemn incitement against Sufi Muslims and Shiite pilgrims. It's time to heed the words of Gandhi: "Intolerance is itself a form of violence and an obstacle to the growth of a true democratic spirit." Together, we must work towards a world where we are strengthened by our differences, and not defined by them. That is what America embodies, that's the vision we support."

Your dishonesty about history and science are bad, your foolish belief in prophecy laughable, but your bigotry is beyond despicable.

You really have no clue of the meaning of "reputable site". I shouldn't be surprised that your only sources of information are people as ignorant as you are.

No Dean. A statement like that, that precise phrasing, cannot come out of the mouth of a Christian. That was abnormal The context does not excuse it.

Phil,

Ready to give up?

I don't blame you. So much of that hogwash you profess to believe is so stupid, so ludicrous, that there is nothing to do but pretend it isn't there. All you can do is to pretend no one ever mentioned it, and hope nobody ever does again.

Still, I would like to know which god in your pantheon you think is in charge of the prophecy and history-steering department.

It's not Mary, is it?

Another Matt @ #359,

Correct. This is mostly a problem with Protestants, because they sit in church and are told what the Bible "means".

Until 50 years ago, Catholics were told what the Bible "says", because it was only read to them in Latin. But today you can hear it in your own language, and reach your own conclusions.

I have great respect for the consistency with which the Catholic church has preserved its understanding of the Bible for nearly 2000 years now. That is no small achievement. Most Protestant churches have already ditched the theology of their founders, after less than 500 years.

Because I came out of the Protestant tradition, I only know the prejudicial mindset that it fosters. I found myself having to overcome many ideas about what the Bible actually meant before I could simple read the words on the page for what they were.

Once I was freed of the Protestant bias, I thought I was standing all alone, in defiance of all Christian traditions. Imagine my surprise when I discovered that most of my understanding was consistent with the teaching of the Catholic church.

I do see room for improvement in explaining the doctrine. I am not proposing a complete change of the doctrine. As I see it, the fundamental truths are correct, but there is more detail to be communicated. Regarding Original Sin, that extra detail will answer the Problem of Evil.

But I do see it as in my power only to suggest what changes would be helpful, as it is indeed up to the Magisterium to make any official changes.

Brother Phil,

Dean continues to simply troll.

Phhht is just ranting like a mad man. That became obvious on post #351, when he asked you why you didn't tell him the "ancient forecast" ... that you had just told him on the very post that he was responding to!

He is clearly bordering on losing the plot altogether. Don't do anything that might push him over the edge. Telling a madman that there is no conspiracy against him merely confirms in his mind that you are part of the conspiracy. There is literally nothing you can say.

Another Matt brought home to me that we Christians really need to get our own house in order before we go telling non-believers that their logic doesn't stack up. I'm not sure if you read the article I linked to in post #251, but the author uses rock solid Catholic and Protestant doctrine to prove the Problem of Evil (by shattering the strongest defense). In other words, so long as we continue to describe Original Sin in the normal theological terms, we are in fact portraying a Creator who is not Benevolent, or else is not omnipotent or not omniscient.

We Christians should be having the discussion amongst ourselves on how to fix this. Otherwise we are talking out of both sides of our mouth. "Yes, that's what my church teaches, but it's not what the Bible really means", is not ever going to be a satisfactory answer, especially to a skeptic.

Gordon,

It is us Catholics who you think are worshiping her as mother God. But we do no such thing.

Let's see, you expect her to hear your prayers. You expect her to be invisibly present. You think she's immortal. You believe she has magical supernatural powers. You think she can work miracles. You think she had sex with your ghost god while remaining a virgin, even after giving birth. You think she herself was conceived without any sin, like sex, for example. You believe she ascended bodily into heaven, where she still lives, occasionally coming back here to earth to appear to you believers and work her magic.

Sure looks like god worship to me. Enlighten me, Gordon. Just what can gods do that Mary can't?

Phhht,

Let’s see, you expect her to hear your prayers (Yes). You expect her to be invisibly present (not exactly, she can hear us). You think she’s immortal (Yes, we all are). You believe she has magical supernatural powers (No). You think she can work miracles (No). You think she had sex with your ghost god (No) while remaining a virgin (Yes, see first half), even after giving birth (?). You think she herself was conceived without any sin (Yes), like sex, for example (No, sex is not a sin). You believe she ascended bodily into heaven, where she still lives (Yes), occasionally coming back here to earth to appear to you believers (Maybe) and work her magic (No).

An awful lot of wrong assumptions there. Mary hears us, and prays to God for us. God does all the "magic". Pretty simple, really.

By Gordon (not verified) on 02 Mar 2015 #permalink

In reply to by phhht (not verified)

and Gordon,

How many gods DO you believe in? One? Three? Four? Billions and billions?

pointing out that the sources phil presents are run by conspiracy theory pushers, liars, and worse, is stating a fact. The fact that phil thinks it is wrong to condemn racists and bigots is not trolling. Nor is it wrong to point out that he again is lying. Your lack of decency seems to rival his.

What is wrong in quoting the bulk of the words Phil is so intent on misrepresenting? His bigotry is a plus for his character now?

And why are you four so comfortable and handy with llies? I thought that was frowned upon in your cults.

Phil:

Some Biblical "prophecies":

Ezekiel 28:7-14 - Tyre will be destroyed by Nebuchadnezzar

Ezekiel 29:8-12 - Egypt will become a barren wasteland

Ezekiel 30:12 - The Nile will dry up

Isaiah 19:18 - The Nile will dry up and a sea bordering Egypt also will dry up

Ezekiel 28:24-26 - Israel will live in peace with its neighbors.

Just a few examples of Biblical prophecies that have FAILED. Even if the one example you give is conceded, shouldn't a divinely inspired book of prophecies have a batting average of 1.000? Clearly the Bible doesn't measure up.

sean samis,

To cause or allow disproportionate suffering, or suffering of innocent third parties is malevolence or indifference, either of which amount to evil.

Indifference to the suffering of the innocent may be immoral, at least in those cases, as I conceded in, I think, the same comment. But the argument is indeed over whether indifference in action to suffering is necessarily immoral. A malevolent entity is immoral/evil by definition, but we don't think that for indifferent beings necessarily, as seen in the examples I gave of people who could be considered to be at the very least indifferent in action towards suffering and yet aren't considered immoral.

I’ve always wondered:
If Heaven is a paradise, do people there have free will?
If not, then FW cannot be all that valuable.

If there is FW in Heaven, do people in heaven sin?
If ‘yes’, then sin must be part of the deity’s plan.
If ‘no’ then why don’t they sin? FW is supposed to cause sin, how can there be FW in Heaven without sin?

As I pointed out to eric, the people in heaven are not everyone. They are selected to get into heaven, and the Free Will Defense is indeed that usually the main reason we need it is so that we can freely make choices and freely develop our character so as to meet the selection criteria. Also, it is not that free will produces sin, but that to have free will you have to be able to choose to sin, and that therefore if you don't choose to sin it is because of your character, and because of the parts of your character that you are responsible for.

So people who don't sin in heaven don't sin, generally, because their developed character is such that they won't sin there. There are other alternative theological solutions, as I outlined in a comment. but the one above is the one that makes the most sense to me.

Answer: the missing things are knowledge and empathy.

Neither of these guarantee morality. Knowledge of what is good and what is bad is required, but even with that someone can still choose the bad/immoral, and all empathy does is get you to at most care about others, and you can do immoral things when you care about others. Your only defense here is to take a strong Utilitarian line and argue that it is all about the most good for the most people, which empathy MIGHT help you with -- the emotional part is really bad when it comes to people who are not like you -- but, again, there are many reasonable moral systems that deny that. So the answer's not as strong an answer as you imply.

I understand why humans test things; this is necessary for us to gain the information we need to understand things. We are only human.

Why would a deity test us? Only if they in fact do not know us. So all the claims about God knowing me, knowing my innermost secrets, etc. would be false. Is God only human?

If there is a God in any meaningful sense of the term, there is no reason for tests in the first place. He already knows what He’ll find. ‘Testing us’ does not explain or justify either evil or suffering.

Recently, I was going through some course books teaching IP concepts. Each chapter has a test at the end of it. No one was marking it and so the book wasn't giving it as a test so that it or the authors would know that I passed it. So what were those tests for? There are two reasons for them:

1) To prove to MYSELF what I knew and what I didn't, and how well I was doing.
2) To provide feedback on how I was doing so that I could know what I needed to focus on and what I was learning well.

Both of these are perfectly valid when talking about tests in this world that would no longer be required in heaven.

By Verbose Stoic (not verified) on 02 Mar 2015 #permalink

VS @246:

if people are trying to claim that the intent was for this world to be suffering-free then that seems to be an invalid move without extra support.

AFAIK most Christians accept that the pre-fall Eden was suffering free (if you’re a literalist) or intended to represent a suffering-free state (if you aren’t). The fall is the Christian mythological explanation for the existence of suffering and evil in our world. So you’re just plain wrong in your claim above; yes, in Christian theology, it is fairly clear from the text that the intent was for this world to be suffering-free.

No, as even you agreed it’s part of the definition of what heaven is: no one thinks that people in heaven do bad things. That’s what heaven means. You are trying to argue that another argument implies that people in heaven WOULD do bad things, which then causes a contradiction in the definition and so then causes an issue. You cannot then turn around and insist that I can’t assume that the definition of heaven is as it is

You can define heaven any way you want, my point is that if you fashion your definition to include an assertion you’re trying to convince me to accept, you’re not going to get anywhere because that’s apologetics, not rational argument. If I ask “how is it that all these evil people with free will don’t do evil in heaven” and you answer “well, that’s just part of the definition of heaven,” I’m sorry, I don’t see that as very responsive. Your answer appears to be to make a heaven a place only very very good people get into. That’s a fine answer that Gordon also seems to accept, but it is not mainstream Christianity at all. Its not the RCC’s faith + works and it’s not protestantism’s sola fides. Gordon, to his credit, at least admits that he holds a nonstandard view. You, in many of your arguments, seem to want to claim that you’re representing some standard view when (IMO) you aren’t. In mainstream Christianity, the main criteria for getting into heaven is acceptance of Jesus as lord and asking forgiveness. It is manifestly not merely or primarily being a good person.
The same basic ‘problem of definition’ holds for your response to theodicy. You can define God any way you want, and many conceptions/definitions will legitimately not have a theodicy problem. But if you start with a tri-omni God and then attempt to eliminate theodicy by defining away the problem of suffering, you aren’t going to convince any nonbeliever that you’ve solved it because defining it away is not what we consider a viable solution.

Gordon @322:

The core of the Law of Moses is the Ten Commandments. Everyone agrees that they are a good set of basic rules.

No, they are not. US law explicitly rejects commandments 1-5 and 10: rather than agreeing that punishment for these things is appropriate, our legal code enshrines the ability to do these things as a right. Our legal code agrees with the 10 C's on 6 and 8 (theft and murder). 7 (adultery) most people consider immoral but its mostly considered a non-criminal, civil issue today. 9 (false witness) we punish in some limited circumstances, such as when you're under oath in a criminal trial, but lying in general is protected under the first amendment's protection of free speech.

So, of the 10 things it talks about, the code would criimnalize 6 that we think people should have a right to do in a free society. It would criminalize 2 things we generally think should be handled outside of the legal system (lying and adultery), with some rare exceptions in the case of lying. The code gets 2 of 10 things right. That's not a very good record. 2 of 10 or even 4 of 10 if we're being charitable in interpretation is an F in school.

Gordon @355:

You all have the ability to hear from God. You don’t have to join some club first. You described this same thing in an earlier post. The only distinction is a small dose of humility, which you also mentioned in that early post, I recall.

Different people seem to be getting contradictory messages from God. So how do we tell the real ones from the fake ones?

Moreover, please drop the implication that no atheists have ever tried religion before (and if we just sincerely did, we would find that it worked). Most of us tried it for decades. Sincerely. And it didn't work. Instead of insisting we just need to be more humble and try your approach, assume we have tried your approach, it didtn't work, and tell us why. Were we No True Believers the entire decades we were believing? Surely you see the problem with claiming that!

VS @387

So people who don’t sin in heaven don’t sin, generally, because their developed character is such that they won’t sin there.

Everyone sins (according to church doctrine). There is no such thing as a non-Jesus human who free-will chooses to live a sinless life, nobody who develops the character you claim people do. We all fail. In fact, that is one of the pirmary tenets of Christianity: we all fail, and that is why Jesus' forgiveness is necessary. In Christian theology, nobody deserves heaven: nobody earns it, nobody passes the criteria. IMO your "developed character to make it to heaven" idea is an extremely non-Christian theology. Its more like the a UU or other fairly liberal pan-religious notion.

Lastly, again, you're going to have to explain how your notion is at all consistent with either sola fides or the RCC's belief + faith, because they don't appear to be. Sola fides explicitly rejects the notion that moral behavior has anythnig to do with it at all: sola fides says the one and only criteria for getting in is sincere faith in Jesus etc. A person with the moral character of Jeffrey Dahmer, Hitler, and Charles Manson could all get into heaven under sola fides, so long as they are/were sincere in their belief in Jesus. Martin Luther's writings on the Jews are a pretty good example. The guy was an absolute, raging anti-semite, but I don't know of anyone who would claim he wasn't sincere in his christian religious faith. So how exactly does your theology deal with him? Is he in or out? If he's in, what reasoning leads you to expect that a free-willed heaven-dwelling Martin Luther is not also a raging anti-semite?

Verbose Stoic wrote

Indifference to the suffering of the innocent may be immoral, ... we don’t think that for indifferent beings necessarily...

We disagree on this. “Indifference to the suffering of the innocent” is necessarily immoral, IMHO.

... the examples I gave of people who could be considered to be at the very least indifferent in action towards suffering and yet aren’t considered immoral.

I guess I missed your examples. Perhaps you could give me the comment numbers so I could look them up.

... it is not that free will produces sin, but that to have free will you have to be able to choose to sin, ...

You are correct in this: free will is merely an ability; the ability to make even a bad choice. The first problem is why are we so inclined to make bad choices?

The second problem is Whence sin? Did your God put it into the world? If your God hates sin, putting it into the world would be a malicious thing to do.

The third problem is, if your God actually hates sin and wants a heaven populated with sinless persons, why did He not just make sinless persons and fill heaven with them?

Why would your God create creatures whose fitness for Heaven is so uncertain? Intending to punish His creations for all eternity for flaws He put into them would be malicious.

Neither of these [knowledge and empathy] guarantee morality.

I’m not looking for guarantees, but for requirements. Your position seems to be that it’s about the “character” good people develop. What distinguishes this character if not empathy? How do good people develop this character without knowledge?

So what were those tests for? There are two reasons for them:
1) To prove to MYSELF what I knew and what I didn’t, and how well I was doing.
2) To provide feedback on how I was doing so that I could know what I needed to focus on and what I was learning well.

Your points 1) and 2) work only if

1) some expert evaluates your answers and provides a “score” and if
2) this expert score (the test result) is made known to you in time to learn from it.

If you took these tests but were never given the results until a final pass/fail decision is made then these tests would be useless unless you were permitted to retake the course. And even then, it would be a waste of effort to test students without sharing the scores immediately

In real life,

1) we must evaluate our “answers” ourselves (which we are incompetent to do) or,
2) we must pick some other person to be our expert (which we are incompetent to do).

Either way it’s the blind leading the blind. And

3) We don’t get to retake the course.

In life, by the time you find out how well you did, it’s too late. You can’t retake the course, and the harm to others is done.

Both of these [reasons for testing] are perfectly valid when talking about tests in this world that would no longer be required in heaven.

My question is not why they might not be required in heaven, but why they make sense in life.

Neither of your two points would justify the evil and suffering inflicted in this world by some deity. What is the purpose of “testing” a child with bone cancer? Or “giving” them a childhood in a ghetto? Those are the kinds of questions Steven Fry asked, and they are very valid questions. Your answers don’t suffice. In that you are not special; no one else has provided satisfactory answers either.

Consider some of the teenagers who have been in the news recently: those who have been lead to believe that their moral duty is to join ISIS. They have self-evaluated their experiences and concluded (with support from others whom they think “expert”) that joining ISIS is the morally correct thing to do.

How would they learn from this “test” in time?
How could God justify “testing” them this way?
How could God justify allowing their “tests” to include the deaths of others?

That makes zero sense to me.

sean s.

By sean samis (not verified) on 02 Mar 2015 #permalink

seam samis,

Let me start here:

I guess I missed your examples. Perhaps you could give me the comment numbers so I could look them up.

It's 189, but I'll requote it here:

The problem was obvious when you could talk about evil as a real thing in the world. But that’s not philosophically tenable any more. So the Problem of Evil translates to the Problem of Suffering. This means that you have to consider it morally wrong to not prevent the suffering of others if you can. But there are a lot of cases where we reject that. Few people think that someone is morally obligated to forgo going to the movies instead of giving that money to a homeless person to buy a meal. Peter Singer’s big push is to argue for us to take that idea on MORE, but even THEN it’s not that stringent. So it isn’t clear that you are morally obligated to relieve someone else’s suffering just because you can.

There’s also the issue of how God is differently situated than we are, which leads to the point I keep pushing at you and you keep side-stepping. Even if we concede that we are morally obligated to try to eliminate diseases like, say, smallpox, we are not in a position to be able to eliminate all of them any time soon. God, on the other hand, would be. So if we argue that God is morally obligated to eliminate smallpox, how do we then avoid concluding that God is not also morally obligated to eliminate measles and the common cold? If we argue that God is morally obligated to eliminate bone cancer and childhood leukemia, how do we then avoid concluding that God is not also morally obligated to eliminate leukemia, cancer … and ingrown toenails? Ultimately, it’s hard to see how these arguments avoid turning into arguments that this world should not have any suffering at all, but that’s not a tenable position in any Christian religion because of the explicit push that this world WILL have suffering, and it’s only heaven that won’t.

Even IF we can avoid that, we also have reason to think that one is not morally obligated to try to eliminate those diseases if one can, because someone going into medical research is not morally obligated to research cancer as opposed to, say, that disease where people sneeze when they go out into the sun, and someone can give money to the latter and not the former without being considered to be acting immorally.

Finally, to tie into something that I think Gordon was talking about, Goodness does not require God to be nice. If God has a reason for us to have to struggle in this world, even if He could achieve that end with less suffering, that doesn’t mean that God has to do it that way. To tie back to what I thought the explicit comment was, God can indeed decide that Justice trumps Mercy without impacting his overall Goodness. So that tangle needs to be unwound before we can really settle anything here.

Essentially, to make this argument work we need to know that God is acting immorally, and that requires us to know what is and isn’t moral. It turns out that the concepts of moral and benevolent are more complex than they look at first glance.

So, moving on:

We disagree on this. “Indifference to the suffering of the innocent” is necessarily immoral, IMHO.

So how do we settle this, given the above arguments?

You are correct in this: free will is merely an ability; the ability to make even a bad choice. The first problem is why are we so inclined to make bad choices?

Why shouldn't we be inclined to make bad choices -- ie to at least sometimes be tempted to be immoral -- in this world? If we were never tempted to do the bad, there'd be no moral praise for us for not doing the bad but instead doing the good. If all of my innate desires were only to do moral things, then I'm not a moral person if doing them; I am instead a person who simply does what I like to do. Moral fortitude comes from either conditioning what you want to be only what's moral, or resisting one's temptations to be immoral things and so doing the moral thing even when you would rather do the immoral, and so it is only moral considerations that stop you from doing that which is immoral. Ultimately, there must be an underlying moral component to being moral, something about it that is itself moral. At least part of that must come from you and your conscious choices for you to be truly moral. Your notion here denies that to us.

The second problem is Whence sin? Did your God put it into the world? If your God hates sin, putting it into the world would be a malicious thing to do.

Sin is not a thing in the world, so just like evil this objection goes away. You're really asking why God puts people in the world who can sin, which I've already taken a stab at. The same argument applies to your comment about heaven: God wants people who either consciously love him or who consciously act good, or both. Not just any sinners, but not just any non-sinners either.

I’m not looking for guarantees, but for requirements. Your position seems to be that it’s about the “character” good people develop. What distinguishes this character if not empathy? How do good people develop this character without knowledge?

I think that empathy has no actual real relation to morality at all, and think that relying on empathy is a terrible way to do morality. This is at least in part because I'm Stoic-leaning and distrust emotion, and that's all empathy gives. I don't want to help someone because I can feel their pain or because I want to alleviate it, but because it is the morally right thing to do, because it is what it means to be moral. If someone is suffering and it would be immoral of me to act to alleviate it, I firmly believe I should let them suffer. I won't like it, but I wouldn't like DYING or suffering MYSELF to be moral either ... and my fervent hope is that I WOULD be able to do that.

More on my rejection of empathy is here: https://verbosestoic.wordpress.com/2011/08/22/empathy-is-not-noble/

But let's move on from that for now:

Your points 1) and 2) work only if

1) some expert evaluates your answers and provides a “score” and if
2) this expert score (the test result) is made known to you in time to learn from it.

If you took these tests but were never given the results until a final pass/fail decision is made then these tests would be useless unless you were permitted to retake the course. And even then, it would be a waste of effort to test students without sharing the scores immediately

In my analogy, the answers were given in the back of the book. But there was still a lot of benefit in just seeing how easy it was for me to understand the questions and to answer them. The same thing applies here: seeing what the consequences are of our actions and working towards either a better understanding of morality or a better understanding of what God might want comes from living in a world where those decisions MATTER, and things happen. We might hurt people by doing wrong, but we can see that, work to make restitution, and work to improve ourselves to avoid that next time. Or, alternatively, we can harden our hearts to that, look at the pure practical benefit we get from it, and choose that over what we think is moral. Or personally decide that being moral is just doing what suits us best. At the end of the day, we'll end up where this all takes us ... and there probably isn't just one right answer either, or at least not a simple one. It's not necessarily about doing all good things, but about becoming the sort of person we want to be: one who tries to do good and/or not sin, or one who doesn't try at all, or one who revels in badness and/or sin.

(Yes, eric, that last part isn't commonly said and could be considered non-standard, but it's also a philosophical examination, Live with it).

Neither of your two points would justify the evil and suffering inflicted in this world by some deity. What is the purpose of “testing” a child with bone cancer? Or “giving” them a childhood in a ghetto?

Why do you think that that tests them and not us? Why do you think that this world is writing specific tests instead of a simple testing environment?

We have a world where bad things can happen to good people, to people who don't deserve it, but also to people who do deserve it, and everything in-between. How we react to that is important ... and under our control.

By Verbose Stoic (not verified) on 02 Mar 2015 #permalink

Ok Guys,

I am going to have one last crack at this before this thread closes. And when it does, I am done.

Eric, early on you basically asked for a world where Evil was impossible. But that is self-contradictory for the following reasons:

EVIL is when bad things happen, that cause us to suffer. This can be caused by other people, or by forces of nature.

Given that God is Benevolent, and so he does not want anyone to suffer. Therefore:

GOD'S WILL is the course of action that will lead to a good outcome, with no suffering. Anything contrary to God's Will, is therefore Evil, by definition. This definition is key.

What has been missed is that it is the very possibility of suffering that makes a concept like "Evil" meaningful. If it is impossible to suffer, as Eric seems to suggest it should be, then there is no such thing as Evil.

And if there is no such thing as Evil, then everything is God's Will. Therefore, there would be no such concept of "NOT God".

If there is no concept of NOT God, then we could not see ourselves as distinct from God. Yet we do.

This distinction we call FREE WILL. Free Will is the ability to think of ourselves separately from the mind of God. This is a gift of God.

It is this fact of this gift that then creates a scenario that seems to call into question God's Benevolence.

FREE WILL is the ability to make a choice contrary to God's Will. But everything contrary to God's Will is Evil. Therefore:

FREE WILL, in order to be meaningful at all, is the ability to choose Evil.

Any option that protects us from making a choice contrary to God's Will, eliminates Free Will.

Stated another way, any option that allows us to only make choices that avoid suffering, is within God's Will, therefore it eliminates Free Will.

The doctrine of Original Sin is explained in such a way as to imply that we do not actually have Free Will. It says that we have no choice but to Sin. If true, this would mean that we have no choice except to go contrary to God's Will, which is to commit Evil.

But this is clearly wrong by the above definitions, because clearly we have the option to make choices that avoid suffering. We make such choices (we do good) all the time (and irrespective of religious persuasion). We just don't make such choices EVERY time.

So the doctrine of Original Sin is poorly explained, because it compromises Free Will just as badly as Eric wants to with his objections.

What the Garden of Eden story shows is that Adam and Eve had complete Free Will. They were not forced to make the choice that led to suffering.

The story shows that God clearly warned them about the consequences of making the wrong choice. Some complain that he did not give them enough information, but we don't know the whole story. It records only that they were warned about the most serious consequence: Death. They may have been told more. But this warning was sufficient in itself.

Now, suffering serves an important purpose. It shows us why God's Will is good. This is circular:

Evil is everything that causes suffering. When we suffer, we discover why Evil is, in fact, a bad thing. If we did not suffer, Evil would not be Evil. While this should be self-evident, this tends to get lost in discussions on the Problem of Evil.

So, if we do not want to suffer, we should choose God's Will all the time.

But if we are forced to make that choice, then we do not have Free Will.

But to test our Free Will, and prove we really have it, we must try something that is not God's Will. When we do that, we will suffer. And in suffering, we will discover why God's Will is good.

God is Omniscient, so he knows the consequences of every choice. He used this knowledge to warn Adam and Eve. He later used this knowledge to give us all the laws in the Bible.

We are told what is Good (what will lead to no suffering), and yet we continue to make choices contrary to God's Will (his instructions as found in the Bible). This is the same as saying that we continue to make choices, of our own Free Will, that lead us to suffer.

By the explanation of God's Will given above, that it is all things that lead to no suffering, the very fact that we suffer proves that we are not following God's Will. This is true whether or not you believe that the Bible accurately explains to us how we can avoid suffering.

But what you find consistently in the Bible, is an end to suffering. The end of the story, as told by Moses, the Prophets, and the New Testament writers, is that "God shall wipe away every tear" and "The last enemy to be defeated is Death."

So the Bible addresses the Problem of Evil, and says that we will get it right eventually. There will come a day when we choose, of our own Free Will, to do God's Will - to choose only the good.

Some of our choices have direct consequences. When we break moral laws, we cause suffering directly. Clearly this is under our control, or we have no Free Will.

But the Garden of Eden story clearly shows that there were indirect consequences as well, that manifest themselves in nature. The "curse" that the ground would yield up thorns is one such indirect consequence. Again, not all the consequences are listed in the story. Just a very obvious and applicable one that drives the point home.

So, while we cannot prove that all suffering caused by forces of nature is indirectly a result of our choices, a good portion of it is, as stated in the story. What the story also shows is that, prior to the first bad choice by Adam and Eve, there was no suffering. So even suffering due to natural causes did not exist in the beginning.

What has been missed in the discussion so far is that without suffering we would not know the consequences of Evil. Indeed, without suffering there would be NO Evil. This circular definition is crucial to unraveling the full story on the Problem of Evil.

Because suffering and Evil do serve a useful purpose. It is an essential purpose. They are in fact, necessary to the entire construct.

With no Evil, there would be no Free Will. We would be indistinguishable from God, for we would simply do his Will all the time. We would be unable to conceive of ourselves as distinct from, or separate to, the Creator.

So the fact of Free Will, proven by the fact that we think of ourselves as separate to God, is the only answer to the Problem of Evil. In a world with no bad choices, we would be unable to tell the difference. We would lack individuality. We would lack personality.

In that sense, this world can be seen as a cosmic laboratory experiment. It is going rather badly at present. But we can fix it, because the answer is to be found in the choices that we make, and in the fact that God did indeed endow us with the power to make the correct choice, every time.

This last point is contradicted by the present doctrine on Original Sin. But the New Testament paints a different picture when it says things like, "Be ye perfect" and "Little children, I am writing to you that you Sin not."

And this is my bottom line on the Problem of Evil. The presence of Evil in our world is adequately explained in the Garden of Eden story. But unlike similar stories like Pandora's Box, it does not end there. The story goes on, and includes all the instructions that we need to fix the problem.

But Christians have missed this, due to the poor explanation of Original Sin. So we need to fix our theology first. And we will.

By Gordon (not verified) on 02 Mar 2015 #permalink

In reply to by Verbose Stoic (not verified)

Eric @ #391,

Interesting that VS and I have independently reached the exact same conclusions on Original Sin.

Yes, Christian doctrine teaches the opposite of what we are saying. You have accurately described the established doctrine of both Catholic and Protestant traditions. This is a big problem. We are heretics.

And yet, our understanding is both consistent with the Bible AND provides an answer to the Problem of Evil.

This is a double proof, for its adds up both by examining the text objectively AND it adds up logically.

Gordon,

You might have had me with your account of the POE except that the magnitude of the evil poses a problem for me. For instance, yes Adam and Eve sinned. Therefore, negative consequences ensued. Why, though was it necessary for a just and benevolent God to allow those negative consequences to have such a disproportionately huge impact, not only on Adam and Eve, but on all their descendants, the earth and possibly the entire universe?! Would it not have been more benevolent and just for God to just punish Adam and Eve in such a way that the negative consequences would befall only Adam and Eve? Most Christian theology posits that not only did human death arise because of the fall, but death in general. What did the poor animals do to deserve to die?

I realize that your beliefs conflict with orthodox Christian theology, but it still seems that God's punishment was a bit disproportionate. I presume God could have meted out his wrath in a more directed way. Why did He instead choose to punish indiscriminately?

eric,

AFAIK most Christians accept that the pre-fall Eden was suffering free (if you’re a literalist) or intended to represent a suffering-free state (if you aren’t). The fall is the Christian mythological explanation for the existence of suffering and evil in our world. So you’re just plain wrong in your claim above; yes, in Christian theology, it is fairly clear from the text that the intent was for this world to be suffering-free.

This world meaning the one that we are currently in, that is not the Garden of Eden, that is post-Fall? THIS world was meant to be suffering-free? Really?

No, it clearly wasn't. Whatever the causes for us to be in the world we're in now, from the start this world is stated as being one where there is in fact suffering, right back from Genesis, and that holds whether one takes that as literal or metaphorical. So, no, I'm not just plain wrong, and am in fact just plain right.

You, in many of your arguments, seem to want to claim that you’re representing some standard view when (IMO) you aren’t.

Which is, of course, why WAY back in 189 I DIDN'T make a long paragraph tracing through the various positions and how they might address your argument.

Oh, wait, I DID:

This started from you insisting that there was an issue with people having free will in heaven and yet not having there have to be evil. My argument was that people were still free to do bad things in heaven but that once there they wouldn’t. When you asked how that happened, I pointed out that the people who get into heaven are people who won’t do bad things. You argued that that doesn’t seem true theologically. At the end, though, what is true is that people in heaven will not do bad things and that there is indeed a criteria for entry. This ranges from they are moral people — ie they won’t deliberately act immorally — to they are people who won’t act immorally without the temptations of the flesh to they are people who believe in God and so won’t deliberately disobey his commands to they are people who believe in God and/or are moral and will be buttressed by God after having made the free choice to strive to meet the entrance criteria to their free will will be taken away because it’s no longer needed once they’ve been judged. All of these mean that in heaven there will be no need for more tests or growth, because the selection is already done, to whatever criteria you need.

Ultimately, at the abstract, general level, it is not the case that everyone gets into heaven, and that fact alone allows for multiple theological solutions to the issue that people in heaven might act according to their free will and yet not do bad things. So, no, not presenting my view as standard at all here, thanks.

You can define heaven any way you want, my point is that if you fashion your definition to include an assertion you’re trying to convince me to accept, you’re not going to get anywhere because that’s apologetics, not rational argument.

Except that as I pointed out YOUR argument relies on that common understanding -- well-represented in folk religion -- that people in heaven don't do bad things, because if someone accepted that they still would you wouldn't have an argument any more. Thus, I can rely on the thing that YOU accepted and defined in your argument to argue against you. You can't base your argument on a certain definition and then get all huffy when someone actually, you know, uses that definition in arguing against you.

Everyone sins (according to church doctrine). There is no such thing as a non-Jesus human who free-will chooses to live a sinless life, nobody who develops the character you claim people do. We all fail. In fact, that is one of the pirmary tenets of Christianity: we all fail, and that is why Jesus’ forgiveness is necessary.

Yes, we all fail. We all fail to always act according to what we know is right and what our character tells us is right. Now, given all the reasons why all of the things that might make us fail are likely to not be in heaven, why would we FAIL to live up to our character and our ideal character in heaven? Failing to be perfectly moral does not make one an immoral person or have an immoral character; failing to always live up to what God wants doesn't mean that one doesn't actually love God or strongly desire to follow it. That we are not perfectly strong and so sometimes give in to our temptations through weakness doesn't mean that we're bad people, and doesn't mean that we'd still fail if all of those temptations were removed. And there is every reason to think that those temptations are removed in heaven, through potentially various means.

A person with the moral character of Jeffrey Dahmer, Hitler, and Charles Manson could all get into heaven under sola fides, so long as they are/were sincere in their belief in Jesus. Martin Luther’s writings on the Jews are a pretty good example. The guy was an absolute, raging anti-semite, but I don’t know of anyone who would claim he wasn’t sincere in his christian religious faith. So how exactly does your theology deal with him? Is he in or out? If he’s in, what reasoning leads you to expect that a free-willed heaven-dwelling Martin Luther is not also a raging anti-semite?

Let's presume that being a raging anti-semite is wrong. Let's also make the rather safe presumption that being forgiven means legitimately understanding that what you did was wrong and genuinely regretting it. Luther, presumably, would come to understand that his anti-semitism was wrong and humbly and genuinely apologize and ask forgiveness from God for doing it. Now that he realizes that it's wrong in a full understanding, and doesn't have ingrained habits to fall back on -- there's no reason to think that in heaven God would allow for wrong ingrained habits to remain -- why would he act anti-semitically any more? What reason could he have to do that?

And before you talk about it not requiring forgiveness but only requiring belief, remember that YOU brought up forgiveness earlier, so I'm following on from your own argument, at which point if you want to make that sort of dodge I can quite reasonably demand that you decide what concepts of God, heaven, and Christianity -- at least -- you are aiming to refute before I bother to reply. This is on top of the fact that that wouldn't help you anyway; if Luther really has genuine faith and God tells him that being anti-semitic is wrong, Luther will presumably stop being anti-semitic because of his desire to do what God wants him to do, which is the entrance criteria for THOSE sets of theologies.

By Verbose Stoic (not verified) on 03 Mar 2015 #permalink

VS (to Sean):

Why shouldn’t we be inclined to make bad choices — ie to at least sometimes be tempted to be immoral — in this world? If we were never tempted to do the bad, there’d be no moral praise for us for not doing the bad but instead doing the good. If all of my innate desires were only to do moral things, then I’m not a moral person if doing them

Older women have far fewer violent tendencies than young men. In your system, do older women not go to heaven because they can't resist a temptation they generally don't feel? If the answer is "no, they can go to heaven too," then God could give young men the (non-)violent tendencies of older women and they could still get to heaven, correct? In which case, that's a better world God could have created, and its hard to see how we can call a being that chose not to create that world as omni-benevolent.

Ultimately, there must be an underlying moral component to being moral, something about it that is itself moral.

I don't see why a decision must have an underlying biological or psychological temptation before it can be a moral one. Whether to kill or not is still a normative decision when we have no strong passions one way or the other, right? Not all crimes are crimes of passion: we recognize that cold-blooded reasoned decisions to commit immoral acts are still normative decisions. The biological temptation is unnecessary. A God who sticks biological temptation into his creatures is therefore putting his thumb on the scales in favor of suffering, crime, and immorality.

[sean]What is the purpose of “testing” a child with bone cancer? Or “giving” them a childhood in a ghetto?

[VS] Why do you think that that tests them and not us?

Because punishing an innocent third party as a "test" for someone is not consistent with the notion of an omni-benevolent diety. Which brings us back to the fundamental theodicy problem: the claim of God's omni-benevolence is inconsistent with what we observe in the world and with other claims made about God. Like this one you make here, that God is going to test person A by inflicting a terrible disease on innocent person B. Claiming some kid's bone cancer is a test for their parents is not a solution to the theodicy problem, it is the theodicy problem.

Gordon:

Eric, early on you basically asked for a world where Evil was impossible.

No, I've been pretty consistent in pointing out that the amount of evil is the problem, not the existence of some philosophical lower limit. You keep missing or ignoring the point that it is trivially easy to imagine a world with far less suffering where humans still have free will, and this is inconsistent with the notion of God having the power, knowledge, and desire to give humans a pleasant life. Take all the Mansons of the world, and give them the nonviolent predispositions of MLK. Since we acknowledge MLK had free will, this cannot violate anyone's free will.

But what you find consistently in the Bible, is an end to suffering. The end of the story, as told by Moses, the Prophets, and the New Testament writers, is that “God shall wipe away every tear” and “The last enemy to be defeated is Death.”

So the Bible addresses the Problem of Evil, and says that we will get it right eventually.

For most nonbelievers and philosophers (of both the believing and nonbelieving stripe), that is not a sufficient answer to the problem of evil. Specifically, the notion that some people will be granted eternal bliss eventually is insufficent justification for the suffering God causes or allows to happen along the way.

But the Garden of Eden story clearly shows that there were indirect consequences as well, that manifest themselves in nature...So, while we cannot prove that all suffering caused by forces of nature is indirectly a result of our choices, a good portion of it is, as stated in the story.

I completely agree with your interpretation of the story. The problem is, a God who would set up the universe in such a way as to cause indiscriminate wide-ranging damage to the cosmos as a result of a single act of a pair of humans has done something evil or insane. It is not benevolent to fashion a tree that causes such indirect consequences when its fruit is eaten.

And yet, our understanding is both consistent with the Bible AND provides an answer to the Problem of Evil.

I've seen no adequate answer. "Free will" doesn't explain natural evils. VS' reply is that God doesn't view suffering as an evil the way most humans do. Which makes for a very odd definition of 'benevolent.' Do you share his view or do you have some other defense of natural evils? Because I haven't seen it if you do.

You have also never addressed the problem that 'free will' is no explanation whatsoever for the range of human predispositions for evil. We could all still have free will if we had the non-violent tendencies of the least violent human, so why doesn't God give us such tendencies en masse? Free will simply cannot be the answer to that.

You have thirdly never solved the problem of God committing or endorsing apparently evil acts, which has absolutely nothing to do with human free will. Such as harming an animal or person for a crime some other person commits. Yes, I get that he also rewards people for the good deeds others commit, but in my mind and I think in most people's minds, the latter does not make the former acceptable; it's still nonbenevolent. We would certainly not accept such behaviors in humans: if I torture a kitten, that act is not made benevolent if I then give a million dollars to my neighbor Bob for helping an old lady across the street.
So, it seems to me you haven't really given good answers to many of the key problems theodicy brings up.

VS:

This world meaning the one that we are currently in, that is not the Garden of Eden, that is post-Fall? THIS world was meant to be suffering-free? Really?

No, it clearly wasn’t.

Oh my god this is not that hard; why are you twisting everything? The first part of Genesis lays out the notion that God intended the original creation to be suffering-free. When Christians say "the intent of this world was for it to be suffering free," that is generally what they mean. That is also what I mean in the context of discussing bible-based theology.

YOUR argument relies on that common understanding — well-represented in folk religion — that people in heaven don’t do bad things, because if someone accepted that they still would you wouldn’t have an argument any more.

My argument is that heaven is a serious problem for folks who claim free will makes human evil a necessity, as these same folks posit a (future) human existence in which free-willed humans do no evil. Your response seems to be that heaven is populated only with those people who do no evil any more, having acquired that ability by necessarily living through this one. Your version of heaven actually makes God out to be less merciful, less benevolent than the standard Christian version, as fewer people get into your heaven. More souls burn for eternity in your system. This is not benevolent.

Now, given all the reasons why all of the things that might make us fail are likely to not be in heaven, why would we FAIL to live up to our character and our ideal character in heaven?

You cannot preserve people 'as they are' in heaven without them also carrying with them their limited capacity to understand others, their biases, etc... Which is a source of our evil conduct. If we still have those, we're going to do evil in heaven (or, at least, you have the burden of proof to say why we wouldn't). OTOH if you say God magics all our imperfections and biases away while preserving our essential free-willed character, then I'm going to again ask you why he just doesn't do that now.

Now that he realizes that it’s wrong in a full understanding, and doesn’t have ingrained habits to fall back on — there’s no reason to think that in heaven God would allow for wrong ingrained habits to remain

So, according to you our ingrained habits are not philosophically necessary. God, in his omnipotence, has the power to eliminate them any time he wants while still preserving our free-willed nature. He does so in heaven, but not here on Earth. You don't see this as a problem for your treatment of theodicy?

if Luther really has genuine faith and God tells him that being anti-semitic is wrong, Luther will presumably stop being anti-semitic because of his desire to do what God wants him to do,

If humans can be created with the nature that they resist evil by simply being told its wrong, then why aren't we that way here on earth?

Secondly, if God showing up in person to tell Luther it's wrong doesn't violate Luther's free will in heaven, why doesn't God just come down here and now and tell us wrong and right in person? It can't violate our free will (becasuse you just told me God could do it to heaven-residing Luther) and it would reduce the amount of evil and suffering by a great amount.

Eric,

You're going to love this. I am as miffed as you are at why VS can't see that you clearly understand the Garden of Eden story.

This world was created "Good" and therefore was created to be free of all suffering. You see this in the story. I see it. But VS does not. I don't know why.

Sure suffering goes all the way back to Genesis, as VS says. To chapter 3 specifically. Chapters 1 and 2 clearly portray a world in which there was no suffering, and never would have been without the story in chapter 3.

But onward to our main discussion:

You and Sean T raise the big bugbear as being the magnitude of the Evil we see in the world.

Evil is the messenger that tells us that ignoring God is the wrong thing to do. If we ignore that messenger, he shouts louder. And Louder. And LOUDER. Evil increases the longer we ignore God's Will.

You could liken this to an infection. If you treat it right away, it gets better quickly and the damage is minimal or not at all. But if you ignore it, the pain increases and the infection spreads. The increased pain is a message to tell you that this problem requires your full attention. But you can continue to ignore it if you want. Possibly at peril of life or limb.

You want a Benevolent God who lets you go around trashing the planet, so to speak, with him following along cleaning up the mess you make. You want him to make Evil not really Evil. You want to ignore his instructions of how to avoid Evil, and then have minimal consequences for committing the Evil, simply because he is such a nice guy.

I think this is about the point that Moses stepped in and started using words like "jealous" to communicate the fact that that dog just won't hunt. This is a mistaken understanding of Benevolence, and a lack of appreciation for just how badly wrong we have got things.

Once you understand that the fact of suffering is our responsibility, you will see that the Degree of suffering is also. The problem has gotten worse by virtue of our failure to adequately address it. And this applies especially to Christians, because they think they are obeying God's Will, when they are only getting part of it right, and not getting enough right to fix the original probem. If anything, they are more culpable than someone who denies that God exists.

I go back to my assertion that Free Will is really the ability to choose Evil. The more we make that choice, and the longer we make it, the greater the Evil becomes. And it does have indirect and unpredictable consequences. So we can look at someone's suffering and not be able to see exactly how it was caused by man's decisions. We then ask how a Benevolent God could let it happen.

But he allowed it to happen the moment he decided to allow man the ability to choose Evil. And without this choice, our Free Will disappears. With this choice, God actually gives up control over the magnitude of suffering, because he has placed no limits on our ability to choose Evil.

When I said that we will get this right eventually, I didn't mean that eternal bliss in Heaven makes all our suffering here worthwhile. What I meant is that we will eventually figure out that suffering is our own damn fault, and that we can eliminate it by choosing to obey God's will. This is a here-and-now proposition for this world, not the next.

You and I do tend to agree on the story itself. But the fruit part is just a metaphor. We are not being punished today because of a single mistake by Adam and Eve. We are suffering the consequences of having made the exact same mistake ourselves. And we do have the power to fix the mistake. This is where I depart from standard theology on Original Sin.

We continue to choose to act in a way contrary to the Will of God, which means that we continue to make choices that lead to the suffering of ourselves and others. Sometimes directly, but quite often indirectly.

For example, you might buy a shirt because you need one. Surely this is not Evil, right? But do you know how that shirt was made? What if you found out that it was made in a factory in China under conditions that are indistinguishable from slave labor? We know that such scenarios play out every day in our world. But we have no idea of the degree to which we are culpable in them simply by virtue of our shopping habits.

And remember, it is precisely the point at which we personalize the Evil, by acknowledging our own role in it, that we are then empowered to do something about it. This why it is so important to get the doctrine of Original Sin right. Were you or I born into a perfect world, we would screw it up the first opportunity we got! This is the problem we must address.

I explained that natural evils are a follow-on consequence of choices made by man. We see this when God says that the ground will now bring forth thorns and weeds, making work (growing food) harder. Man made a decision contrary to God's will, and this natural effect, a bad one, was the consequence.

We can take an example like cancer, which Fry blames God for, and point to the fact that much (most? all?) of it is caused by man's decisions. For example, we know that most lung cancer is caused by smoking. We simply do not understand enough yet to be sure what choices of ours (if any) are leading to increased cancer in children. But what we do see is an increase in cancer overall, and many studies suggest that this is due to certain habits that are now common in modern society. God didn't suddenly decide that we needed more cancer. We brought this on ourselves. This is especially inescapable if you deny God's existence, because then you have no one else to blame. So why blame God at all for something that seems obviously linked to various changes in societal habits?

An atheist especially should be asking, what decisions are we now making differently to before, that have led to an increased rate of cancer overall. Science should be able to give us an answer for this.

In fact, you have pushed me to this realization. It is precisely the point at which Mr. Fry has no answers that he invokes God. He is mystified by the occurrence of bone cancer in children. He is indignant about it. As an atheist he should be directing his indignation at modern society for bringing this on our children. But he does not. In the back of his mind he still blames God.

If he did not blame God, but placed the blame where it rightly belongs, at the foot of modern society, he would never come up with such a hypothetical question to ask a hypothetical God. Rather, he would ask this God he just met something like, "Bone cancer in children? What have we been doing wrong?"

Fry is no different to the rest of us. Exactly where our own understanding ends is where our belief in God takes over. But some of us are more up front about this than others. Christians with a blind faith in God can be excused for praying to God to heal the bone cancer in their child, and leaving it at that. Atheists have no excuse for not saying, "We brought this on ourselves, therefore we can fix it."

In short, Mr. Fry is revealed to be a complete hypocrite.

On the range of predispositions to do Evil, I really see no problem. This is implicit in Free Will. Some of make worse choices than others, solely because we can. The more you want this limited, just as you want the consequences of Evil limited, the further you depart from Free Will.

It should be pointed out that MLK, like so many others, was informed by a Christian morality which taught him among other things, to pray for his enemies rather than to fight them. He wasn't just born with some natural predisposition that was better than most. He was a preacher. He actually believed the stuff he preached. Funny that.

Your example of older women only applies to certain types of Evil. Mostly violent stuff. When it comes to gossip and judging, two of the greatest evils in the world, in my experience, older women are the absolute worst. My own dear Christian mother would not have to read a paragraph of your writing before declaring with her infallible judgment that you were bound straight for Hell. She would happily send you there, and that is Evil through and through.

It is fundamental (and correct) Christian teaching that we are all sinners. We are all responsible for causing suffering in our world. We may do it differently, and some may be predisposed to do a little bit more and others a little bit less, but none of us escape this. Not one of us can claim to have never caused another person to suffer. And none of us can state with any credibility that, from this day forward, we know how to ensure that we will never contribute to any further suffering in this world. The goal is a world where we stop doing stupid stuff to one another, not where we all just do a little less Evil than we do at present.

Finally, I'll use your example of Pharoah to address the issue of God committing or endorsing Evil acts. This must be seen in the wider context. Pharaoh was not innocent. He was causing the suffering of thousands, and possibly millions. It was in this context that we get a story of God's dramatic intervention.

We do see God "picking on" Pharaoh in the story. He hardens Pharaoh's heart so he can send further plagues. But this is merely consistent with Pharaoh's previous behavior. If you have spent years stealing from your company, but when you finally get caught you apologize and ask to be forgiven, is it unjust for your manager to say he will forgive you after the jury has found you guilty and sentenced you to prison?

You have this habit of separating out single acts of God from their context, and giving a blanket condemnation solely on the basis that God killed someone, and killing is wrong. This is not really a fair description of Evil, especially when people in general look to God to provide justice. The Bible tells many stories of God's justice, where people reap exactly what they have sown. And since they sowed Evil, they reaped Evil consequences. The story is told as if God directly visited those consequences on them, whereas other people, even atheists might simply have described it as karma or something similar.

So before you go calling an act of God, Evil, read up on what Evil acts men were doing that were connected to this. Because by your present rules, even if the story did not say that God brought the bad thing about, you could still object and say that God was not benevolent because he failed to intervene to prevent the bad thing from happening.

The heart of the story still comes down to the fact of man's choice to ignore God's Will. Everything flows from that one wrong choice, which we continue to make today. We choose Evil, which is precisely Evil because it brings suffering upon ourselves and others. God did not make this choice. And so the stories where God seems involved in Evil must be seen in their context. In no story are men simply sitting around minding their own business when suddenly - Blam! - God steps in and wrecks their day or their life. Sodom was guilty of great Evil, Pharoah was guilty of great Evil, the people before the Flood were guilty of great Evil, and so it goes in story after story.

You have to make a choice. Is the story about God endorsing an Evil act, or is it about God punishing Evil? Because these are opposites. Once again it seems that what you are really asking is for God to mitigate the consequences of Evil and not allow the full consequences to be brought to bear. This is a misunderstanding of his Benevolence.

By Gordon (not verified) on 03 Mar 2015 #permalink

In reply to by eric (not verified)

Verbose Stoic;

Thank you for your examples of “people who could be considered to be at the very least indifferent in action towards suffering and yet aren’t considered immoral.” It was quite illuminating.

I think the nugget is your reference to Gordon’s argument, and this: “God can indeed decide that Justice trumps Mercy without impacting his overall Goodness.

Ah. If you’ve bought-into the nonsense Gordon dispenses, then I am really, really disappointed. Gordon’s position is ultimately based on the assertion that his God (and yours?) exists and operates outside of any moral system. This deprives the concept of his God’s “overall Goodness” of any actual meaning. It’s as if we need to add an asterisk to the word ‘good’ whenever it refers to his God. The foot note at the bottom would say something to the effect that “This word does not mean what you think it means. In fact, it is entirely devoid of meaning

The phrase “God is Good” is then reduced to a formalism, to mere lip service. It is equivalent to “God is X” where the value of X is not knowable.

If the phrase “God is Good” has any actual meaning, then the word ‘good’ must mean what English speaking humans mean in ordinary speech when they use the word ‘good’. (References to God’s goodness in other languages should be treated similarly.) If the phrase “God is Good” has actual meaning, then most certainly if some God decides that Justice trumps Mercy that can severely undermine claims that God is Good.

Goodness does not require God to be nice.But it does require God to actually be good.

”If God has a reason for us to have to struggle in this world, even if He could achieve that end with less suffering, that doesn’t mean that God has to do it that way.If God could achieve His ends with less or no suffering, then God’s contrary choice negates any claim that He is good.

Unless, of course, your claim is that might makes right; that raw power is its own justification. I reject those; do you?

Regarding

Why shouldn’t we be inclined to make bad choices — ie to at least sometimes be tempted to be immoral ...

When I wrote that we are “inclined” to evil, I write of something more than mere “temptation”. I think history shows a preference towards evil sometimes in all and often in many. If the inclination toward evil is something God put in us, then He is responsible for its foreseeable effects.

Regarding

... there’d be no moral praise for us

Moral praise? That is unimportant; that appeals only to pride. If we all did what was right, there’d be no moral praise and vastly less suffering; that’s a trade I am fine with.

Regarding

If all of my innate desires were only to do moral things, ...

And whence these “innate desires”? What is the value/virtue/utility/etc. of desires to do evil?

More importantly, do people really “want to do evil” or do they just want to do things that gratify them somehow without concern for whether they are evil or not? Is indifference to evil what drives it? Or an actual desire to do evil because it is evil? I think it’s more about indifference to evil rather than any desire to do evil.

Taking something slightly out of order:

I think that empathy has no actual real relation to morality at all, ...

Empathy is simply the ability to understand the experiences of others, and to understand why one’s choices matter. If you object to the word ‘empathy’ then use ‘compassion’. I’ll stick with empathy for the time-being, if only for consistency.

Empathy/compassion are a particular kind of very valuable information about moral choices. When any valuable information is missing, culpability for a choice is reduced if not negated.

Regarding

... relying on empathy is a terrible way to do morality

I do not rely on empathy, I rely on the combination of information and empathy: knowing what your choice is about and caring about making the right choice.

Regarding

Sin is not a thing in the world, so just like evil this objection goes away.

This depends on what “sin” is. I conceive of sin as the reasons we do evil, principally weakness, ignorance, fear, selfishness, and indifference. This list may be non-standard, but this is what I’ve concluded, and these are actual things in the world; the objection survives.

Regarding, “.”

You’re really asking why God puts people in the world who can sin, ...

It is not for you to say what I am “really asking”. I am not asking about why there are people who “can sin” but why all of us seem to prefer to do evil, or are often unable to avoid evil acts.

Regarding

God wants people who either consciously love him or who consciously act good, or both.

Then why did your God make so many people who are unable to do what He wants? Is your God a poor craftsman?

How do people “consciously” choose to do something without information or the inclination to choose wisely? You have previously rejected knowledge and empathy as important, yet without them, we are demanded to make “conscious choices” without information and without the inclination to care.

The answer is that an uninformed choice about a matter where the import of the choice is hidden cannot be a conscious choice. You appear to be tripping over the same fact Gordon has: uninformed choices do not permit a rational attachment of responsibility or guilt to the uninformed choice. It may be foolish to act blindly, but if we have been blinded and forced to act, we are not guilty of our blind acts.

Regarding

I don’t want to help someone because I can feel their pain or because I want to alleviate it, but because it is the morally right thing to do, because it is what it means to be moral. If someone is suffering and it would be immoral of me to act to alleviate it, I firmly believe I should let them suffer.

This is interesting because it raises the question of when would it be moral to allow suffering?

In another thread I wrote

What is Evil?

Evil is any act which 1) causes harm, 2) is intentional and 3) is unnecessary.

Harm: any physical injury, financial loss, or impairment of liberty; or a substantial risk of any of these against the express consent of the one harmed or placed at risk.

Intentional: includes premeditation, recklessness, and unreasonable negligence.

Unnecessary: not justified by mitigation or prevention of other, greater harms or injustice nor justified by the consent of the one harmed.

This probably covers more than 80% of genuine evil, and is a good starting point. I do not claim it is complete. Suggestions will be considered.

So when, if ever, would it be moral to allow suffering? When the suffering is a necessary part of preventing greater harms to the person suffering.

Having surgery causes suffering, but if that is necessary to prevent permanent disablement or death, then inflicting the suffering of surgery and recovery are moral, especially with the consent of the patient.

Without some similar justification, failure to alleviate suffering is necessarily immoral.

This leaves unasked another question: when would it be moral to allow suffering of innocent persons or third parties? The answer is quite simple: NEVER.

Regarding

In my analogy, the answers were given in the back of the book.

Then your analogy is inappropriate because in life, we don’t get to look in the back of the book. We don’t know how well we did until it’s too late to do anything with that information.

Regarding

... seeing what the consequences are of our actions and working towards either a better understanding of morality or a better understanding of what God might want ...

This requires that we draw the correct lesson from our own suffering (which we are incompetent to do) and it assumes it’s morally acceptable to experiment on others with our bad choices.

Regarding

... living in a world where those decisions MATTER, ...

To appreciate this fact requires empathy or compassion, which you have rejected as having “no actual real relation to morality at all ”.

Regarding

We might hurt people by doing wrong, but we can see that, work to make restitution, and work to improve ourselves to avoid that next time.

Without empathy, we have no ability to care about harms we cause, and no reason to make restitution. And this still makes the learning of the sinner more important than the lives of their victims. This is a very strange morality you propose.

Regarding

Why do you think that that tests them and not us?

I don’t. Why do you think unnecessary “tests” justify the evil your God put into the world? Why do you think the these unnecessary tests justify suffering inflicted on innocents and third parties?

Regarding

Why do you think that this world is [blah blah blah] How we react to that is important ... and under our control.

All that may be true about the world, but at the end of the day, how does that not negate the idea that your God (who supposedly made this abattoir) is good? If anything deserving of the designation “deity” exists, none of what you write is either necessary or optimal; how you describe the world is one full of pointless evil and suffering which any deity who was actually good would not tolerate.

sean s.

By sean samis (not verified) on 03 Mar 2015 #permalink

Sean:

It’s as if we need to add an asterisk to the word ‘good’ whenever it refers to his God. The foot note at the bottom would say something to the effect that “This word does not mean what you think it means.

You'll need more than one. Its been my contention over a year or more that VS claims a rational* belief in a benevolent** God***.
*Rational = not directly contradicted by evidence, already in one's belief net. Note this makes beliefs in things like sasquatch count as rational.
**Benevolent = okay with human suffering
***God = more a deistic prime mover than Jesus type.

Empathy/compassion are a particular kind of very valuable information about moral choices. When any valuable information is missing, culpability for a choice is reduced if not negated.

Based on VS' @397 response, he thinks a complete understanding would allow a decent free-willed human to make good decisions all the time. That's why there's no bad acts in heaven - God gives us this understanding we may lack, and everyone there is decent. Its unclear, in this scenario, why a benevolent God would withold complete understanding from our mortal selves.

The problem with Gordon's answer is that there is no way to know if it is correct. Unlike science, it makes predictions that are not subject to verification. A claim that all will be "made right" at some unknown time in some unknown way is not satisfactory - especially when the additional claim of we can't really expect for this "making right" to be at all be consistent with human ideals of justice is made.

I can't see any reason to expect a correct answer from any religion(using that term broadly to include theism in general) and especially Judaism/Christianity. If the Bible is supposedly the word of the God, then it implies that this God is a fiction; it in no way could be true of a God with the characteristics that modern theologians believe it to have. The OT God is a human with a few extra powers - working in a trial and error fashion like human designers - bumbling its way through the whole mess. Are you telling me that Genesis 2 is true - that this God thought farm animals to be a suitable mate for Adam? That Adam knew who were more suitable mates than God did? This God even sends a messiah to save the Jews (his chosen people) and in one of the cosmic blunders of all time saves everyone but the Jews. Even after 2000 years of evangelism often times by torture, forced conversion, and every other means, it still hasn't made a bit of difference. But of course in Gordon's world this is the Jews' fault - not God's.

The best one can one can make of the OT is that it is a typical indigenous story; it tries to explain how to live sustainably in a specific place. In these cultures, science and religion are intertwined and inseparable. Their way of living within the ecosystem - often termed Traditional Ecological Knowledge and Wisdom (TEKW) - involves three components: knowledge, practice and belief. The practice of living in one place for millennia depends on science - local observational knowledge of an ecosystem allowing for prediction of future events - and religion - belief in a human role within an ecosystem.

The OT is full of tales that put a supernatural spin on observational knowledge in order to cajole, convince, control, or coerce people to do something which will keep them alive. Exodus has many - e.g. adding branches to bitter water to make it sweet, obtaining water by splitting rocks, harvesting manna, etc. The first key is observational knowledge - what additions to water will render unpotable water potable, where to find hidden springs and how to unblock them, what is edible and what is not. The second is convincing others to go along with you. A good story or song or dance will help get others on board and - remember - claiming you have the local deity backing you never hurts.

Every other indigenous religion has these same stories - overlaying supernatural causes onto observational knowledge. You can learn about ecology and morality from these stories, but Gods not so much.

I also assume my brain works in similar ways as most other humans - this is induction - look it up. I have been religious, have meditated, have spent days and weeks alone, don't clutter my mind with cell phones, iPods, and the like, walk, garden, and on and on. Plenty of time for a God - if there were one - to stop by and chat. Of course, ideas pop into my brain seemingly from nowhere, but voices from Gods - not so much.

By Michael Fugate (not verified) on 03 Mar 2015 #permalink

eric and sean samis,

I don't know if I'll have time to make a substantive response today, but for some issues, you need to check out the posts on my blog that I've referenced in this comment thread. For the latest set of "asterisks", I would appreciate if both of you would read the post link at 95, because it is a reply to you on that specific issue from that specific thread that is now closed, and we can continue the comment discussion over there if you'd like.

For sean samis, I would appreciate it if you would read the link on empathy I provided. It turns out that we both agree and disagree there, but that provides a context that is very important. One aside, though, is that I don't reject knowledge, merely empathy.

By Verbose Stoic (not verified) on 03 Mar 2015 #permalink

Sean S @ #401,

Why are you raising a straw man to argue against? This is just silly.

I have clearly distinguished Evil as being those things that cause suffering and God's Will (and Good) as being those things that do not lead to suffering. My entire recent explanation was in those terms only, with no need to even discuss the irrelevant side issue that you have now decided should be the only issue.

I have clearly explained why God cannot "achieve his ends with less or no suffering". See my last post to Eric, just above, for further detail on this.

I do not know where you get this "uninformed choice" notion from. It is the exact opposite of what the story says. It appears to be just another straw man argument.

I note that you continue to sling around the same silly assertions, and have made no effort to address a single point laid out in my latest post (#394) where I make a final effort to explain my position completely. Shoot holes in that, and we have something to discuss.

eric;

Regarding VS, you wrote

Its unclear, in this scenario, why a benevolent God would withold complete understanding from our mortal selves.

Well, I’ll let VS clarify VS’n opinions regarding this, but if VS agrees with your summation, then the question of why we are denied this understanding now is a pertinent question.

VS also wrote in this thread

That we are not perfectly strong and so sometimes give in to our temptations through weakness doesn’t mean that we’re bad people, and doesn’t mean that we’d still fail if all of those temptations were removed. And there is every reason to think that those temptations are removed in

So why would a benevolent deity not remove these temptations now?

Actually, even that’s no quite right. If this deity created us, these temptations were imposed on us by that deity. How could a benevolent deity justify that if, upon entrance into Heaven these temptations are removed? If these temptations are not evil, they belong in heaven, they are part of us. If these temptations cannot be allowed in heaven, then they must be evil, and these evils were created by the deity.

Regarding Martin Luther, VS wrote that

Now that [Martin Luther in heaven] realizes that [anti-Semitism] is wrong in a full understanding, and doesn’t have ingrained habits to fall back on — there’s no reason to think that in heaven God would allow for wrong ingrained habits to remain — why would he act anti-semitically any more?

If anti-Semitism is wrong (I fully agree.) then why does a benevolent deity allow it to exist in the shade of ignorance anywhere?

sean s.

By sean samis (not verified) on 03 Mar 2015 #permalink

Michael @ #403,

Do you have any control over your own future? If so, how much?

Do you believe that your choices actually determine your future? Put in modern terms, do you believe the Law of Attraction?

Because if you do believe that you determine your own future, then you can prove that my answer is correct - by choosing it for your own future!

Do you want a world without Evil? A world without suffering? then determine that you will not rest until you have achieved such a world. I have determined just that.

I started by only choosing to provide my children a better future than my parents had provided for me, but in putting this into practice I eventually discovered that I had the power to do so much more.

So the Bible becomes a self-fulfilling prophecy, because it gives us the very instructions we need in order to know how to create this future free of suffering, free of Evil, and even free of Death. Lofty goals, to be sure. But can you think of any other goals worth devoting your life to?

Didn't think it possible gordon, but with (assuming you are serious)

Do you believe the Law of Attraction?

you have just reached a higher level of stupid. There is no real world support for that bit of foolishness.

Gordon:

Evil is the messenger that tells us that ignoring God is the wrong thing to do. If we ignore that messenger, he shouts louder. And Louder. And LOUDER. Evil increases the longer we ignore God’s Will.

Its a nice thought, but that's inconsistent with both the Genesis story of the fall (maximal shouting at the first moment of disobedience) and with empiricism, which sees AFAIK no correlation between acceptance of the Christian God and good behavior.

Once you understand that the fact of suffering is our responsibility, you will see that the Degree of suffering is also.

No, it is not. I'm not the one who decided that as long as humans sin, puppies must die. God did that; God set up that system. Its on you to explain why that was philosophically necessary or benevolent (or both). You haven't.

I explained that natural evils are a follow-on consequence of choices made by man. We see this when God says that the ground will now bring forth thorns and weeds, making work (growing food) harder. Man made a decision contrary to God’s will, and this natural effect, a bad one, was the consequence.

Given that God set up the rules of nature to begin with, that he gave the tree the properties it had, that man's sinfullness would bring forth thorns etc., he's responsible for the system functioning that way. Look, we aren't talking about I push someone, they fall down here. There is no causal connection between me telling a woman she looks skinny in that dress and cats dying of old age. If you are going to claim that the latter is a necessary and natural consequence of the former, you'd better have more than just bare assertion to back it up. Because having the latter be a consequence of the former sure seems like the arbitrary whim of a malicious God to me.

We can take an example like cancer, which Fry blames God for, and point to the fact that much (most? all?) of it is caused by man’s decisions.

This is pure baloney. About a third of all humans get a potentially lethal cancer during their lifetime. The most common form (in the US) is breast cancer, followed by colorectal. These cancers are not strongly correlated with lifestyle choices. It would frankly be awesome if science could confirm such a correlation, because that would make for a very easy fix. Unfortunately, what you claim appears to be just flat out wrong. Cancer strikes people without regard for lifestyle choices, devout and satanic alike.

As an atheist he should be directing his indignation at modern society for bringing this on our children. But he does not. In the back of his mind he still blames God.

No, again, please stop with this. Its very clear he was answering a journalist's hypothetical question about what we would say if he found himself face to face with God. Giving an answer to that question does not imply he's angry with God.

Your example of older women only applies to certain types of Evil. Mostly violent stuff. When it comes to gossip and judging, two of the greatest evils in the world, in my experience, older women are the absolute worst.

Okay, fine...so why doesn't God reduce the mostly violent stuff by making men like women in that way and reduce the gossip you think is so evil by making women like men in that way? You haven't answered my main point at all: there is variation. This variation results in unnecessary amounts of evil behavior. Free will can't explain the need for God to give us this extensive variation, so why did he give it to us?

We do see God “picking on” Pharaoh in the story. He hardens Pharaoh’s heart so he can send further plagues. But this is merely consistent with Pharaoh’s previous behavior. If you have spent years stealing from your company, but when you finally get caught you apologize and ask to be forgiven, is it unjust for your manager to say he will forgive you after the jury has found you guilty and sentenced you to prison?

That is a terrible analogy. To make a much more accurate one: yes, it is immoral to coerce a known thief into a theft. That's analogous to what God did. It doesn't matter whether P would have chosen to act that same way, slipping him a divine mickey was wrong.

The Bible tells many stories of God’s justice, where people reap exactly what they have sown.

Those stories aren't the biggest problem. Its the stories of God punishing people who didn't commit the crime in question that are a problem. The midianite 1-year olds, for example. As well as the stories of God punishing people or creatures for things that don't seem to be crimes at all. Babel? Elisha's bears? Canaan cursed for seeing his dad naked? Jesus cursing a fig tree for having no figs? These are not benevolent acts.

The heart of the story still comes down to the fact of man’s choice to ignore God’s Will. Everything flows from that one wrong choice,

Yes I agree. But I think it is monstrous for a god to set up a system in which human's choice to ignore God's will causes cosmic entropy, death, destruction, etc... At its absolute worst, a moral system would be for humanity's choices to ignore God's will to cause those humans who choose that path to suffer, not everything to suffer.

Sean S,

I had never heard of the "Big Globe conspiracy" before, so I Googled it, and found that it was just how flat-earthers describe the collective brainwashing they find themselves up against.

I ended up watching a video put together by a flat-earther, where he goes through all the evidence for his belief, but mostly where he shows that the average man accepts a round globe without any demonstrable evidence whatsoever.

On the surface, that seems right. What verifiable evidence could the average man on the street pull out that would even suggest, much less prove that we inhabit a globe that orbits the Sun? I couldn't come up with any off the top of my head.

However, this was not what made an impression on me. What I realized, in thinking through the logic, was that the approach used to prove a Flat Earth was IDENTICAL to the approach employed here by the atheists when asserting their disbelief in God!

First, you both start by dismissing ALL the evidence that most men considered satisfactory for centuries. It is easily done. If some evidence is raised, it is immediately objected to as not really proving anything. You all have done this time and again, such as by denying all notions of Design, to the point that you sound more disconnected from reality than the Flat Earthers.

Second, you assert that your side has "testable empirical evidence" on your side. And you pull out a few things. It doesn't really matter what they are, because the examples used are always things that no one disputes. So now your side has "real" evidence, while the other side has none.

Third and finally, you call upon the other side to put some "testable empirical evidence" on the table. And no matter what is brought up, it is dismissed in one way or another as not impressing you at all.

In the case of atheism, scientific evidence is demanded as proof of the supernatural, when this is a contradiction in terms. Science observes nature. As of yet, no one has devised a system for observing the supernatural, and all proponents of the supernatural have claimed from time immemorial that the supernatural is in no way bound by the laws of nature. Yet based on this false demand that Science be applied outside its proper domain, all other evidence is rejected.

In the case of the Flat Earthers, they come back again and again to the simple fact that, from the perspective of the average man, ALL evidence that he can observe points to a conclusion that the earth is flat. So on this basis they simply reject all evidence that is not accessible to or testable by the average man, such as pictures from spacecraft.

Thanks for bringing up such a great example of how false beliefs are supported. I will, from now on, refer to all atheists as tireless opponents of the "Big God conspiracy"!

Now, for a serious and related scientific question. Given that we do indeed live on a Globe orbiting the Sun, and which has the Moon orbiting it, I would like to know if anyone has a reasonable explanation for the following "coincidence":

The Sun is approximately 400 times the size of the Moon. It "just so happens" that it is also about 400 times the distance from Earth that the Moon is. Thus, by sheer coincidence, the two most prominent celestial bodies in our sky, though radically different in scale to each other, appear from our perspective to be pretty much exactly the same size.

There seems to some intentional Design behind this coincidence. But I first want to see if there are any other explanations out there worth considering first.

By Gordon (not verified) on 04 Mar 2015 #permalink

In reply to by eric (not verified)

Sean @407 (and VS). Re: Luther, it's also worth pointing out that VS said this back in @35: "You don’t need moral growth in heaven because the people who get into heaven have succeeded, so you wouldn’t need any kind of moral dilemmas to work at."

Now, with the Luther thing I was asking VS about a hypothetical protestant heaven, not his conception of heaven, so in fairness there's no overt contradiction between this statement and our discussion of Luther. Maybe he's just not in VS's heaven. But there does seem to be a problem with saying 'no moral growth needed' on one hand, and saying that God can wipe away all our ingrained habits on the other. If the wiping away is possible and doesn't violate free will, then only letting people who don't need it into heaven seems excessively strict. If VS' heaven does allow in people who need a bit 'o wipe, then they are in fact undergoing moral growth in heaven, where there is no evil or sin, in which case we are back to the problem of evil here on earth being unnecessary because we obviously then don't need it for moral growth.

Gordon, been reading Norman Vincent Peale, have you? The same criticism applies - anecdotes without verification. And no I don't of any reason to think that - if you mean by the law of attraction - thoughts are energy and can cause mass to accelerate. That is pure unadulterated woo.

By Michael Fugate (not verified) on 03 Mar 2015 #permalink

eric;

Regarding VS, you wrote

Its unclear, in this scenario, why a benevolent God would withold complete understanding from our mortal selves.

Well, I’ll let VS clarify VS’n opinions regarding this, but if VS agrees with your summation, then the question of why we are denied this understanding now is a pertinent question.

VS also wrote in this thread

That we are not perfectly strong and so sometimes give in to our temptations through weakness doesn’t mean that we’re bad people, and doesn’t mean that we’d still fail if all of those temptations were removed. And there is every reason to think that those temptations are removed in

So why would a benevolent deity not remove these temptations now?

Actually, even that’s no quite right. If this deity created us, these temptations were imposed on us by that deity. How could a benevolent deity justify that if, upon entrance into Heaven these temptations are removed? If these temptations are not evil, they belong in heaven, they are part of us. If these temptations cannot be allowed in heaven, then they must be evil, and these evils were created by the deity.

Regarding Martin Luther, VS wrote that

Now that [Martin Luther in heaven] realizes that [anti-Semitism] is wrong in a full understanding, and doesn’t have ingrained habits to fall back on — there’s no reason to think that in heaven God would allow for wrong ingrained habits to remain — why would he act anti-semitically any more?

If anti-Semitism is wrong (I absolutely agree it is.) then why does a benevolent deity allow it to ever exist in the shade of ignorance?

Larry Moran commented that

The premise, as described above, is that the gods are omnibenevolant AND they want to create a world where there is no evil.

That’s not quite right. The premise is that gods are capable of creating an evil-free world. The problem is “why don’t they?”

See Noevo commented that

God is merciful, yes. But God is ALSO JUST.

The POE challenges not only God’s goodness, but also his justice. Denying humans the information and ability to avoid evil acts and then punishing them for those acts is unjust.

An FYI. I have something that came up suddenly, so I will not have time for these very good conversations for a week or more (depending on whether others complete their tasks as scheduled) so this will be my last for a bit. If this thread expires by my return, we’ll just have to pick this up on an a different thread, maybe some discussion of whether the Earth is flat (NO!) and the physics of that! And the Big-Globe conspiracy!

‘Til then, take care all.

sean s.

By sean samis (not verified) on 03 Mar 2015 #permalink

Eric @ #410,

How would a world look in which the consequences of Evil were limited? By what mechanism could they be limited?

Variation is implicit in Free Will. This should be obvious. So answer the above, since it does not seem obvious to you. I don't see anything reasonable in most of what you are still asserting.

Take a car as an example. I buy a car for my teenage son to get back and forth to school each day. It is a good thing, created for a good purpose, and I have given it to him for that and any other good purpose he cares to use it for.

How do I stop him from driving recklessly? How do I limit his desire to go too fast in the car? How do I control his desire to use the back seat to steal the virginity of young girls? How do I prevent him from killing anyone with it? How do I stop him from killing himself with it? How do I ensure that he never drives it into a crowd of people, possibly killing many all at once? How do I make sure it is not used as the getaway car for a bank robbery? How do I make sure it is never filled with explosives and parked outside a Govt building? How do I in any meaningful way limit the sheer magnitude of Evil that my son is now capable of simply because I gave him a car?

I already know your answer, so answer this instead: Even if I am the omnipotent Creator, how do I limit the magnitude of Evil that is possible? And your answer either needs to preserve Free Will, or assert that Free Will is a bad thing.

Regarding cancer, this is a straight statistical exercise. Don't be lazy here. You acknowledge that serious cancer will affect about one third of all people in their lifetime. Just pull up the cancer statistics from 100 years ago and before. The increase has been huge, even within the span of my own memory.

The standard explanation is that people are living longer now, and cancer is more likely to occur in old age. So pull up the statistics on childhood cancer and show me that the incidence in children has NOT changed over the last 100 years, and then you will have made your point.

But in the off chance you find that the incidence of cancer in children has in fact increased dramatically in our day, then you are indeed in denial and our dear Mr. Fry is indeed a hypocrite.

Strangely enough, all the statistics I can find on the subject start in the mid-1970's. I have been unable to find a 100 year trend for childhood cancer, which is curious.

http://curesearch.org/Incidence-Rates-Over-Time

Gordon,

I have pointed out to you in the past that the human brain is really good at seeing patterns in seemingly random data. I also mentioned that the human brain is also very good at finding patterns in truly random data. Your "coincidence" about the ratio of the size of the moon and that of the sun being the same as the ratio of the distances from earth to those bodies is just an example of the latter phenomenon. There is NO evidence for design here.

Consider, would you think it design if there were a similar ratio involved with moons of another planet? If life had evolved on Mars instead of earth, you probably would not even realize that this coincidence existed.

Finally, the nice thing about science vs. religion is that science allows for precise quantification of matters such as this. If your evidence that there is design is that the ratio of the moon's size to the sun's size is equal to the ratio of the moon's distance to the sun's distance is the same, then you either must simply give it up or postulate a VERY imprecise designer. According to Wikipedia, the radii of the moon and sun are 1787 and 696,342 km respectively. The average distances of the moon and sun from the earth are 384,399 and 149,600,000 km respectively. The radius ratio is 400.9. The distance ratio is 389.2. Your designer seems to have erred by about 3% if he really intended these ratios to be identical. Human designers can certainly do better than this, so I think it's much simpler to just chalk this one up to coincidence.

Sean T,

Yep, its all just part of the Big God conspiracy. Nothing to see here, folks. Just move along.

The fact that life is here on Earth, not Mars, is irrelevant.

The fact that the mathematics are not precise is crucial, and proves something, yet the fact that the Sun and Moon look equal in size to the naked eye means nothing at all. (In fact, the distances vary, so at times one is larger than the other and vice versa, as can be observed in the differing types of eclipses.)

But thanks for confirming that you have no explanation whatsoever for this remarkable coincidence, other than the required assertion that you are unable to see anything remarkable in it at all.

By Gordon (not verified) on 04 Mar 2015 #permalink

In reply to by Sean T (not verified)

Gordon:

In the case of atheism, scientific evidence is demanded as proof of the supernatural, when this is a contradiction in terms. Science observes nature. As of yet, no one has devised a system for observing the supernatural, and all proponents of the supernatural have claimed from time immemorial that the supernatural is in no way bound by the laws of nature.

Actually many preachers today claim there are observable miracles. It wasn't too long ago that people claimed they could weigh souls (they weigh 21 grams, evidently). The current sophisticated theological trend in claiming God is untestable is a historical and academic modern response to its dismal failure in testing, rather than some necessary and original property of the God hypothesis.

I also find the whole supernatural/natural distinction to be something of a semantic dodge. We can test faith healing and the medical efficacy of prayer. We can test the accuracy of revelation and divinely revealed knowledge. It doesn't matter what you label these things, they don't empirically work. Any hypothesis which would predict that they would work is empirically disconfirmed; it fails. Yes, you can retreat by modifying your hypothesis and claiming God would leave no statistical evidence of anything, but then you are essentially giving up on design as a scientific theory. You cannot insist on the one hand that we find no evidence because God intentionally leaves none, but insist on the other hand there is evidence for design.

In the case of the Flat Earthers, they come back again and again to the simple fact that, from the perspective of the average man, ALL evidence that he can observe points to a conclusion that the earth is flat.

You're partially right and partially wrong. Yes, modern science relies on a body of evidence that includes observations not directly accessible to the person on the street. Religious fundamentalists often favor a "Baconian" model of science, which gives more weight to direct observation and less to theories or indirect evidence. They do this because they want to reject theories of science that often incorporate indirect observations. Ken Ham's baloney about historical vs. observational science is an example of how creationists attempt to ignore or discount indirect observational methods and the concept of hypothesis testing.

But you're wrong in thinking there's no direct evidence of a round earth. Even in B.C. times, flat earthism was rejected by most people because they simply watched ships disappear down over the horizon. Also IIRC because of the annual cyclical motion of the stars. Eratosthenes of Cyrene measured the curvature around 240 BC, and while this was a notable achievement, no scholar at the time responded to it "omg Earth is round!!!" because they already accepted that it was, based on observations. It is something of a modern urban legend that people were flat earthers prior to the renaissance; flat earthism had been rejected by most people literally thousands of years earlier, with nothing more sophisticated than bronze age tools and observational methods.

By what mechanism could they be limited?

Alter the biology of the most violent members of the human race to be more like the biology of the least violent members of the human race.

I already know your answer, so answer this instead: Even if I am the omnipotent Creator, how do I limit the magnitude of Evil that is possible? And your answer either needs to preserve Free Will, or assert that Free Will is a bad thing.

You reduce the biological and neurological variation in the population so that everyone is born with the "good" tendencies currently only a few are born with. You cannot argue that this removes free will unless you are willing to claim that people born with good tendencies/without bad tendencies have no free will.

Let me use an example. Alcoholism can cause a lot of suffering. Some people are genetically born with susceptibility to it, but not all people. Remove that genetic susceptibility to it, and the world would contain less evil. Removing that genetic susceptibility cannot be argued to eliminate someone's free will given that the majority of humans (including, probably, you) aren't born with it in the first place.

So, why does God allow some people to be born with a genetic predisposition towards alcoholism when humans born without it have free will?

Just pull up the cancer statistics from 100 years ago and before. The increase has been huge, even within the span of my own memory.

Deaths attributed to cancer have gone up because (1) diagnosis has gotten better and (2) people die of less other things. In the 1800s, even among the well-off, the incidence of childhood mortality was something like 30% due to disease and other factors. Against that background it would've been simply impossible to even see a 1 in 10,000 rate of cancer (which is about what it is).

Which brings up another huge flaw in your argument. The childhood death rate due to "natural evil" has gone massively down in the last hundred years. Probably two to four orders of magnitude down, from on the order of 10% in the early 1900s to more like 0.01% in the early 2000s. This site note that the infant mortality rate (which is not considering children) went from 165 per 1,000 in 1900 to 7 per 1,000 in 1997. This is perfectly understandable under an atheistic or deistic framework: we cured diseases and developed better care methods, so the death rate went down, and there is no need to think 'sin' has anything to do with it one way or the other. But if you're claiming that our sin causes untimely deaths and that you expect there to be a large uptick in modern untimely deaths due to a large uptick in sinning, then the evidence directly contradicts your hypothesis: there are far less untimely deaths in western nations today than there ever have been in any other recorded place or time on earth.

Strangely enough, all the statistics I can find on the subject start in the mid-1970’s. I have been unable to find a 100 year trend for childhood cancer, which is curious

Its not strange or curious at all. Cancer often went unrecognized or undiagnosed prior to the mid-twentieth century. Even when they had the time and education to recoginze it - which they often didn't - nobody was concerned enough about childhood deaths by cancer to track it when the annual chilhood death toll from disease was something like a thousand times higher.

Eric,

Back in the infancy of Science, there is no telling how people thought it applied. Today, we are without excuse.

The very existence of the terms "natural" and "supernatural" is because people long before you and I saw that there was an important distinction. You just want to muddy the water for the sake of continued argument, and so you refer to clear use of language, and precise application of the scientific method as "a semantic dodge". Go argue with Merriam-Webster, not with me.

Your proposal for limiting Evil presumes that these predispositions are rooted in genetics, rather than in environmental factors. Yet this very subject is hotly debated. There is evidence for both sides, but not enough to prove either one with certainty. Personally, I find environment to be a much more reliable predictor of a person's predispositions.

And if I am right and you are wrong on this, then your whole premise falls over.

You go all fuzzy on the cancer statistics as well. Even the chart I linked to did clearly show an increase since 1975. I suspect it would be far more obvious if we could go back further. I suggested 1915 (100 years), and then you throw in some unsubstantiated statistic from "the 1800's" to try to avoid admitting that the scientifically demonstrable facts are against you.

What remarkable coincidence? The bible was written long after the moon was in the sky causing eclipses. There is little "coincidence" at all in the authors incorporating spectacular astronomical events they already had knowledge of into their theology. They also incorporated rainbows and floods. Is it a remarkable coincidence that these things occur and that the bible mentions them?

This is like claiming that it is a remarkable coincidence how JK Rowling mentions Harry Potter visiting a city called "London" in a place called "England", and wow, there is actually a city called "London" in a place called "England." Only God could explain such a remarkable coincidence!

The very existence of the terms “natural” and “supernatural” is because people long before you and I saw that there was an important distinction. You just want to muddy the water for the sake of continued argument, and so you refer to clear use of language, and precise application of the scientific method as “a semantic dodge”

Science has and does study things like the effect of prayer on healing rates. Science has and does study things like ghosts, souls, telepathy, telekinesis, out of body experiences, and so on. They all fail to pan out. They're irreproducible. Call them whatever label you like, it is empirical and historical fact that (a) proponents of these things have, in the past, agreed that science can study them and in fact used science themselves, (b) science has studied these things, and (c) they have all failed. Faith healing doesn't statistically or epidemiologically work. Neither does prayer. No form of revelation or out of body experince can reproducibly yield information the subject didn't already know. The barn door is open and the horse has left the building; its a bit late now to proclaim that it is not philosophically possible to open the barn.

Your proposal for limiting Evil presumes that these predispositions are rooted in genetics, rather than in environmental factors.

Yep, it does depend on some predispositions being genetic. I don't think that's a presumption at this point, I think it's pretty well confirmed fact. Male:female differences in violence have to do with biological development and testosterone levels. Hormonal balances matter to behavior, imbalances often contribute to bad behavior, and genetics affect hormonal balances. Your blood sugar and oxytoscin levels are going to affect how nice or nasty you are, and different people have different levels in part for genetic reasons.

If your answer to the POE depends on their not being any genetic predispositions that contribute to sinful or bad behavior, I think you're in deep trouble. IMO any solution to the problem of evil has to address the fact that god allows humans to be born with genetic predispostions toward bad behavior that we know are not philosophically necessary, because not everyone has them.

Speaking of coincidences, isn't it telling that the two things that Christianity demands it followers to emulate are sheep - an animal which retains absolutely no native intelligence and is easily subjected to herding, shearing, milking, removing its babies, and slaughtering - and children. This projects the message loud and clear that Christianity wants no independent thought and only passive obedience - where do I sign up? I can't believe I ever left - those Bishop's crooks are so subtle....

The existence of a God is often drilled into us by our parents from day 1 - we don't come to God through "evidence" of design in nature, but we come to "design" because we believe God is an agent. The God belief comes first and then we "see" the supposed "evidence" of its agency. Of course we really do see design in nature. We see it because natural processes produce regular patterns we can detect - natural selection is good at just those things.

When we see Jesus in a pancake does that mean God put it there to proclaim his glory - or is it just the propensity of humans to recognize faces? The things you have never answered Gordon is what actually is designed - is the Giant's Causeway in Northern Ireland or the Devil's Postpile in California designed by your God or are they the result of natural processes of chemistry and physics? Do they look like walls and pillars and stepping stones because they were designed by intelligence? What about the weather - meteorological patterns - God or not? Then we come to biology - natural processes can account for the diversity of life on earth - this is the beauty of Darwin's idea.

By Michael Fugate (not verified) on 04 Mar 2015 #permalink

eric,

Oh my god this is not that hard; why are you twisting everything? The first part of Genesis lays out the notion that God intended the original creation to be suffering-free. When Christians say “the intent of this world was for it to be suffering free,” that is generally what they mean. That is also what I mean in the context of discussing bible-based theology.

I'd reply that it's YOU that's twisting everything, except that this just seems to be the result of your amazing ability to not get the context of a discussion/argument, despite clearly being smart enough to do so, and in this case taking Gordon with you. So let me spell it out for you. Here's the abstract, generalized discussion of what Genesis says here:

Once upon a time, God created a world presumably without suffering in it, and then placed humans in it. However, that became unworkable for some reason and so God tossed humans out of that world into this one, a world which he explicitly promised would contain suffering. Thus, we are in this world that contains suffering.

So, taking that as our starting point, my argument is that if the implication of your argument is that this world can't contain suffering, then you have something that's massively suspect theologically since the starting premise theologically is that this world will have suffering and there are theological reasons given for why that has to be. You can make a philosophical argument for it, but that's also hard to do for all of the reasons that I've given about suffering before.

Also, this summary rather neatly answers your argument about why we can't have the things that get rid of suffering here if we can have them in heaven. Heaven is the return to that paradise without suffering, for those who somehow "earn" it (and, again, I listed a wide range of theological positions on that that you continue to completely ignore). God never said that the Garden of Eden would be unworkable for those who meet that criteria, but that it was unworkable in general. So why does this world have suffering if heaven doesn't need it? Because this world is unworkable without suffering, for again a wide range of reasons.

There are LOTS of philosophical and theological issues to work through here, but you aren't even off the ground floor as far as I can see.

My argument is that heaven is a serious problem for folks who claim free will makes human evil a necessity, as these same folks posit a (future) human existence in which free-willed humans do no evil.

Which is presumably heaven, yes, and so it is indeed true that you are assuming that heaven, by definition, includes people with free will and yet who don't do evil. So, conceding that your griping at me about using that as a definition is not my simply defining out a problem, at least on THAT score?

Let's put aside my specific views, because I haven't actually outlined them enough for you to criticize them as you are and we aren't even past the generic concepts yet that you seemed to be starting with and then dropped and then started with and then dropped and then ...

Plus, doing this would just get us to a point where you'd say that my view isn't standard and thus I shouldn't be talking about it as if it was, even if I don't, which is another rabbit hole that I don't want to go down.

You cannot preserve people ‘as they are’ in heaven without them also carrying with them their limited capacity to understand others, their biases, etc… Which is a source of our evil conduct. If we still have those, we’re going to do evil in heaven (or, at least, you have the burden of proof to say why we wouldn’t).

How much of our sinning comes from the demands of our physical body? Or from competing demands and limited resources? We probably won't have any of these in heaven (the latter for certain, the former probably depends on your theology). So a lot of the reasons for GOOD people to do evil are gone. Also, you'd know what God wants for you, so people who genuinely love God would definitely not make mistakes.

But, at any rate, you aren't actually answering the charge, which is that we fail because we aren't perfect and don't have perfect will. It is definitely in no way problematic to suggest that in heaven not having perfect will will not be a problem, or else you have the burden of proof to show that to be who we are we have to be imperfect. There are reasons to think that being imperfect are part and parcel of this world, but not of heaven.

So, according to you our ingrained habits are not philosophically necessary.

Ingrained habits are not part of our identity; we can change them and still be who we are. However, they are the results of who we are and our conscious decisions, and so would reflect on any judgement that God might give . Once God has judged us appropriately, then they wouldn't matter anymore, and so can be changed to reflect what we should be. Obviously, though, before judgement that can't be done because our ingrained habits and those we maintain and those we consciously break are important reflections of who we are up until that point. But, again, after judgement, in Christianity, they aren't needed anymore.

If humans can be created with the nature that they resist evil by simply being told its wrong, then why aren’t we that way here on earth?

The comment here is that Luther is misinterpreting what God wants. When he gets told that directly from God Himself, he won't do that anymore. Could we just have God tell us directly without issues? Theologically, this is actually very complex, but the short answer is "Probably not", because that's something that we do have to decide, for the most part, for ourselves because it is part of the development of our characters. Again, once judged, not important anymore.

Older women have far fewer violent tendencies than young men. In your system, do older women not go to heaven because they can’t resist a temptation they generally don’t feel?

And speaking of twisting ...

The point I'm making is that if we were created to never want to do anything that happens to be wrong, then it is not an indication of our moral character that we resist doing them. It's only if we WANT to take an action that we know is wrong that we can show that we prefer being moral to doing what we would otherwise want to do, or in short that we want to be moral more than we want to do that thing that we really do want to do. That's all I'm saying there: do you disagree? If you don't, then asking why God can't make it so that we are never tempted to do things that are morally wrong is to eliminate our moral character, which is clearly not a good thing if we are to actually DEVELOP a moral character.

You also misunderstand these tendencies. First, there are more tendencies that can lead to immorality than just violence, and for the most part everyone gets their own sets. Evolution and the roles that we have biologically play a role in that as well. Second, these tendencies aren't necessarily immoral. Violent tendencies can be used for bad ends or for good ones, such as to defend someone from someone else's immorality, even if that person isn't using violence themselves. As the Stoics put so well, those tendencies don't have any inherent moral value, so treating them as inherently good or inherently bad is a problem. But, then, this is one of the issues that the "The only criteria for morality is suffering" runs into.

I don’t see why a decision must have an underlying biological or psychological temptation before it can be a moral one. Whether to kill or not is still a normative decision when we have no strong passions one way or the other, right?

Who said anything about strong passions? Presumably, a cold desire to kill is still a desire to kill; even if just in that instant, you want to kill that person for some reason. It is not at all an indication of your moral character to bravely decide to not kill someone that you have no desire and no reason to kill, surely. Again, that's all I'm saying there: that your proposal is to make it so that no one ever wants to murder someone, and my reply is that if no one ever wants to murder someone then refraining from murdering people is no indication of any kind of moral character. Do this for everything immoral and no one can in any way be held morally responsible for anything they do, because their considerations are NEVER moral ones, but are just practical ones: they just don't want to. In fact, they'd never have to ask, wonder, debate or consider whether murder is immoral, or whether there are cases where killing someone is moral, because practicality would just settle the question. They'd never develop any idea of morality, or any personal moral code. They wouldn't be moral agents in any sense of the word. Since there is pretty good reason both theologically and philosophically to think that being moral agents is a good or desirable thing, your proposed model does not look like a good one, at least not necessarily a good one.

Because punishing an innocent third party as a “test” for someone is not consistent with the notion of an omni-benevolent diety.

Except that it's more that God created the test bed, this world, which is a world where bad things can happen to good people, and to people who clearly don't deserve it, as well as to people who DO deserve and who are bad people. The test for everyone is to figure out how to react to that world. The moral thing to do is to figure out how to stop bad things from happening to other people as much as you can. You've added on railing against anyone that might have created that, because you think that that sort of test bed is just bad. Fair enough, but your railing isn't actually an argument, and the issues are far more complicated than you seem to realize, and that's why this discussion keeps going no where, especially when you're arguing with someone whose reply is entirely "Your argument isn't sufficient because things are far more complicated than you think they are".

By Verbose Stoic (not verified) on 04 Mar 2015 #permalink

VS @ #424,

I see where you are coming from now. I just don't see being kicked out of the Garden as meaning that we are in an entirely different world now.

It does make for a clearer explanation, but it then implies that there is nothing we can do to fix THIS world we find ourselves stuck in now.

Christianity has always suffered from various doses of the Gnostic belief that this material world is essentially Evil. The Gnostics even had it as created by an Evil demi-god.

Most Christians think of Original Sin as effectively hard-wired into their genetics, such that there nothing that they can do about it.

Either way, whether we are unchangeable or our world in unchangeable and inherently Evil, no one will put the correct meaning to the words:

"Thy Kingdom come, Thy will be done, in EARTH as it is in Heaven."

I was told repeatedly as a young Christian that this world is like a sinking ship, so all we can do is try to get as many people into the lifeboats as possible before we leave.

Today, I utterly reject such a notion, and consider it to be damnable heresy.

Thus, I agree with Eric that THIS world that we presently find ourselves in was originally created to be free of suffering.

I then go on to assert that we brought the suffering on ourselves, and therefore we have the power to fix the problem. And the teachings of Jesus tell us exactly how to do just that.

And when we get it right, then the "New Jerusalem" comes down to Earth, being in effect the reversal of our being kicked out of the Garden. But I take these as metaphors, not as something that locks us in our present position with nothing that we can do about it.

Put another way, popular (quasi-Gnostic) theology has most Christians sitting around waiting for God to fix things. I see that it is our role to obey his commands, and when we do this we will see the problems start to fix themselves. The New Jerusalem speaks to the completion of this process, not the start of it.

By Gordon (not verified) on 04 Mar 2015 #permalink

In reply to by Verbose Stoic (not verified)

Gordon,

With all due respect, you seem to be a bit irrational about this one. I never claimed that the difference in the apparent sizes of the sun and moon demonstrate that God cannot exist. Certainly that is not a valid claim. I merely pointed out that this difference is indicative that the sun/moon apparent size coincidence is really not good evidence that God DOES exist. It's not really evidence either way in terms of the existence of God. Unless, of course, your God is a piss poor designer. Most human designs have tolerances far lower than 3%; should not your God be able to design to a more stringent tolerance than human engineers do?

It's not a matter of imprecise mathematics. The math is completely precise. It's the "design" that's off. Your weasel word "approximately" comes into play here in a scientific explanation of why the sizes are approximately the same. There is no scientific explanation because none is needed. How improbable is it that these two bodies appear to be APPROXIMATELY the same? Well, that kind of depends on what you mean by "approximately", doesn't it?

Current scientific understanding is that the moon was formed via a collision between the proto-earth and another pre-planetary body. Given this understanding, the sun's size and the moon-sun distance were pretty much established. The eventual moon-earth distance was also likely constrained by the energetics of the collision. What's left is the moon's size which would have been determined by the post-collision distribution of mass. That could have taken on various values, but it would not have been unconstrained. The point is that the moon's actual radius is 1737.1 km. Would you still say that the moon and sun appear to be "approximately" equal in size if the moon's radius were 1700 km? How about 1800 km?

I admit that we probably don't have the knowledge needed to figure the probability of your "coincidence" exactly. However, given your predisposition to see design, I suspect that you would accept a fairly wide range of sizes as being "approximately" equal in apparent size. Therefore, the probability of such a "coincidence' is likely higher than you think.

Gordon:

You go all fuzzy on the cancer statistics as well. Even the chart I linked to did clearly show an increase since 1975. I suspect it would be far more obvious if we could go back further. I suggested 1915 (100 years), and then you throw in some unsubstantiated statistic from “the 1800’s” to try to avoid admitting that the scientifically demonstrable facts are against you.

You're missing the forest for the trees. In making your cancer claim, you are saying that (1) human sin causes natural evil, (2) sin is on the rise, therefore, (3) natural evils like childhood cancer are increasing.

But yet over the past 100 years we have seen a two- to four- order of magnitude decrease in the chilhood death rate due to "natural evils." This is utterly inconsistent with your claims. How is the fact that infant mortality dropped from 197 per 1,000 in 1900 to 7 per 1,000 in 1997 consistent with your theology? Do we sin today at a rate 28 times less than we used to?

Gordon, do tell what God was thinking by "designing" the sun and moon to appear similar in size? What message was God trying to convey by this act? If they weren't similar would you conclude that they weren't designed? Say like the moons of Jupiter?

By Michael Fugate (not verified) on 04 Mar 2015 #permalink

Once upon a time, God created a world presumably without suffering in it, and then placed humans in it. However, that became unworkable for some reason and so God tossed humans out of that world into this one, a world which he explicitly promised would contain suffering. Thus, we are in this world that contains suffering.

So, taking that as our starting point, my argument is that if the implication of your argument is that this world can’t contain suffering, then you have something that’s massively suspect theologically since the starting premise theologically is that this world will have suffering and there are theological reasons given for why that has to be.

Your last comment simply doesn't follow from your narrative. The world obviously doesn't have to contain suffering since God designed one that didn't. It makes no sense at all to say 'once upon a time my theology says there was a whole teacup. It fell and broke. Therefore, whole teacups are a theological impossibility.'

it is indeed true that you are assuming that heaven, by definition, includes people with free will and yet who don’t do evil.

I'm restating the common christian position and pointing out its flaws. Restating someone's position on heaven does not mean I accept it as definitionally true. You have yet to give a reasoned argument why God could not restore humans to the edenic or heavenly state. This flaw in your position is clearly shown in your little narrative: you simply accept that "that [suffering-free world] became unworkable for some reason" and then you argue from there. That's apologetics. If you want to develop a viable solution to the problem of evil you have to tell us why it was unworkable and why it would be unworkable for humans today, rather than just accepting as a premise that it must be unworkable.

Also, you’d know what God wants for you, so people who genuinely love God would definitely not make mistakes.

You can't have it both ways: you can't claim such an express presence would undermine our free will (here) and yet not undermine our free will (there). The post-judgment/pre-judgment criteria just doesn't wash, given that different people die at different times and develomental stages of their lives.

The point I’m making is that if we were created to never want to do anything that happens to be wrong, then it is not an indication of our moral character that we resist doing them.

That's a straw man argument which ignores my example: women still get the urge to do violence, they just experience far less of it than men. So, do they not go to heaven? If they do, what's stopping God from granting men the non-violent tendencies of women?

This again gets back to the variability among humans, which simply cannot be theologially dismissed by "free will." We all have free will, but some of us are far less genetically tempted to do evil than others. So some of that temptation is clearly unnecessary for free will, for character development, and for entrance into heaven. Why does God give us or permit to exist this excess, unnecessary genetic temptation? And when this excess genetic temptation causes us to fail to reach heaven and we burn in hell for eternity due to having a genetic burden that was metaphysically unnecessary, what does that say about God's benevolence?

Eric,

You are exhibiting more symptoms of denial.

I made no claims from or about the Bible in my post about the size of the Sun and the Moon. The remarkable coincidence is implicit in the Creation itself.

Here we have two celestial objects - the only two that dominate our sky - that were created under very different circumstances. One is a ball of fire that Earth orbits. The other is a ball of dust orbiting the Earth.

By sheer virtue of gravity, the distances of these two heavenly bodies from the Earth differs widely, by about 400 times. That is a massive difference.

It then becomes the most remarkable coincidence possible, unexplainable from any perspective other than Design, that the size of the distant body "just so happens" to be about 400 times larger than the nearer body, such that from our perspective, which is the only perspective that actually matters to us, these two objects appear to be exactly the same size to the naked eye.

Of course this does not prove God. It merely points to a Designer as the only logical explanation.

But what this really makes clear is that you guys simply don't want to spend any time pondering the possibility of Design. It scares you. So you go straight into denial, and start making me say things I never said, to change the argument away from the single, inescapable fact that you cannot explain.

You want to change the subject on the cancer discussion as well. The issue that reveals Mr. Fry's hypocrisy is the increased incidence of childhood cancer in modern society. You want to discuss the fact that, on the whole we have greatly improved the treatment of all diseases. This speaks to a different point that I also raised, and you now wish to discuss only that point because you know that you cannot win the first one.

Even if my other point is wrong, it does not change the fact that Mr. Fry is a complete hypocrite. He is blaming a God whom he claims does not exist, for a problem that he does not understand.

Yes, Science has attempted to study spiritual matters such as the effectiveness of prayer on healing the sick. There is a fundamental problem with this. It reduces everything to the rules of Magic. Wave and wand and say, "Abracadabra", and then lets measure what happens. Then repeat.

Magic is not how God works. It may be how many Christians think about prayer, but that is a problem in their own thinking, not in how God is described in the Bible. In there, he is pretty down on the whole concept of magic.

The important distinction between the natural and the supernatural has been understood for a long time. You simply cannot put God in a box.

Michael,

The Devil's Postpile looks like an abandoned lumber yard to me. Perhaps it is. The Giant's Causeway has similarly straight rocks, but doesn't look like any Design I've ever seen.

It is really funny to see you guys pull out a few examples of the diversity of nature to refute the notion of Design. This is desperation. There is some need to not concede an argument even after you know you have lost.

All of you dismiss the size of the Sun and the Moon as meaningless, because again, were you to think about it for an instant you would have to concede that your worldview comes up short.

Your point about sheep and children is a good one. Very key, in fact. "Unless you become as a little child, you shall not enter the Kingdom of Heaven." I am only just now learning how essential this teaching is. But I don't see any purpose in discussing it with you here, as it is hard enough for Christians to grasp, and they actually want to understand.

Sean T,

Sorry Sean, I was a bit flippant in the opening line of my last post to you. You obviously misread my meaning, and I can see why.

The size of the Sun and Moon is indeed a "coincidence" once we find out how widely their sizes and distances actually are apart from each other, because they appear to be the same size from our perspective. This does not prove anything. But in searching for an explanation beyond "purely coincidental", the only one that pops up is Design.

I only used the word "approximately" in an attempt to avoid the petty argument that you raised anyway. It only takes a High School level course in Physics to learn about "significant figures". That was when I learned, much to my amazement, that some numbers just don't matter. As one who was obsessed with the precision of math, my mind struggled to grasp the implications. All that effort spent coming up with the "right" answer, for nothing?

But eventually my mind let go, and I understood that at a practical level, there was a limit to the utility of precise figures. If someone asks you what 98 times 103 is, and you give an immediate answer of "10,000", that is often more helpful than taking the time to carefully calculate out that the "right" (and therefore ONLY) answer is, "10,094".

So it is with the exact measurements you are worried about. All that matters here is what we perceive with our eyes. To the naked eye, the Sun and the Moon appear to be exactly the same size. So exact, in fact, that when we observe various eclipses, the sleight variation in distances is enough to make the Moon completely hide the Sun on some of them, and not completely hide it during others. Thus, depending on the eclipse, sometimes the Moon appears to be slightly larger than the Sun, and sometimes it appears to be slightly smaller. How much closer to "the exact same size" do you want to get? Surely you can see that the mathematics at this point becomes completely irrelevant.

The size of the moons of Jupiter is similarly irrelevant, as we do not observe them with the naked eye. Our perspective from here on Earth is all that matters. Now, if you could show that every planet's moons were sized and distanced such that they looked the same size as the Sun from the surface of the planet, then there is probably a mathematical explanation behind it all. I don't know if anyone has ever done this. I only assume that you will not find such a "remarkable coincidence" on any other planet, but I could be wrong.

Since we have several mathematicians on this forum (hosted as it is by a Professor of Mathematics), you guys should take this as a challenge. Talk about a great "Problem of the Week!"

Show me a mathematical relation between the sizes of planets and moons, and the distances of their orbits, such that the size of our Moon merely fits the pattern, and my argument goes out the window.

There is a necessary element missing from atheism. We call it "Wonder". Because we assume a Creator, we find ourselves repeatedly in awe at the wonder of his creation. This makes us ask further questions. Admittedly, not enough Christians share my sense of wonder, but perhaps you guys can see that you are missing something important when your mind fails to engage in a meaningful way when presented with truly remarkable facts.

This is not about winning some argument. This is about understanding the Creation, whether or not you believe that God created it. After all, what do you think Science actually is?

ORTHODOXY

Let me recommend to all the skeptics here that you read the definitive answer to all of your arguments in a book published just over a century ago.

All the arguments you raise are rehashed versions of issues debated quite at length in the late 1800's. G.K. Chesterton answered all of those arguments, and explained the Christian perspective in exquisite detail, in his book Orthodoxy.

He lays out the groundwork for any rational belief system, and then shows why Christianity is the best one. Even if you don't agree with his ultimate conclusion, you will see the weaknesses in many of your arguments.

This is deep and challenging reading. You have never encountered anything like it. It is also as pertinent as if it were written yesterday. I believe you will all find it richly rewarding if you are serious about basing your views on sound reasoning.

You can read the book online, or download it to an eBook reader, for free. Here is one such link:

http://www.pagebypagebooks.com/Gilbert_K_Chesterton/Orthodoxy/

I have had to resist the temptation to quote him at length in many of my previous posts, and I will give only one poignant citation from the book that explains the dilemma I have faced:

"It is very hard for a man to defend anything of which he is entirely convinced. It is comparatively easy when he is only partially convinced. He is partially convinced because he has found this or that proof of the thing, and he can expound it. But a man is not really convinced of a philosophic theory when he finds that something proves it. He is only really convinced when he finds that everything proves it. And the more converging reasons he finds pointing to this conviction, the more bewildered he is if asked suddenly to sum them up. Thus, if one asked an ordinary intelligent man, on the spur of the moment, "Why do you prefer civilization to savagery?" he would look wildly round at object after object, and would only be able to answer vaguely, "Why, there is that bookcase . . . and the coals in the coal-scuttle . . . and pianos . . . and policemen." The whole case for civilization is that the case for it is complex. It has done so many things. But that very multiplicity of proof which ought to make reply overwhelming makes reply impossible."

What you will find is that Chesterton has solved this problem by writing an entire book. The case he makes is so complex and so complete, that simply citing a paragraph here or there could not possibly do him justice.

And frankly, he puts all other Christians you have ever met to shame. His approach is intellectually invigorating, and even when you feel he is getting lost in a maze of assumptions or circular reasoning, he soon emerges with a point that stops you in your tracks.

Even if all you want to know is why Christianity is still around after 2000 years, and why it will still be going strong another 2000 years from now, read Chesterton.

Gordon,

You are still confused. You want to fluff off the fact that the distance ratio and the size ratio of the sun and moon is not identical as just "mathematics". Sure, estimates can be useful, but when one is designing a system, typically estimates are not used; more precise measurements are.

Let me try a different approach. You are claiming that the apparent size of the moon and sun demonstrates design. From your posting, it's pretty clear that God is your designer. Let's assume for the sake of argument that this is true. Now, presumably God wants the moon and sun to be the same size as seen from earth. First of all, why would he make one so much larger and farther away? If you are designing a system where two objects need to be the same size, would you not simply use two objects that actually are the same size? I'm sure God is sufficiently well-versed in thermodynamics and astrophysics that He could have designed a moon-sized sun located at the same distance as the moon is that would have provided the right amount of heat to earth. (Or He could have designed another energy source to provide the necessary energy.)

Since that's not the actual situation, then for some reason God obviously wanted to have the sun and moon appear to be the same size, but actually be different in size. Given that assumption, why did He make it so that there is the difference that I quoted in my earlier post that you seem to just fluff away? Was God such a shoddy designer that He needed a 3% tolerance? I would think that God would be a superior designer to humans, wouldn't He? I am sure glad you'd be comfortable, for instance, if you fly on airplanes with components with a 3% tolerance, but I am not. Why should I expect less precision from God than I do from Boeing's engineers?

BTW, the relationship that you ask for does not exist. The sizes of the satellites seem to be independent of the distance of their planets from the sun, or at least that's true for our solar system. We have just begun to detect extrasolar planets, however. We cannot detect satellites of those extrasolar planets, though. Therefore, it's not impossible that some relationship exists in general, but that our solar system for some reason does not display this relationship. Or it's just possible that it's a coincidence, nothing more. We don't need to invoke God, though, every time we detect a coincidence.

eric,

Let me start from this to clarify the position:

I’m restating the common christian position and pointing out its flaws.

This means that what you're doing here is a theological examination, looking at what the theology says and trying to demonstrate that it's inconsistent. What you're doing in these cases is looking for INTERNAL consistency; examining whether if we take all of the premises the position entails we end up with an inconsistency. This means, however, that you have to accept those premises for the sake of argument, because your argument relies on them. So, for the first case:

Your last comment simply doesn’t follow from your narrative. The world obviously doesn’t have to contain suffering since God designed one that didn’t.

The issue is that none of the theological positions are saying that this world couldn't physically possibly have been one that didn't contain suffering. It couldn't have not contained suffering and been a world where God's purposes were fulfilled. Which is where the issue comes in: if you derive theologically a position that actually contradicts explicitly stated theology -- ie that for whatever reason this world is going to have suffering because God explicitly says "Here there be suffering" -- then the first and most plausible interpretation is that you've screwed up your derivation somewhere. So we'd need to look at how you get there, and the argument is that you get there by interpreting "benevolence" in ways that imply it, even as you try to avoid that by focusing on specifics instead of on generalities. But if benevolence would insist that theologically God, for example, couldn't fulfill the purposes that are stated and accepted theologically AND that are generally accepted INSIDE THE THEOLOGY as being reasonable, then you again run into the issue that your interpretation seems wrong. So we need to look at how you got there.

And you get there by interpreting benevolence in a certain way. But as I've pointed out, it isn't clear that a benevolent (by which I mean omnibenevolent) can't allow suffering. I've given lots and lots of examples of how this is problematic. I'm not saying that you aren't right and that the Problem of Evil absolutely DOESN'T work, but it's no where near as clear as you're saying.

In short, if you derive conclusions from the premises of a theological position that produce contradictions with explicit statements, your derivation will seem suspect and need to be challenged. And you won't be able to turn around and deny the premises you used in your derivation if someone points out they lead to a different conclusion, which is what the whole "people won't be evil in heaven", to my mind, reduced to: I used that fact to derive a different implication, and you accused me of apologetics and insisting that it was true by definition, when all I said is that because your argument insists on that I'm free to use that premise AGAINST your argument just as you are free to use it FOR it.

(Also, I have no idea what you mean by "definitionally true" here. I'm talking about the concept we're talking about, not claiming that the concept exists or is true).

You have yet to give a reasoned argument why God could not restore humans to the edenic or heavenly state.

God can. He won't, for the myriad reasons that everyone in this thread who has been arguing against you have pointed out. You just don't want to address any of them.

This flaw in your position is clearly shown in your little narrative: you simply accept that “that [suffering-free world] became unworkable for some reason” and then you argue from there.

No, actually, I don't. The narrative simply points out that the theology EXPLICITLY states that Earth and heaven will be different and gives reasons why. No one is expecting you to simply accept those reasons as, say, solving the problem, but we can reasonably expect you to, when making your criticisms, acknowledge that they are there and be prepared to argue against them. In fairness, you do at some point, but we got to this point by you trimming out all of those discussions and narrowing it down to definitional debates. You didn't say, for example, in the discussions with me that you accepted that God said that suffering in this world is a consequence of the Fall but that that is unjust, but instead kept asking what reason there possibly could be. No progress can be made if you keep doing that.

And you can't object that I made the conversation too abstract because you continually claim that my positions are too non-standard as a rebuttal to them, or at least in lieu of addressing them, which forces me to be more abstract and general to avoid those sorts of complaints.

You can’t have it both ways: you can’t claim such an express presence would undermine our free will (here) and yet not undermine our free will (there). The post-judgment/pre-judgment criteria just doesn’t wash, given that different people die at different times and develomental stages of their lives.

Um, why would that matter? People still get judged on what they've done up until then. You can argue that you don't think that fair, but that's a completely new argument. The fact is that free will is required pre-judgement, and not after. You can take many, many -- as I outlined -- theological positions from that simple fact about Christian theology ... or, at least, the ones that your argument would apply to.

That’s a straw man argument which ignores my example: women still get the urge to do violence, they just experience far less of it than men. So, do they not go to heaven? If they do, what’s stopping God from granting men the non-violent tendencies of women?

That's because your example doesn't actually demonstrate anything, and is a prime example of you switching between specific cases and general claims. So, imagine that God did that. Do you think that then we'd have a good world, a proper world, a world where the Problem of Evil is dealt with on that score? Presumably not, because you were using this as a comparison with heaven where no one does any bad things whatsoever and has no immoral tendencies at all, and demanding to know why we can't have that here. So this change wouldn't satisfy you on this count, and so, yes, we'd have to have NO genetic tendencies, at least, towards bad things. So, no, not a strawman, unless you are changing your argument.

This again gets back to the variability among humans, which simply cannot be theologially dismissed by “free will.” We all have free will, but some of us are far less genetically tempted to do evil than others. So some of that temptation is clearly unnecessary for free will, for character development, and for entrance into heaven. Why does God give us or permit to exist this excess, unnecessary genetic temptation? And when this excess genetic temptation causes us to fail to reach heaven and we burn in hell for eternity due to having a genetic burden that was metaphysically unnecessary, what does that say about God’s benevolence?

I'm going to pound on this for a bit to give you a chance to argue substantively.

1) Whatever innate or genetic tendencies we have, we can overcome them and so avoid Hell. Young men can resist their violent tendencies, alcoholics can resist drinking, and so on. There is NO case where simply having the tendency causes us to fail' in the end, it all comes down to us.

2) You can argue that some people have a harder time overcoming their tendencies and so those tendencies decrease their chances of overcoming them, which isn't fair. The problem is that your examples don't demonstrate that, as while older women don't have the violent tendencies to overcome it is likely that they have other tendencies to overcome. It turns out to be amazingly difficult to determine based on tendencies who has an easier or harder time of it especially since even a more apathetic set of tendencies only switches off the moral failings that you can have.

3) You ignore the fact that violent tendencies, for example, are not necessarily tendencies towards immorality; sometimes violence can be morally right or morally necessary and people with less violent tendencies can therefore end up failing morally to that commitment. Think of physically presenting a rape, for example.

4) You assume that these tendencies are genetic and so not under the control of the people, but many of these are environmental. There might be some genetic/physical link -- to testosterone, for example -- but there are women who are more violent than some young men, for example, which indicates that environment and development play a key role. Because of this, your claim that God created them that way doesn't wash in the context of a discussion where free will is presumed, because environment and development would be influenced by the free will decisions of the person and the people they encounter, making the argument problematic unless you deny free will ... which YOU can't do here and retain an initial argument worth addressing.

By Verbose Stoic (not verified) on 05 Mar 2015 #permalink

Think of physically presenting a rape, for example.

"Presenting" should obviously be PREVENTING here.

By Verbose Stoic (not verified) on 05 Mar 2015 #permalink

Gordon, That you do not see design in the Giant's Causeway and the Devil's Postpile means there is no universal design detector; others do see design there - hence the names and stories surrounding them (remember a wall is a wall). Blows your silly argument clean out of the water. You do realize that if you propose intelligent design then you have to be able to explain the plan behind the design, don't you? You can't just say its designed without telling us for what purpose it was designed. Why do the sun and moon apparent similar in size from earth?

As for the whole sense of wonder trope - really? I work with research scientists and have for 30 years, I don't know anyone with a more developed sense of wonder than my coworkers and can find no difference between those who are theists and those who aren't. Once again evidence triumphs over assertion and anecdote - which is all you have. This is the problem with apologetics - you start with the answer and twist, discard, mutilate evidence to fit that answer. Science works from the evidence to the answer - something you cannot understand.

By Michael Fugate (not verified) on 05 Mar 2015 #permalink

I am wondering if by Gordon's logic that, even though there is good evidence for ancient water on Mars and therefore life?, we can know God didn't put life there because the ratios of the moons and the sun are off? Not to mention, the moons Phobos and Deimos are misshaped blobs - not spheres (no design obviously) and the farthest moon (Deimos) is the smaller of the two.The sun would appear almost 10 times larger and the Phobos would appear almost 5 times larger than Deimos. If they were the same ratios, then would we know for sure there was intelligent life on Mars? This all getting closer and closer to Douglas Adams' water puddle...

> This is rather as if you imagine a puddle waking up one morning and thinking, "This is an interesting world I find myself in - an interesting hole I find myself in - fits me rather neatly, doesn't it? In fact, it fits me staggeringly well, must have been made to have me in it!' This is such a powerful idea that as the sun rises in the sky and the air heats up and as, gradually, the puddle gets smaller and smaller, it's still frantically hanging on to the notion that everything's going to be all right, because this World was meant to have him in it, was built to have him in it; so the moment he disappears catches him rather by surprise. I think this may be something we need to be on the watch out for.

The Salmon of Doubt pg.131.

By Michael Fugate (not verified) on 05 Mar 2015 #permalink

Michael Fugate,

“there is good evidence for ancient water on Mars and therefore life”

Therefore life? That’s a pretty good hop Michael. You need quite a few miraculous accidents to happen. Would they occur in the water?

This is an interesting video produced by Georgia Tech/NASA about ribosome, and your deity is given full credit.

https://www.youtube.com/watch?v=Z2XOhgRJVb4

Michael,

No, no, you've got the logic reversed. I was supposed to say "Yes", that I saw a design, to which you were going to reply that there was No Design, as these were natural occurrences, thus disproving my "silly argument".

So really, no matter what, everything points to your disproof of all proof, except of course, for when you are the one who actually wants to prove something.

Thank you for doing the math, and confirming for us that there is no pattern to distances and sizes of moons, at least not when comparing Earth to Mars. Funny how you even manage to turn this around and try to make it look like it does not support my point. You are one big mental tangle.

As for a sense of Wonder, if you have one, it sure doesn't show. Nor is it evident from any of the other replies from atheists. Assert all you want about your co-workers, but when an amazing scientific coincidence only ruffles your feathers, it is obvious that somewhere along the way you forgot how to appreciate life to the full.

By Gordon (not verified) on 06 Mar 2015 #permalink

In reply to by Phil (not verified)

Sean T,

Somewhere along the way you lost a fundamental connection with reality. Like Mr. Fry, you are so eager to blame God even hypothetically, that you believe your numbers actually mean something.

For one thing, you couldn't verify the correctness of those numbers if your life depended upon it. For another thing, they are merely averages, as the distances change depending on where Earth and the Moon are in their orbits.

For you to fault God for a 3% difference in unverifiable numbers, whilst repeatedly ignoring the tangible facts that everyone observes from the ground, tells me that there is no hope of continuing any rational discussion. I am up against a deeply held blind faith here, not logic.

Or you are just trying to be funny, and failing miserably. You can't be serious about God being a bad designer because he did not place a Moon-sized Sun orbiting the Earth as well. Because if you are, then you are advocating a Geocentric model of the galaxy, and you need to join the Flat Earth Society. They claim exactly that!

It's been fun, but I need to move on and spend my time on futile arguments against the blind faith of Christians. This has been very worthwhile up to now, and you have forced me to rethink things many times. So I will remember you for that, and not for this rather ignoble ending to an otherwise spirited and challenging exchange.

Maybe it's not clear. If you think my argument is silly, then it's you who should rethink what you're arguing, not me. My argument is simply to assume for the sake of argument that what you said about the apparent sizes of the moon and sun is true and see where it leads. I am certainly not claiming God is a bad designer. I am an atheist; I can't claim that God is a bad anything. I am simply raising questions that you should be raising if you truly believe that the sizes of the moon and sun really are evidence of design. if you find that the consequences of such inquiry are absurd, then maybe it's your initial assumption about this being evidence of design that is faulty.

Just a few points about my logic: we CAN design things to tolerances that are tighter than 3%. Therefore, we should assume that God can do better than 3% as well. Despite your protestations, we CAN measure the sun-moon and earth-moon distances to a better accuracy than 3%. Even combining variations about the average distances and measurement error cannot give you a tolerance as great as 3%. The apparent size of the moon and sun as seen from earth are only APPROXIMATELY the same. Again the word "approximately" tears down your argument. Why would an all-powerful God who intended that the sizes be the same make them only approximately the same?

I'm not sure why you are getting so defensive about this argument anyway. You could very reasonably and rationally just say, "well that argument about the sizes of the moon and sun was faulty," and it would make NO DIFFERENCE to your overall arguments about God, your theology or anything of importance. I have already fully conceded that the fact that the apparent sizes of the sun and moon are not quite identical has no bearing on whether or not God exists, whether or not the Bible is true, or any other such question. It's really not germane to the real point you're trying to make here, so why defend it, especially since it's factually incorrect to state that the apparent sizes of the moon and sun as seen from earth are identical?

I suppose the real point I'm trying to make is that you cannot just look at some feature of nature and say "it's obviously designed" (or it's obviously not designed either for that matter). Design is something that is not easily detectable. Define what features would make you conclude that something is 100% designed. "I know it when I see it" is not a valid definition; that's really begging the question. What's at question here is HOW do we know something is designed. So far, all I've seen from you is "I know design when I see it"-type argument. Such arguments are unconvincing. If you want to objectively conclude that something is the result of design, you must come up with a way of looking at that thing and making the determination based on its properties. I would contend that such a definition is impossible. We must know the history of the feature, not just its observable properties. For instance, if you look at a table with 10 pennies all showing heads, you cannot tell from that alone whether someone set those pennies on your table intentionally in that arrangement or if they were just dropped on the table and happened to all land on heads. Similarly, it's very difficult to determine, without knowing how something came to be the way it is, whether it's the result of undirected natural forces or of design.

Sean T,

If anyone looked a table with 10 pennies on it, and then noticed that all ten were heads up, they would rationally conclude that someone deliberately arranged them that way.

Even though it is possible for them to have been dropped and to have simply all fallen heads up, the odds are so slim that we would laugh at someone if they said, "What a coincidence. Someone dropped their pennies on the table and they all just happen to land face up!"

We would wonder what they had been smoking, or worse.

I was really hoping you were just joking, but you are pursuing this line of argument with all seriousness, so I will bow out.

There is some age old advice about arguing with a mad man.

By Gordon (not verified) on 06 Mar 2015 #permalink

In reply to by Sean T (not verified)

If anyone looked a table with 10 pennies on it, and then noticed that all ten were heads up, they would rationally conclude that someone deliberately arranged them that way.

Even though it is possible for them to have been dropped and to have simply all fallen heads up, the odds are so slim that we would laugh at someone if they said, “What a coincidence. Someone dropped their pennies on the table and they all just happen to land face up!”

In a fair and random independent simultaneous distribution of some set of coins, all heads (or all tails) is exactly as likely as any other specified combination.

Your perception of "design" in your coins example is as specious as it is everywhere else.

Phhht,

You guys are funny!

The odds are slightly more than 1 in 1000 that all coins would fall heads up.

Another 1 in 1000 for all tails

Which leaves 998 in 1000 for "any other specified combination" of both heads and tails, which is what we expect from the random landing of 10 coins on the table.

You guys continually bet on the long odds, and wish us to consider that rational behavior. Have you won the lottery yet?

By Gordon (not verified) on 08 Mar 2015 #permalink

In reply to by phhht (not verified)

Gordon's numbers are right (to about the same precision as the sun/moon comparison), and if we could compare our universe to other universes of known origin - the way we can compare coin flips - we could determine a similar probability. Lacking that, we must choose whether to base our theories on demonstrable facts or on dreams, magic, gods...

By Walt Jones (not verified) on 08 Mar 2015 #permalink

Gordon,

You're wrong, not in your numbers (2^10=1024) but in your reasoning.

Suppose that the display of coins on your table was arrived at like this. We take 1024 sets of 10 coins, arrange each set into a unique combination of heads and tails, and stick it to a piece of tape.
We put all those sets of coins into a bin, choose one at random, and place it on your table.

The odds of your choosing the one tape with all heads is exactly equal to the odds of your choosing any other set (if your choosing is fair and random).

You reason that some particular combination of coins on your table (such as all heads) conclusively indicates human intervention. That is not correct, as my example demonstrates.

The odds are slightly more than 1 in 1000 that all coins would fall heads up.

Another 1 in 1000 for all tails

Which leaves 998 in 1000 for “any other specified combination” of both heads and tails, which is what we expect from the random landing of 10 coins on the table.
You guys continually bet on the long odds, and wish us to consider that rational behavior. Have you won the lottery yet?

More garbage. The same could be said for ANY arrangement of the ten pennies, not just the one with all heads. Why does this particular one say to you "design design design"?

Either you don't understand or you are being (once again) intentionally dishonest. I figure it's 50-50 which is true.

Phhht,

I understand your point. But you miss the fact of how our mind works.

When we look at ten coins with some heads up and some tails up, we think that is "random". But when we see all ten either heads up or all tails up, we see "order" in that.

So the 2 possibilities of all heads and all tails are not equivalent in our mind to the other 998 possible arrangements. With those two, we perceive something different, even though those 2 arrangements are equally likely to all the rest.

This is universal human experience. You all experience it, even if you choose to deny it now for the sake of your argument.

By Gordon (not verified) on 08 Mar 2015 #permalink

In reply to by dean (not verified)

Gordon,

<i<I do not experience that, so your claim of universality is as specious as your claim of "design."

I'm pleased, however, that you concede that an array of all heads is no more or less likely than any other arrangement, in reality.

And Gordon, were you also conceding that "design" is simply an illusion, just like the purported significance in that array of all heads?

Because that's how it looks to me.

Phhht,

I assume that you have a house, and that your front door has a lock on it that you lock when no one is home.

Looking at the key, there appears to be no pattern to how it was cut. Yet was that key designed? Does it fit the particular lock in your particular door? Is the exact pattern of it significant?

Someone had to cut the key to fit the lock, and even though there are a limited number of patterns of keys for the number of tumblers in your lock, still, you do not show up at the locksmith and ask for any key out of a barrel containing all possible patterns.

You could just as easily argue that the apparent design of keys is just an illusion, even though there is no discernible pattern whatsoever on your particular key. But that "illusion" is very good at opening your front door, and beats all other options.

Our minds process many thoughts subconsciously, or at least so fast that we do not follow fully the logical chain leading to a particular conclusion. We use previously learned facts, and skip right over others, when reaching most conclusions.

So, if we break down what happens when we see ten coins all heads up, it goes like this:

1) Pattern recognition: "Hey, all those coins are heads up"

2) Probability: Even though almost no one will calculate that the odds are precisely 1 out of 1024, we all know intuitively that the odds are extremely low.

3) We are certain enough about the odds that we decide straight away that the chances of this happening randomly are effectively Zero.

4) So any decision or statement we make with regards to the arrangement of coins will be to the effect that they were certainly arranged that way intentionally, even though there is a small probability that it happened by chance.

5) The odds are actually even better that the numbers tell us, because we will be correct in every instance where someone did arrange the coins this way, and we will only be wrong 1 time in 1024 when the coins are dropped randomly. The other 1022 times (leaving out all tails) we will correctly conclude that the coins were dropped randomly.

So when we see a pattern, our immediate intuition is an extremely reliable guide. It is when we do NOT detect a pattern that we cannot answer whether the coins were dropped or arranged that way. Similarly, if you showed me your key, I could not tell whether it fits the lock in your front door.

A pattern almost always demonstrates Design, while the absence of a pattern does not necessarily disprove it. This is the whole point. The evidence is overwhelmingly in favor of Design, even if there is some vanishingly small possibility that the order of things, especially Life itself, came about without the intentional efforts of a Designer.

So, do you spend your day down at the racetracks, betting on the longest odds every time? It does attract the biggest payout, so if your thinking is correct, you should be a very rich man. Most of us have kept our day jobs.

By Gordon (not verified) on 08 Mar 2015 #permalink

In reply to by phhht (not verified)

Gordon, you load of crap argument could be applied equally well to the situation where, from left to right, the coins alternate heads and tails. You are simply picking one because you find it striking.
My freshman stat students know better than to make the claims you are making.

dean,

But that case is striking for the same reason that all heads or all tails is striking: there's no particular reason for a random process to produce it, but we can think of reasons why a directed or intelligent or designed process would. So a more fair comparison would be if you had one head, and then three tails, and then one head, and then one tail, and then two heads, and then one tail and then one head ... but almost no one will think that that one is designed or intentional, and will generally think it random.

Design, I think, is never assessed on the basis of pure probability, but instead on the appearance of intent. This includes all of the other examples talked about previously. Getting into probabilities is just confusing things.

By Verbose Stoic (not verified) on 09 Mar 2015 #permalink

It seems like Gordon and VS are assuming that the world is supposed to look like it does, with all of the coins heads up. If all of the coins were tails up (to stretch and mix the metaphor), Captain Kirk might be evil, but we can't say that a world would not exist - it just might not be our world. Same for any other combination of coins.

The chances of any specific person winning a lottery are very low, but many people have won lotteries.

By Walt Jones (not verified) on 09 Mar 2015 #permalink

In reply to by Verbose Stoic (not verified)

There is no particular reason for randomness to place "one head, and then three tails, and then one head, and then one tail, and then two heads, and then one tail and then one head "

Only the deluded see design where there is none, although if you are a creationist then it is likely that probability does confuse you.

The larger point (wishing for a preview button instead of a submit button alone) is this: all of the arrangements of the coins are governed by probability alone: all heads, all tails, or sequences of alternating cases stand out simply because people don't believe them to be random. That's a bias of the mind, it in no way hints that there was any intended design or intervention.

There is no means of detecting which set of 10 coins was intelligently designed and which was randomly tossed. Ten heads set out by me on purpose looks just like one I randomly tossed. You need to read the mind of the tosser/arranger to know. Not only that, given a heads/tails sequence is binary, every single arrangement is a decipherable pattern if you know the language in which it was written. A seemingly uninteresting sequence could be much more intelligently designed than all heads. Patterns are everywhere we look - some like constellations, tea leaves, palm lines, goat entrails and the like were once thought to rule our lives - now not so much.

The design argument, even if true, is dead because it has no predictive power without knowing the mind of the designer. Get back to us when know your god's mind.

By Michael Fugate (not verified) on 09 Mar 2015 #permalink

dean,

The point is that in the last example it doesn't look like there's any intention or purpose that could explain that pattern; it is quite unlikely that any intelligent agent would pick that one on purpose, for a specific reason. Whereas for the patterns, we can see that there are reasons why an intelligent agent might do that on purpose, if for no other reason than to reproduce the pattern. That makes it a much more reasonable claim to say that the patterned one indicates a purpose than to say that the random one was, despite the fact that random processes can produce that sort of intentional seeming pattern and that intentional processes can produce that random seeming pattern.

So calculating how likely each pattern is doesn't really do anything, because the reasoning is not really "Wildly improbable" but is instead more like "Would come up with an arrangement that seems intentional totally at random".

Michael Fugate,

And yet, we try to figure out the minds and intentions of people all the time, even with imperfect knowledge ,so that doesn't seem like much of an objection. In fact, it sounds almost like a concession, claiming that maybe it really is purposeful and designed but that we can't prove it either way. If an ID theist argues that they know that it was designed, that might work against that argument, but it doesn't allow you to say that it wasn't designed either if you can just say that maybe it was random.

By Verbose Stoic (not verified) on 09 Mar 2015 #permalink

The point is that in the last example it doesn’t look like there’s any intention or purpose that could explain that pattern; it is quite unlikely that any intelligent agent would pick that one on purpose, for a specific reason. Whereas for the patterns, we can see that there are reasons why an intelligent agent might do that on purpose, if for no other reason than to reproduce the pattern.

The hypothetical intelligent agent could pick any of the patterns: the ones that stick out to you because the seem special, and the ones that don't, to hide its tracks. There is still no good reason to decide one pattern indicates design and another does not.

VS,
So your "designer" is just a human? You do realize the difference between a god and a human, don't you? Seriously do you think that if we understand human behavior, we know a god's? Go read some Hume, please.

There is nothing random about gravity, thermodynamics, electronegativity, natural selection and the like. It is not randomness v. intelligent design - please learn at least a bit of science before making comments.

By Michael Fugate (not verified) on 09 Mar 2015 #permalink

Michael Fugate,

There is nothing random about gravity, thermodynamics, electronegativity, natural selection and the like. It is not randomness v. intelligent design – please learn at least a bit of science before making comments.

If you know that it isn't designed, then you won't be making the argument that I was talking about, and so presumably will be saying a lot more than what was said in the comment that I replied to. And insisting that when someone says "This looks like a mind did it" that a reply of "You don't know the mind of God" is a rather odd argument; it sounds more like the argument the ID theorist will say when the reply is given that, no, it REALLY doesn't look like it was designed, which makes it an odd argument to oppose ID with.

In all honesty, I have no idea what argument you're trying to make here or why it relates to what I'm saying.

dean,

The hypothetical intelligent agent could pick any of the patterns: the ones that stick out to you because the seem special, and the ones that don’t, to hide its tracks. There is still no good reason to decide one pattern indicates design and another does not.

If your counter is that the seemingly random arrangement might actually be designed, you'd be right ... but it seems less than relevant here, especially since you seem to be using it in a context where the intelligence is trying to hide the fact that the things were deliberately arranged. That in no way impacts whether we should consider the things that seem to be things that are arranged for a purpose to have one by default, even as we accept that we could be wrong. Again, I'm not sure what this argument is aimed at.

By Verbose Stoic (not verified) on 09 Mar 2015 #permalink

It was convoluted: the arrangement of the coins is irrelevant - whether all heads, all tails, alternating, or the one you proposed. None of them are evidence for the influence of a designer, as basic probability tells us all arrangements are equally likely - and so, all are equally unlikely.
Concluding that a particular arrangement is evidence of design comes from one's personal bias.

and again proofreading fail: my intention was to close the bold tag after the word "unlikely". Apologies.

Dean @ #453,

"Left to right" is another pattern.

I was assuming the coins were scattered randomly, with no other pattern discernible other than the number of heads and tails.

The moment you see a line of ten coins (left to right), irrespective of heads or tails, you would have to out of your tree to claim that they "coincidentally" fell that way.

If you come back with "no Design" on this one, I will send the nut house out to collect you straight away.

By Gordon (not verified) on 09 Mar 2015 #permalink

In reply to by dean (not verified)

The moment you see a line of ten coins (left to right), irrespective of heads or tails, you would have to out of your tree to claim that they “coincidentally” fell that way.

In any pattern? I'm not sure what the word I've bolded is indicating.
Regardless:

So I think it quite reasonable to think that if we say all heads or all tails that someone probably arranged it that way,...

I'll still disagree. The all heads or all tails scenario catches our eye because of the uniformity (as other specific patterns might, although not so strongly) - but as long as one realizes it is no more likely to occur simply by chance than any other arrangement, then without any other information you should not (I would say can't) assume they were purposely left that way.

By dean (not verified) on 09 Mar 2015 #permalink

In reply to by Gordon (not verified)

dean,

I think you're missing my point on that, at least. My point is that we don't judge in terms of absolute probability, but on an appeal to apparent purposefulness. When we see patterns that we can easily imagine someone doing on purpose, we reasonably, in my opinion, consider that the more reasonable belief than that it was random. The harder it would be for a more random process to produce the same pattern and the more obviously the pattern seems to be on purpose, the more reasonable it is to consider that designed.

So I think it quite reasonable to think that if we say all heads or all tails that someone probably arranged it that way, even though it could be done randomly, and also quite reasonable to consider the pattern I gave as random even though someone might deliberately arrange it that way. We aren't going to demand certainty, I hope, and we can easily see that in the first cases that pattern is one that might appeal to a mind while it's harder to see how the last pattern would be. And we can maintain this until we know what the process was and whether it was random or intentional.

So I don't see what's wrong with this at all.

By Verbose Stoic (not verified) on 09 Mar 2015 #permalink

It is important to note that there is one assumption here - that they were randomly tossed, and the two sides are equally likely to fall up. Perci Diaconis (and others) have done some interesting work that indicates a subtle bias: the chance a flipped coin will come up with the same side that was originally up is about 0.51.

http://statweb.stanford.edu/~susan/papers/headswithJ.pdf

Gordon,

Without regard to linear ordering, there is nothing at all special about a distribution of, say, all heads.

Just as there is nothing special about a distribution of half heads and half tails, or nine heads and one tail, or any other distribution.

No matter how you try, you cannot rationally justify your claim that a distribution of all heads (or all tails, or any other distribution) constitutes even the slightest indication of intelligent agency.

You argue that you just know, looking at a particular distribution, that it just must be the result of intention.

That is the "design" fallacy in a nutshell.

Verbose Stoic,

So I don’t see what’s wrong with this at all.

What's wrong is that you have nothing.

All you can do is to insist that you don't see what's wrong with your "reasoning."

You cannot offer even the slightest bit of testable evidence that you are correct. You cannot offer any plausible rationale. All you can offer is your unsupported opinion.

That's what's wrong with the "design hypothesis."

The first thing I would do if I saw a bunch of coins all lying with heads up would be to turn them over to see if they were two-headed coins. If that were not the case and I were sufficiently interested, I would try to get more information - whose coins they were, how did they get there, were they magnetic on one side, et cetera. Trying to "reason" an answer to a question before one has enough evidence is an unfortunate and largely useless human tendency for which science was invented to compensate.

The evidence I have seen, and see every day (latest example, 20 minutes ago on my TV: a Dyson vacuum cleaner commercial which stated that Dyson has tried over 500 versions trying to make the "perfect" vacuum cleaner) is that the distinction between design and evolution is meaningless. Humans design (and I think reason) by a process of evolution. Had Paley been a watch-designer who knew the history of watch development he might not have said, "The watch was designed and so were the trees in this forest," but rather "The trees evolved and so did the watch." (Because he had no understanding of how either trees or watches developed he grasped at the non-explanatory explanation: something I have never seen did this by a means I don't understand.) We have all seen watches, phones, cars, TV's, etc. evolve in our life times. The trees just evolved so slowly over so long a time that we barely noticed.

We like to evolve things (Feynman called it, "the joy of finding things out", in my case power-generation turbines and computer programs) a) probably because that is a natural pro-survival tendency which biological evolution has programmed into us; b) because doing so is a good way to earn the money which we use to survive and reproduce.

Those of us doing the designing (assuming it has not been reduced to rote "design rules") notice that, like mutations, most design changes either do nothing or make the product worse - or would do if they survived the design reviews and prototypes and field testing. However, if you try enough things that don't work (such as Edison's thousands of light-bulb filaments or hundreds of battery elements), and if there is something that will work, eventually you will find it, and later find improvements to it.

(Yes, my conclusion could be wrong because I am missing some contradictory evidence. However at this point in my life the observational evidence I have accumulated seems overwhelming to me - all of it hard evidence which everyone can see and replicate, like the Dyson and Edison examples.)

phhht,

"You argue that you just know, looking at a particular distribution, that it just must be the result of intention....That is the “design” fallacy in a nutshell."

If you really want to confront the design fallacy, forget coins and flipping, and use something real like amino acids in sequences that actually do something.

Phhht,

Thank you for confirming your sanity.

So if they are laid out in a line, we agree that human intervention was almost certainly involved. And this despite the infinitesimally small chance that they could have simply landed that way.

But you are not really arguing the statistics. What you have failed to do is to address my careful explanation of why our mind intuitively "bets" on the 998 out of 1000 odds over the 1 in 1000.

You refuse to acknowledge the simple fact that we instantly recognize, without thinking about it, that all heads only happens 1 time in 1000, and so is very unlikely to have happened randomly.

While it is true that all other possible distributions are just as likely, the rest of them (other than all tails) lack a feature that would make us view them as anything other than random. Thus, our mind puts the "all heads" distribution in its own category with only one other distribution, "all tails".

But if we compare why you are still arguing over one in a thousand odds, but conceded the one in a gajillion odds of the coins landing in a straight line, it seems to come down to the fact that you can visualize one thousand, and so can discuss it in the realm of the "possible".

The chance of coins landing in a straight line is so small, that we think of it as impossible, even though it is not. We know that if we spent the day dropping ten coins and observing the pattern, that we would get all ten face up before the day was over. We might spend our lifetime doing this, and never once have them all land in a straight line.

So somewhere between the lower odds and the higher odds, you concede that Design becomes the more rational explanation.

As for me, I'll take that 998 out of 1000 chance of winning every day. Those odds are good enough for me. I will lose no sleep whatsoever on the 1 in 1000 chance that I was wrong.

This is practical. Let's say this is a test to determine whether I live or die. I will still bet my life on those odds. I will say that the coins were arranged, not simply dropped.

For if the coins were arranged, I have a 100% chance of being right. And if they were dropped, I have only 1 chance in 1024 of being wrong.

What man in his right mind wouldn't make decisions based on such favorable odds?

Gordon,

You refuse to acknowledge the simple fact that we instantly recognize, without thinking about it, that all heads only happens 1 time in 1000, and so is very unlikely to have happened randomly.

I do not recognize that.

You're simply wrong, Gordon.

Go ahead, bet your life. You'll die.

Phhht and Dean,

I wish you both the very best of luck in the improbable reality that you have chosen.

Read Chesterton's "Orthodoxy". Free your mind.

By Gordon (not verified) on 10 Mar 2015 #permalink

In reply to by phhht (not verified)

We do recognize the probability that the chance of landing all heads is 1 in 1024. Just as every other arrangement. The rest of your comment does not follow from that fact, only from your imagination. Your "argument" us simply a steaming pile of imaginary crap, just like everything else you've just as all of the talk of prophecies that have come true, and all the imaginary signs of end times

Gordon,

You never fail to disappoint. I was hoping that you would not think too deeply about my question, and you didn't. It would be difficult to modify the coin question appropriately without introducing some other "design" for you to cling to, so I'll try a different tact.

Suppose in the next Powerball drawing, the numbers drawn are 1,2,3,4, and 5 with 6 for the Powerball. Is this evidence that the lottery was rigged?

You almost got there in your post #449. That is how our mind works - we do intuitively conclude "design' in situations like the one I presented. That intuition often times leads us astray, however. Why is it that you think our intuitive understanding of nature must be infallible? Intuition has led us astray so many times over the course of human history that any argument based on our intuition is immediately suspect.

As a final challenge to see how your intuition does - let's change my original problem. Instead of one table, you walk into a room with 10000 tables. There are 10 pennies on each table. On exactly one table, all the coins are all heads. What are your thoughts?

Dean,

How about some citations from Chesterton that are clearly racist or bigoted.

And what sort of SAT score does it take to become a Math teacher these days? I'll put my SAT math score up against yours any day.

By Gordon (not verified) on 10 Mar 2015 #permalink

In reply to by Sean T (not verified)

Sean T,

I agreed with your point when Phhht insisted on repeating it again and again, as if it proved something.

Then I laid out exactly how my intuition would lead me to the correct answer almost every time. You guys have ignored that, and want to insist that the mere possibility that I could be wrong actually means something.

Intuition leads us to correct decisions in most cases. I have made better and better decisions the more I have learned to trust my intuition. An example like the coins I can explain in logical terms. I make intuitive decisions now that I could not even explain to you. Yet they are highly reliable.

One of the things I had to learn was to not worry about being wrong. Sure, I am wrong some of the time. But that is made up for by the overwhelming majority of times when my intuition proves to be correct.

The Powerball example is not as clear as the coins. And the 10,000 tables removes intuition altogether. There are too many possibilities to even begin to guess what is going on. Your coin example was very clear. I got your point straight away. But it had the unintended consequence of proving to be a stronger argument FOR Design than against it!

How about some obvious examples of how, "Intuition has led us astray so many times over the course of human history ..."?

And don't bother with the Geocentric universe example. If that had "led us astray", then NASA would not still use it to this day when calculating trajectories for launching spacecraft.

By Gordon (not verified) on 10 Mar 2015 #permalink

In reply to by Sean T (not verified)

I wish you both the very best of luck in the improbable reality that you have chosen.

The fact that your understanding of high school level math is deficient comes through each time you say something this stupid.

Read Chesterton’s “Orthodoxy”.

I stay away from the writings of bigots and racists. He was in the intersection of those groups.

You are conflating your personal bias for order when all coins are heads with some probabilistic meaning. That is pure garbage. As far as qualifications: I'm pretty sure that my qualifications exceed anything you have, given my career and experience, but that isn't the point. There is no mathematical reason for your comments about the reason all ten would be heads: it comes purely from the desire to see design in the world.

Look back at his writings about the "problems of the Jews" as seen by residents of the middle east, or his defense of Ford's anti-semitism as being "justified".

Or not. I know which route you'll take.

Dean,

I am interested in both subjects, but Chesterton was a voluminous writer, so a reference to which book or collection of articles would be helpful. I find it fascinating that you are even aware of such writings.

I am not questioning your academic credentials. You claimed my understanding of High School level math was "deficient", so I cited the standard test for proficiency in High School level math. I want to compare your proficiency with mine at that standard.

This is another test of my intuition. I am betting on the odds that my score is higher than yours, even though I can't possibly be certain. (If it was 800 I could be certain, but it is not). But the odds are heavily in my favor, so I have stuck my neck out at the risk of being proven wrong.

You are the one making wild, unsubstantiated claims. Time to see whose claims actually hold up when tested!

Gordon,

Thank you for making my point. My last example with the 10000 tables shows that intuition is a poor guide. Your intuition was unable to tell you anything, whereas a mathematical analysis would show that such an arrangement actually has very low probability of occurrence. It would be rational to conclude that this situation was designed! There would only be about a 1 in 1600 chance that randomly placed coins on 10000 tables would be all heads only one time. That's certainly not to say it's guaranteed that this came about by design; it could have merely been an improbable occurrence. It is to say that your intuition is lacking.

That's pretty much been the whole history of science if you are familiar enough with it. Intuition is not a good guide to the way the universe works. Think about it, in antiquity there was no real reason to believe that the earth rotates, for instance. The "intuitive" model would be that the earth is motionless with the sun and moon revolving around it. Obviously, the counterintuitive explanation of the earth rotating is correct, though.

Other examples abound. These are really the way that scientific theories gain acceptance; they make counterintuitive predictions that turn out to be correct. One of my favorites was during the debate about the nature of light, proponents of the particle theory pointed out that if the wave proponents were correct, there should be a bright spot at the center of the shadow of a sufficiently small circular object placed in a light beam. Everyone scoffed at that counterintuitive result and it almost resulted in the death of the wave theory, until, of course, the bright spot was found at the center of the shadow right where it was predicted.

Another example. If a baseball player throws a ball at 70 mph on a train moving 70 mph in the same direction, a person on the ground would measure the ball with what velocity? Intuition tells us that the correct answer is 140 mph, and intuition in this case would be pretty good. However, change these velocities to say 100,000 miles per second, and intuition fails.

Think about it, if intuition were such an infallible guide, what would be the purpose of science and mathematics? We wouldn't need these fields of study to understand the universe if intuition were good enough. Development of science and mathematics occurred precisely because intuition was found wanting.

Sean T,

Yes Sean, there are limits to the usefulness of Intuition, just as there are limits to the usefulness of Science and even Math. Witness your silly appeal to mathematics in disputing the perceived size of the Sun and Moon.

The two are not mutually exclusive, nor is one necessarily a better guide than the other. It depends upon the circumstances. I use both, and sometimes find that one can confirm the other, as a second witness, such as in the coin example.

Yes, based on the mathematics I knew that the 10,000 tables were most likely designed. But only math could tell me this, and I happily admit the limits of Intuition in a scenario such as that. But it is funny to hear you assert that "it is rational to assume that the situation was Designed."

Of course that is the rational conclusion. Why is that rational, while my conclusion in the coin example is not? You have a bias against Intuition, and that bias is supreme EVEN when the mathematics PROVE the Intuition. This is the part I find bizarre about your thinking.

Mathematics has always been my strong point, and so I agree completely with you that Intuition fails as the numbers get larger. That is what I loved about the ten coins example. It works for both Intuition and Math. They point to the same conclusion. And think about it. We can experience the train going 70 mph with the baseball player throwing a ball at 70 mph. Under what real life circumstances would we experience the two 100,000 mph's where Intuition failed us?

So in everyday experience we find that Intuition is a good guide most of the time. You could not help yourself but appeal to the Geocentric model, and yet that model is such a reliable guide that it remains the preferred model used by NASA to calculate trajectories for any spacecraft launched from Earth. So has Intuition failed, or proven to be a more reliable guide than Science in that situation?

You can theorize all you want, and talk about what is "technically" correct. But at the end of the day, we need to make decisions that achieve our objectives, and we need the best information for making those decisions. If Intuition is good enough for the Space program, how come it is not good enough for you?

The only so-called "Science" that I reject is the purely speculative stuff that contradicts things I know to be true. While you go out of your way to reject Intuition, it is a complete misrepresentation to say that I reject Science. Each one has their place, and both are useful, in my view. And both of them have their limits.

By Gordon (not verified) on 10 Mar 2015 #permalink

In reply to by Sean T (not verified)

"I want to compare your proficiency with mine at that standard."

I realize that your view is that argument from authority matters (even though it is a foolish point).
My high school days are far behind me - I no longer remember which test I took, and am certain I don't have records.
but I do have a Master's Degree in math, another in statistics, and a Ph.D. in statistics. I have a very good grasp of the likelihood of coins falling, of what can be concluded, and what is purely fantasy. You are dealing purely with fantasy in your design comments.

BTW Gordon, it's not just in the realm of science where our intuition is lacking. Mathematics contains quite a few results that are counterintuitive, especially mathematics involving exponential growth, large numbers, infinity, and probability. For instance, the Monty Hall Problem has been well discussed on this blog. If you aren't familiar, the problem is derived from the game show "Let's Make a Deal". In it a contestant is given a choice of three doors. Behind one is a car and goats behind the other two. If the door that the contestant selects has a car behind it, he wins the car. After selecting a door, Monty shows a goat behind one of the other two doors and asks the contestant if he wants to switch doors. Should he switch? Intuition, even among those with a good deal of mathematical knowledge, is historically a VERY poor guide to this one.

Large numbers also pose a challenge to intuition. I had a professor in a math class way back when I was in college who drew a line on a chalkboard that was one meter long. He labeled the left end 0 and the right end 1 trillion. Then he asked students to label the points corresponding to 1 billion and 1 million on that line. Almost nobody got it correct intuitively. Just using your intuition, where on that line would you place 1 billion and 1 million?

Infinities are also notorious in this regard. For instance, which set is larger: the set of all integers or the set of all numbers between zero and one? The fact that some infinities are larger than others is counterintuitive to begin with, but I assure you that the logic of set theory is sound in that regard. Some infinities are larger than others. Intuitively, then, which is larger the set of all integers or the set of all numbers between zero and one? Another quick one: which is larger, then number of points on a line or the number of points in a 2-dimensional plane? I am pretty sure that intuition will not get you the right answer on these.

Finally exponential growth is very difficult to grasp intuitively. For instance, suppose that there is a bacterial colony living in a coke bottle. The colony reproduces (as bacteria do) by splitting in half. In this colony, each one splits once per minute. Unfortunately for these bacteria, at midnight, they will occupy the entire space of the bottle. Fortunately, a couple of intrepid explorers went out and found three new coke bottles! That's four times more space than the bacteria have ever had! Surely, the colony is saved now. At what time will the bacteria run out of space (assuming that the three new bottles are the same size as the original one)?

It seems that you are rejecting scientific results because they run counter to your intuition. I am not trying to pick on you or embarrass you here. I just am trying to make you see that conflict with intuition does not make a scientific result any less likely to be true. Intuition is often a very imperfect guide.

Gordon,

I scored perfectly on my SATs, both verbal and mathematical. I am also a genius, according to high school IQ testing.

You?

"It seems that you are rejecting scientific results because they run counter to your intuition"

He seems to think particular results, because of how they appear to us, indicate influence from outside, even though simple chance is sufficient for their explanation.

Intuition is of course a great thing until it comes up against facts. Let's play along with Gordon and Phil and take Genesis as a basis for ID. They wants us to accept that their God said some words and POOF! something came from nothing and from this something this God fashioned living things. Genesis 2 claims that this God took a handful of dust from the ground and shaped humans. Intuition would tell us that human chemical composition should be the same as soil at best and rock dust at worst (the earth was young and all and soil takes a long time to form). So why aren't we primarily Silicon dioxide and Aluminum tetraoxide? How does life become Carbon-based? Bone isn't even made from Silica. That's just on a basic level without going into fashioning cells, tissues, organs and organ systems from a recipe that reads like how to make a clay figurine.

Then there is the folk-taxonomy of grouping on locomotion type rather than other features - bats and whales grouped with flying and swimming things and created a day earlier than the majority of mammals. What do creationists make of monophyletic group called mammals? What was this God's reason for making bats have wings, but being a mammal in every other feature. The same for whales? I don't get how intuition gets one to intelligent design given what we know about biology. Maybe 3000 years ago, but now?

By Michael Fugate (not verified) on 10 Mar 2015 #permalink

"If that had “led us astray”, then NASA would not still use it to this day when calculating trajectories for launching spacecraft."

NASA uses a variety of coordinate methods depending on the mission. For close to the Earth geocentric coordinates give reasonable results. As missions move farther from earth different systems are used. Between planets - heliocentric, near Mars or other planets systems centered there are used.

The baloney-backed creationist line "they use geocentric idea on their missions" is an intentional lie by partial truth: by failing to give the full description the implication is NASA's scientists are supporting the complete validity of geocentrism while being quiet about it. Dishonest comment, but not surprising.

Phhht,

You are also obviously smart enough not to make the silly claim that Dean made about my level of proficiency in math.

Dean can't remember his score, or even what test he took, so he obviously was not exceptional. I was top of my class, but not perfect. And yes, a perfect score on the SAT does put you in the category of genius.

Dean,

You are funny! A Math Professor who wants to dismiss standardized testing!!! Upon what basis do you determine that ANYTHING is true?

I supposed you start each term by informing your class that the answers they give on their math tests are irrelevant.

The Geocentric model gives the BEST results for calculating trajectories for any spacecraft launched from Earth, which as of today is ... ALL of them. It has fewer variables and therefore less opportunity for error. Once clear of Earth and influenced by the gravity of other planets, it makes sense that other models become as good or even better.

But you have only confirmed the point that NASA does indeed prefer the Geocentric model for "close to the Earth". It is a very reliable model, irrespective of whether or not it is technically precise. And remember, this is a discussion of how reliable our Intuition is. The point being, even when our Intuition is "proven" to be completely wrong by Science, it can still prove to be more useful than the scientific model.

As for your own "intentional lie by partial truth", the Wikipedia article on Chesterton does a fair job of describing his views on the Jews. It is clear that he stated his opinion on all matters, and that led him to say negative things about the Jews when they properly applied, and also to defend the rights of the Jews with consistency.

The only difference is, in his day stating one's opinion did not attract an avalanche of attack from the Jews the way it does today. Today, as you so aptly illustrate, a mere whisper of negative opinion is automatic grounds for character assassination.

How about returning to your roots? Your ancestors wrote an amazing book!

I'm not sure where I dismissed standardized testing, although a very good care can be made that many standardized tests are good at assessing test taking and nothing else. Read again what I said.
And I'm not surprised that you twist the comment about NASA using different systems for its missions. Blatantly dishonest of you to continue to ignore the fact that they use different systems for different purposes and claim (falsely) that they continue to use it everywhere. That is simply not as I pointed out.
Nor is it a surprise you are willing to tolerate a racist.

I'm not sure how others can continue to tolerate the stench of ignorance and dishonesty you and phil emit. Searching for an honest statement or a bit of intelligence from you is a waste of time.

Gordon,

You have become dishonest and you have moved your goalposts. I believe I am done discussing this with you.

You are dishonest when you claim that NASA uses intuition in its calculation of orbits. It does NOT use intuition; it uses a coordinate system centered upon the earth for low earth orbit missions. This is just as mathematically based as any other coordinate system. They use it because it is a good enough approximation for the task at hand and because the computation in that system is simpler than in other coordinate systems. That's valid science, but it's not just intuition. As an analogy, do architects routinely account for the curvature of the earth when designing buildings? Of course not; to a close enough approximation, the earth is flat, at least over the amount of surface that a building will occupy. The situation is similar with respect to the space program in terms of near-earth missions. To make a map of a large area of the earth, though, you need to account for the curvature of the earth, and in similar vein, to plan a space mission ranging further from the earth, a geocentric system is no longer adequate.

You moved the goalposts also. You made the claim that (post #481) your intuition is correct in an overwhelming number of cases. Now you are backing off of that and claiming (post #490) that both science and intuition have their limits. So which is it? Is your intuition limited or is it overwhelmingly correct?

Also, what difference does it make whether or not a counterintuitive result is one that you would typically encounter in everyday life? Our intuition was formed precisely under circumstances encountered generally in everyday life. It would be surprising if our intuition didn't work well under such circumstances. Everyday circumstances, however, is not the entirety of the universe. (BTW, my velocity example happens all the time with respect to particle decay experiments). Are we not to study the entire range of phenomena in the universe, or at least as much of that range as we can? Why should we limit ourselves to everyday life? Just as a practical matter, keep in mind that we would not be communicating in this manner if science did limit itself to everyday life and only followed what was intuitive. The transistor, which is the basis of all computers, is a device that was only made possible by the development of quantum mechanics. Believe me, in no realm of science is intuition any more useless than in quantum mechanics.

You challenged me to provide examples where intuition has failed. I provided such examples, and you basically fluffed them off as somehow irrelevant. I am not biased against intuition; reality seems to be, though, at least at a fundamental level. I have shown you time and again situations where intuition fails.

Finally, you claim "The only so-called “Science” that I reject is the purely speculative stuff that contradicts things I know to be true." What you fail to realize is that this statement indicates that you actually do reject science in its entirety. All science is speculative. There really is no such thing as "settled science". At any time an observation could be made that overturns any given scientific theory. It happened to Newtonian mechanics, for instance, after more than three centuries of success of that model. We cannot reject relativity because it conflicted with Newtonian mechanics ie. "what we know to be true". To accept science, you must accept the idea that "what you know to be true' may in fact be untrue. That is the essence of science. The fact that there are some beliefs that you are unwilling to even consider abandoning indicates that you do in fact reject science.

I am not trying to be impolite, but I come to the conclusion that we are merely talking past each other here. I accept science. You do not. I regard intuition as a guide that can help us in everyday situations such that we don't need to do complicated and difficult scientific and mathematical calculations. You regard intuition as some near-infallible guide to the basic and fundamental working of the universe. We are coming from completely different places here. No further discussion would be profitable. Good luck; I wish you well.

"You are dishonest when you claim that NASA uses intuition in its calculation of orbits."

Indeed he is - blatantly dishonest, as in every other topic he's discussed. But it makes you wonder whether
* he accepts the geocentric model as correct
* he is a relativity denier

It speaks volumes about him that it is very easy to believe the answer to both of those would be yes.
Again - good luck to those who try to get any honest, reality-based comments from him or phil. Their self-imposed ignorance is incredibly powerful.

Sean T and Dean,

The only lies going on here are the ones you are telling yourselves.

Were you able to be objective, you would have no need to twist what I say.

But it was fun while it lasted. God bless.

By Gordon (not verified) on 11 Mar 2015 #permalink

In reply to by dean (not verified)

Gordon, the only lie is that the Bible was written or dictated by a God instead of written by humans without supernatural intervention. Your intuition and the intuition of your ancestors is all over the creation story which you and Phil believe to be literally true. Humans die and turn to dirt, therefore God the designer/creator picked up some dirt and fashioned a human. A human wouldn't know that dirt wasn't of what we are comprised, but the God would. A designer/creator would know exactly what was needed and it wasn't dirt. The same with the emphasis on breath and the whole spirit thing. Wrong again.

By Michael Fugate (not verified) on 11 Mar 2015 #permalink

"Were you able to be objective, you would have no need to twist what I say."
Pointing out your lies is not twisting what you say. Latest evidence: your comment

The Geocentric model gives the BEST results for calculating trajectories for any spacecraft launched from Earth, which as of today is … ALL of them.

That is, as has been pointed out at least two times now, completely false. It gives a usable approximation for near Earth missions and is discarded for longer missions. It's possible you don't realize you are lying, because you are convinced that everything you say is correct.

Your comments about your math skills sound hollow (and frankly, I don't believe your "top of your class claim", since your comments about science and math indicate you have no understanding of either.) You clearly don't understand why probability alone is sufficient to explain the coins, and present your personal bias about design as somehow representing something meaningful in that context. Your view is a disturbingly weird as the video that floated around a couple years ago of the poor soul claiming to be amazed as a double rainbow. That was a monumental expression of inability to understand, and I hadn't thought I would see its equal. Then you (and phil, and sn) came along to make that person look sane in comparison.
You've chosen to refute modern science, for whatever reason we can't know. That's your choice. But don't expect to blatantly misrepresent what science says, what it predicts, and do the same with math, and not be called on it. Normal people aren't as comfortable with the foundation free crap from creationists as are other creationists.

Just a brief drop-in: like one of the Seans, I have been swamped with other business. Sorry if I left anyone's question or comment hanging, I will likely be very light on commenting for the next week or two.

"You’ve chosen to refute modern science, for whatever reason we can’t know."

Actually, we're refuting modern materialism on the basis of modern science. Proteins are synthesized in living cells. There is no other known source.

Phil,

Another classic quote from Chesterton:

"The materialists analyze the easy part, deny the hard part and go home to their tea."

Pretty well sums up all the exchanges here.

By Gordon (not verified) on 11 Mar 2015 #permalink

In reply to by Phil (not verified)